Sei sulla pagina 1di 112

Universidad La Salle.

Facultad Mexicana de Medicina.


Curso de Extensin Universitaria para la Preparacin del Examen Nacional para
Aspirantes a Residencias Mdicas.
Examen Mdulo V Subespecialidades.

Examen del Mdulo V.


24 de Agosto del 2011.

1.- Una nia de 9 aos presenta astenia, adinamia, decaimiento y prdida de 3 kilogramos
de su peso habitual. Desde hace 15 das ha padecido fiebre de 39C, disfagia y epistaxis. Al
explorarla se encuentra plida, con adenomegalia cervical y axilar; se palpa
hepatoesplenomegalia y se observan quimosis en las piernas.
El diagnostico ms probable es:

a)
b)
c)
d)

Mononucleosis infecciosa.
Linfoma de hodgkin.
Tuberculosis extrapulmonar.
Leucemia aguda.

La EH se presenta habitualmente con adenopatas cervicales o supraclaviculares, indoloras,


ms consistentes que las inflamatorias y de lento crecimiento.
Al menos, dos tercios de casos asocian adenopatas mediastnicas.

Sntomas sistmicos:
Sntomas sistmicos no especficos pueden ser fatiga, anorexia y ligera prdida de peso.
Sntomas especficos (fiebre inexplicada, prdida de peso de 10% en los 6 meses previos y
sudor nocturno abundante) tienen significado pronstico.
Algunos pacientes tienen prurito.
Exmenes de laboratorio
En el hemograma, se observa leucocitosis con neutrofilia, linfopenia, eosinofilia y
monocitosis. La anemia puede indicar la presencia de enfermedad avanzada.
Se ha observado anemia hemoltica asociada a EH, con test de Coombs positivo,
reticulocitosis e hiperplasia normoblstica de mdula sea. La asociacin de prpura
trombopnica idioptica (PTI) tambin ha sido referida y la respuesta al

tratamiento de la PTI va a depender del estadio en que se encuentre la EH. La elevacin de


VSG, cupremia y ferritinemia reflejan la activacin del sistema mononuclear fagoctico, que
puede servir como test inespecfico para control de seguimiento de la EH. Las anomalas del

sistema inmune en el momento del diagnstico de EH pueden persistir durante y despus


del tratamiento.

1.*** Fernndez-Teijeiro A. Enfermedad de Hodgkin. En: Madero Lpez L y Muoz


Villa A editores. Hematologa y Oncologa peditricas. Madrid: Ergon; 1997.
p. 467-82.

El autor hace un recuerdo histrico de EH, sealando aspectos epidemiolgicos, etiolgicos


e histolgicos, resaltando la presentacin clnica y el diagnstico diferencial, as como los
estudios de diagnstico. Describe con amplitud aspectos teraputicos de radioterapia,
quimioterapia y de efectos secundarios del tratamiento.
2.* Ferrs Tortajada J, Garca Castell J, Berbel
Tornero O, Clar Gimeno S. Factores de riesgo para los linfomas no hodgkinianos.
An Esp Pediatr 2001; 55: 230-8.
Exposicin de factores de riesgo para LNH, revisados de la literatura mdica, con el
objetivo, segn los autores, de divulgar entre los pediatras dichos factores de riesgo.
3.* Ferrs Tortajada J, Garca Castell J, Lpez Andreu JA, Berbel Tornero O, Clar
Gimeno S. Factores de riesgo para los linfomas de Hodgkin. An Esp Pediatra
2001; 55: 239-43.
Exposicin de factores de riesgo para EH, revisados de la literatura mdica, con el
objetivo, segn los autores, de divulgar entre los pediatras dichos factores de riesgo.

2.- Paciente con infeccin por VIH, diagnosticado hace 2 aos, asintomtico. Acude a
consultarlo porque no ha podido ir con el infectlogo que lo ha revisado en los ltimos 6
meses y quien haba solicitado como control una carga viral que se report con 100,000
copias, y una cuenta de CD4 de 33 clulas. Usted le recomienda.
a)
b)
c)
d)

Inicio de tratamiento con AZT y 3TC.


Ir con su infectlogo.
Repetir el estudio.
Iniciar TMP/SMZ y claritromicina.

El conteo de CD4 tan bajo pone al paciente en riesgo de desarrollar un evento por
infeccin oportunista potencialmente mortal en el siguiente ao por arriba del 80%. Es
indispensable, en tanto recibe atencin por el especialista, iniciar tratamiento profilctico
contra Pneumocystis jiroveci y Mycobacterium avium-intracellulare.
Kasper DL, Braunwald E, Fauci AS, Hauser SL, Longo DL, Jameson JL. Harrisons
Principles of Internal Medicine. McGraw Hill. 16 Ed. 1076 p.

3.- Se trata de masculino de 40 aos de edad con leucemia mieloblstica aguda recibi su
tercer ciclo de quimioterapia. Una semana despus inicia con fiebre y un sbito
empeoramiento del estado general. Al realizar una exploracin fsica se presenta una lesin
nodular equimtica y dolorosa, con centro ulcerado y rodeada de eritema y edema, en el
MPI. Present menos de 100 leucocitos/mm3, hemoglobina 7 gr/dL y 30.000
plaquetas/mm3. El germen que es aislado con mayor frecuencia en un caso como ste es?

a) Aeromonas hydrophila.
b) Klebsiella pneumoniae.
c) Staphyloccocus aureus.
d) Pseudomonas aeruginosa.

Pseudomonas aeruginosa (o Pseudomonas pyocyanea) es una bacteria Gram-negativa,

aerbica, con motilidad unipolar.1 Es un patgeno oportunista en humanos y tambin en


plantas.2
Como otras Pseudomonas, P. aeruginosa secreta una variedad de pigmentos como piocianina
(azul verdoso), fluorescena (amarillo verdoso fluorescente) y piorubina (rojo pardo). King,
Ward, & Raney desarrollaron "Pseudomonas Agar P" (tambin conocido como "medio King A")
para mejorar la produccin de piocianina y piorubina; y "Pseudomonas Agar F" (tambin
conocido como "medio King B") para la fluorescena.3
Este patgeno oportunista de individuos immunocomprometidos, P. aeruginosa infecta el
tracto pulmonar, el urinario, tejidos, heridas, y tambin causa otras infecciones de sangre
Pseudomonas puede causar neumonas a grupos, necesitando a veces ayuda mecnica para
superar dichas neumonas, siendo uno de los ms comunes agentes aislados en muchos
estudios. La piocianina es un factor de virulencia de la bacteria y se ha conocido que puede
hasta causar muerte en C. elegans por estrs oxidativo. Sin embargo, la investigacin indica
que el cido saliclico puede inhibir la produccin de piocianina. Uno en diez hospitales se
infectan con pseudomonas. La fibrosis qustica est tambin predispuesta a la infeccin con

P. aeruginosa de los pulmones. P. aeruginosa es el causante de dermatitis, causada por


disminucin del control de la calidad del agua de bebida. El ms comn causante de altas
fiebres en infecciones es P. aeruginosa. Tambin ha estado involucrado en foliculitis de
tinas de agua caliente, en especial aquellas sin un control higinico continuo.8

Ryan KJ; Ray CG (editors) (2004). Sherris Medical Microbiology, 4th ed. edicin,
McGraw Hill ISBN 0-8385-8529-9..
2. Iglewski BH (1996). Pseudomonas. In: Baron's Medical Microbiology (Barron S et al,

1.

eds.), 4th ed. edicin, Univ of Texas Medical Branch (via NCBI Bookshelf) ISBN 09631172-1-1..

3. King EO, Ward MK, Raney DE (1954). Two simple media for the demonstration of
pyocyanin and fluorescin. J Lab Clin Med. Vol. 44. n. 2. pp. 301-7. PMID 13184240.
4. Todar's Online Textbook of Bacteriology
5. Fine et al, JAMA 1996: 275: 134
6. Diekema DJ et al. Clin Infect Dis 1999;29:595
7. Prithiviraj B, Bais H, Weir T, Suresh B, Najarro E, Dayakar B, Schweizer H, Vivanco
J (2005). Down regulation of virulence factors of Pseudomonas aeruginosa by salicylic
acid attenuates its virulence on Arabidopsis thaliana and Caenorhabditis
elegans. Infect Immun. Vol. 73. n. 9. pp. 5319-28. PMID 16113247.
8. MedlinePlus - Enciclopedia Mdica: Foliculitis de la tina.

4.- Una mujer de 36 aos es valorada porque presenta dolor de garganta y adenopata
cervical. Su temperatura es de 37C., pulso de 90/min. y presin arterial de 110/70 mmHg.
El ganglio cervical anterior derecho mide 2.5 x 3 cm. No hay adenopata supraclavicular,
axilar, epitroclear o inguinal palpable. La exploracin abdominal no es relevante. En los
siguientes seis meses la paciente es revalorada por infecciones recurrentes de las vas
respiratorias superiores. Durante este perodo hubo alguna regresin de ganglios
cervicales, pero al parecer el tamao aumenta y disminuye en respuesta a los antibiticos.
Por su adenopata persistente, la enferma es enviada con un cirujano y se realiza una
biopsia del ganglio. Cul de los siguientes describe la histopatologa correspondiente al
diagnstico de la enferma?

a) Linfoma folicular de clulas pequeas hendidas.


b) Linfoma de clulas grandes difusas.
c) Linfoma inmunoblstico.
d) Ganglio linftico normal.

Allen R. M. MMS Medicina Interna. 5. Edicin. National Medical Series. Mc. Graw Hill.
2006. (captulo 4 XVI C 2, D 2). El linfoma folicular de clulas hendidas pequeas, llamado
antes linfoma linfoctico nodular mal diferenciado, es una de las variedades ms comunes de
linfomas no hodgkinianos indolentes. A menudo los pacientes presentan enfermedad en
etapa III o IV. Aun sin tratamiento, los individuos pueden tener adenopata que aparece y
desaparece, pero con el tiempo la enfermedad progresa y se requiere quimioterapia. Un
pequeo porcentaje de sujetos presenta un linfoma ms agresivo, casi siempre de
variedades de clulas grandes o inmunoblsticos.

5.- En la hiponatremia intensa (inferiores 120 mEq/L se debe de tratar con soluciones
salinas hipertnicas mediante el calculo del dficit, reponiendo de la siguiente manera:
a)
b)
c)
d)

Pasar el dficit de sodio en las primeras 6 hr.


Pasar el dficit de sodio en las primeras 12 hr.
Pasar el dficit de sodio, la mitad en las primeras 12-24 hr y se revalora.
Pasar el dficit de sodio en las primeras 3 hr y despus se revalora.

Farreas. Medicina Interna. Cap. 222, Decimoquinta edicin 2004, Pp. 1844-1847. La
hiponatremia con volumen extracelular disminuido se trata con la administracin de
soluciones salinas isotnicas (al 0.9%). La cantidad necesaria de miliequivalentes de sodio se
calcula de acuerdo a la siguiente formula:
Na (mEq)= Na x agua total corporal, es decir
Na (mEq)= (140- Na actual) x (0.6 x peso en Kg)
Donde:
Agua total corporal= 60% del peso = 0.6 x peso en kg
En la practica, suele administrase la mitad de esa cantidad en el transcurso de las primeras
12- 24 h y luego se efectan reevaluaciones correctoras.

6.- Su frecuencia es de 1/10,000, bajo peso al nacimiento, LPH (Labio paladar hendido)
microftalmia, holoprosencefalia, aplasia cutis vertex, riones poliqusticos, malrotacin del
colon, y en nias puede existir tero bicorne, son manifestaciones de:

a)
b)
c)
d)

Trisoma 21
Trisoma 13
Trisoma 18
Trisoma 11

La trisoma 13 representa la tercera aneuploida autosmica viable ms frecuente en la


especie humana, slo superada en frecuencia por la trisoma 21 y trisoma 181. Aunque su
etiologa es an desconocida, estudios citogenticos en individuos afectados han podido
determinar que aproximadamente un 75% de los casos corresponden a no-disyuncin
meitica, un 20% a translocaciones y un 5% a mosaicismo debido a nodisyuncin postcigtica.

La expresin fenotpica de la trisoma 13 es caracterstica y consiste en anomalas faciales,


esquelticas y del sistema nervioso central, siendo tambin frecuentes las malformaciones
estructurales del aparato cardiovascular, genitourinario y gastrointestinal
(Tabla I).1 La mayora de los defectos estructurales asociados a trisoma
13 pueden ser identificados prenatalmente por ultrasonografa.2-6 Esto permite ofrecer
estudio citogentico prenatal en casos seleccionados, establecer el diagnstico definitivo y
manejar el embarazo afectado en forma racional, ya que esta anomala se asocia a una
mortalidad perinatal cercana al 100%.

Bibliografa:
1. Jones KL. Smiths Recognizable Patterns of Human Malformation. 5th Edition.
WB Saunders, Philadelphia. 1997, pp 18-23.
2. Benacerraf BR, Miller WA, Frigoletto FD. Sonographic detection of fetuses with
trisomies 13 and 18: accuracy and limitations. Am J Obstet Gynecol 1988; 158:404-9.
3. Dicke JM, Crane JP. Sonographic recognition of major malformations and aberrant
fetal growth in trisomic fetuses. J Ultrasound Med 1991,10:433-8.
4. Nicolaides KH, Snijders RJ, Gosden CM, et al. Ultrasonographically detectable markers
of fetal chromosomal abnormalities. Lancet 1992;340:704-7.
5. Lehman CD, Nyberg DA, Thomas CW, et al. Trisomy 13 syndrome: prenatal US findings
in a review of 33 cases. Radiology 1995;194:217-22.
6. Seplveda W. Marcadores sutiles de anomalas cromosmicas en el feto: deteccin
ultrasonogrfica, significado clnico y manejo. Rev Chil Obstet Ginecol 1996;61:395-403.
7. Seplveda W, Be C, Bravo M. Anomalas cromosmicas en el primer trimestre del
embarazo: Correlacin entre hallazgos ultrasonogrficos y cariograma por biopsia de
vellosidades coriales. Rev Chil Obstet Ginecol 1997;62:268-74.

Lectura recomendada:
Revista Chilena de Ultrasonografa. Vol 2/N1/1999 Rendimiento de la translucencia nucal
en 120 biopsias de vellosidades coriales
Trisoma 13: Diagnstico citogentico prenatal y hallazgos ultrasonogrficos
Trisoma 13: Diagnstico citogentico prenatal y hallazgos ultrasonogrficos.

7.- Se trata de paciente femenino de 28 que acude por presentar en la mueca una lesin
ppuloescamosa, pruriginosa, violcea y poligonal, con una red de lneas grisceas (estras
de Wickham). Esta lesin es compatible con:
a) Pitiriasis rosada.
b) Liquen plano.
c) Tia versicolor.
e) Melanoma.

Liquen plano
El liquen plano es una enfermedad dermatolgica consistente en una erupcin papular,
prurtica, inflamatoria, de curso crnico que afecta a la piel y mucosas. Los cambios clnicos
y histolgicos son caractersticos de esta enfermedad, pudindose presentar con diversas
formas clnicas. El liquen plano afecta a 1,5-5% de los pacientes de consultas externas de
un servicio de dermatologa. Es de causa desconocida, existen datos que muestran que el
liquen plano representa una reaccin autoinmune, mediada por los linfocitos T y dirigida
contra los queratinocitos basales que expresan autoantgenos en su superficie que han sido

modificados por diversas causas como son infecciones virales (hepatitis C),
medicaciones, alergenos de contacto, neoplasias internas. Esta reaccin de los linfocitos T

contra los autoantgenos modificados por los agentes externos se vera seguida de la
reaccin cruzada frente autoantgenos previamente ignorados dando lugar a una
perpetuacin de la reaccin autoinmune mediada por los linfocitos T que caracteriza el
liquen plano. La lesin clnica elemental cutnea es la existencia de una ppula poligonal,
brillante, con una depresin central cubierta de una escama blanquecina que deja entrever
unas lneas blanquecinas que se conocen como estras de Wickham. Las lesiones se
distribuyen de forma simtrica afectando a las caras extensoras de las extremidades,
especialmente en las muecas, regin lumbar, cara lateral de cuello y rea genital. La mayor
parte de los pacientes refieren un inicio brusco de su enfermedad, que se desarrolla en un
perodo corto de unas semanas. El prurito suele ser muy intenso. El liquen plano, al igual que
la psoriasis, desarrolla el fenmeno de Koebner, siendo frecuente el desarrollo de nuevas
lesiones en reas de rascado o traumatismo. Las lesiones de liquen plano curan
espontneamente o con tratamiento dejando una hiperpigmentacin residual que es ms
intensa en fototipos de piel IV, V y VI. Existen muchas variantes clnicas de liquen plano
que varan con relacin a la zona de aparicin y su morfologa.

Bligard CA, Millikan LE. Acute exanthems in children. Postgrad Med 1986;79:15067.
Bodemer C, de Prost Y. Unilateral laterothoracic exanthem in children: a new
disease? J Am Acad Dermatol 1992;27:693-96.
Drago F, Rampini P, Rampini E, Rebora A. Atypical exanthems: morphology and
laboratory investigations may lead to an aetiological diagnosis in about 70% of
cases. Br J Dermatol 2002;147:255-60.
Frieden IJ. Childhood exanthems. Curr Opin Pediatr 1995;7:411-14.
Gelmetti C, Caputo R. Asymmetric periflexural exanthem of childhood: who are
you? J Eur Acad Dermatol Venereol 2001;15:293-94.

8.- Masculino de 25 aos con un gran melanoma maligno en la espalda. No hay


linfadenopata aparente por lo que est en estadio 1. Cul es el factor pronstico ms
importante en su caso?

a) Grosor del tumor.


b) Dimetro del tumor.
c) Localizacin del tumor.
d) El gnero del paciente.

Factores de riesgo:
{ Exposicin solar.
{ Nmero de nevos (5-6 =3 veces + riesgo)
{ Nevos congnitos.
{ Historia previa de melanoma.
{ Historia familiar.
{ Transplante e inmunosupresin.
{ Xeroderma pigmentoso.

Ante la presencia de una lesin pigmentada presente desde el nacimiento o de nueva


aparicin, debern de tenerse en cuenta una serie de parmetros clnicos (Regla: ABCDE)
{
{
{
{

A = ASIMETRIA
B = BORDE
C = COLOR
D = DIAMETRO.

FACTORES PRONOSTICOS. MM E.C. I Y II.


{
{
{
{
{

GROSOR.
Fx Px ms importante de sobrevida.
Correlaciona con riesgo de recurrencia local.
Riesgo de metstasis en transito.
Determina mrgenes de reseccin quirrgica.

1. Fitzpatrick TB, Jonson RA, Wolf K, Suurmond D. Color Atlas and Synopsis of Clinical
Dermatology. Ed. McGraw-Hill-Interamericana, 2001.
2. Malvehy J, Puig S, Mart R, Castel T, Mascar JM. Melanoma maligno cutneo (I):
epidemiologa, patogenia y diagnstico. Med Cutn Iber Lat Am 1998;6:283-99.
3. Prorok PC, Hankey B, Bundy B. Concepts and problems in the evaluation of screening
programmes. J Chronic Dis 1981;34:159-71.
4. Spratt JS. Epidemiology of screening of cancer. Cancer 1982;6:1-58.
5. Kopf AW, Rigel DS, Friedman RJ. The rising incidence and mortality rates of malignant
melanoma. J Dermatol Surg Oncol 1982;8:760-1.
6. Schreiber MM, Bozzo PD, Moon TE. Malignant melanoma in southern Arizona: increasing
incidence ans sunlight as an etiologic factor. Arch Dermatol 1981;117:6-11.
7. Makie RM. Secondary prevention of malignant melanoma. Pigment cell 1999;11:22-30.
8. Setlow RB, Woodhead AD. Temporal changes in the incidence of malignant melanoma:
explanation from action espectra. Mut Res 1994;307:365-74.

Lectura:
Melanoma maligno: Consideraciones sobre su diagnstico precoz, prevencin y tipos clnicos.
Dermatlogo
Correspondencia:
Xavier Jeremas Torruella
Hospital del Mar
Servicio de Dermatologa. Facultad de Medicina
Universidad Autnoma de Barcelona
Passeig Martim, 25. 08003 Barcelona

9.-Masculino de 35 aos con datos clnicos de importancia para diagnstico de leo mecnico
simple, los datos radiogrficos de esta patologa son:
a) Retencin de contenido en colon, sin niveles hidroaereos.
b) Liquido y aire libres en cavidad peritoneal.
c) Niveles hidroareos a diferentes alturas, con dilatacin de asas.
e) Asas fijas y lisas, con niveles largos.

ILEO MECANICO
A/ Extraluminal
Adherencias postquirrgicas (Causa ms frecuente 35-40%)
Hernias externas (inguinales, crurales, umbilicales, laparotmicas, etc.)
Hernias internas
Torsiones
Vlvulos
Invaginaciones
Efecto masa extraluminal (tumoracin, masa inflamatoria abceso)
B/ Parietal
Neoplasias
Alteraciones congnitas (atresias, estenosis, duplicaciones, etc.)
Procesos inflamatorios (Crohn, postradiacin, etc.)
C/ Intraluminal
Ileo biliar
Bezoar
Parasitosis
Cuerpo extrao
Impactacin fecal

Tumoraciones

leo mecnico
Los signos radiolgicos de una obstruccin intestinal simple son:

Asas distendidas proximales por retencin de lquido y gas.


Niveles hidroareos.
Reduccin o ausencia de gas y materia fecal en colon. Ante el hallazgo de gas en
colon se descarta la existencia de una obstruccin de intestino delgado.

BIBLIOGRAFA:
1) Di Lorenzo C. Pseudo-obstrucction. Current approaches. Gastroenterology 1999;116:980987.
2) Stanghellini V, Cammilleri M, Malagelada JR.Chronic idiopatic intestinal
pseudobstruction: clinical and intestinal manometric findings. Gut 1987; 28: 5-12.
3) Scolapio JS, Ukleja A, Bouras EP et al. Nutritional management of chronic intestinal
pseudo-obstruction. J Clin Gastroenterol 1991; 28:306-312.
4) Malagelada JR, Distrutti E. Managementof gastrointestinal motility disorders. A
practical guide to drug selection and appropiate ancillary measures. Drugs 1996;52:494506.

10.- Cuando se observa una imagen histolgica de acantosis con elongacin de las crestas
interpapilares-que incluso se fusionan entre s, hiperparaqueratosis y acmulos epidrmicos
de leucocitos polimorfonucleares, estamos hablando de:

a) Icitiosis.
b) Dermatitis (eccema).
c) Psoriasis.
d) Epidermlisis.

Psoriasis
La psoriasis es una enfermedad cutnea caracterizada por un curso crnico, que cursa a
brotes y tiene hallazgos clnicos variables. Las manifestaciones cutneas de esta
enfermedad son tan caractersticas (Tabla 1) que el diagnstico suele realizarse con
facilidad. En la actualidad se contempla la psoriasis como una enfermedad de base
inmunolgica, mediada por los linfocitos T, que asocia inflamacin drmica y
secundariamente hiperplasia epidrmica.
Es probablemente la enfermedad dermatolgica mediada inmunolgicamente ms
prevalente.

Tabla 1
Caractersticas clnicas de la psoriasis:
1.
2.
3.
4.
5.

Placa eritematosa.
Escamas blanquecinas.
Buena delimitacin.
Signo de Auspitz.
Fenmeno de Koebner.

Psoriasis. Hiperparaqueratosis con acmulos intracrneos de neutrfilos (microabscesos de


Munro). HE, x 400.

Cambios histolgicos de la psoriasis:

Infiltracin de clulas mononucleares


Hiperplasia epidermica
Acantosis epidermica regular, con hipogranulosis, hiperqueratosis y paraqueratosis
Atrofia epidermica suprapapilar

Infiltracininflamatoria
Pustula espongiforme de Kogog
Microabscesos de Munro
Infiltrado de linfocitos CD4 y CD8 en dermis
Cambios vasculares

Proliferacin de vasos capilares dilatados ocupando las papilas.

Bibliografa:

Arnold HL, James WD, Odom RB. Andrews : tratado de dermatologa. 1993. 4 ed.
Barcelona: Masson; 1993
Ashcroft DM, Po AL, Williams HC, Griffiths CE. Systematic review of comparative
efficacy and tolerability of Calcipotriolin treating chronic plaque psoriasis. BMJ
2000; 320:963-967. [Medline][Texto completo]
Chalmers RJG, O'Sullivan T, Owen CM, Griffiths CEM. Intervenciones para la
psoriasis guttata (Revisin Cochrane). En: La Cochrane Library Plus, Nmero 2,
2002. Oxford: Update Software
Ferrndiz Foraster C. Dermatosis eritematoescamosas (I). Psoriasis.
Eritrodermias. En: Ferrndiz C, ed. Dermatologa Clnica.2 ed. Madrid: Harcourt;
2001. p. 165-175.
Freedberg I, Eisen A, Katz SI, Wolff K, Fitzpatrick TB, Goldsmith LA et al, eds.
Dermatology in general medicine. 5th ed. New York: McGraw-Hill; 1999
Riffiths C E, Clark C M, Chalmers R J, Li Wan Po A, Williams H C. A systematic
review of treatments of severe psoriasis. Health Technology Assessment,
2000;4(40):1-125.
Naldi L, Rzany B. Chronic plaque psoriasis. Clin Evid 2002; 7: 1488-1507

11.- Si el estudio de triple marcador en suero materno indica una elevacin de AFP (alfa
feto-protena) respecto a la edad gestacional, usted debe sospechar como primera opcin:
a) Trisoma 21 (Down)
b) Trastorno monognico
c) Defecto de cierre de tubo neural o de pared abdominal
d) Labio hendido

Alfa Fetoproteina (AFP) es una glicoprotena de PM 70.000 D, cuya secuencia de


aminocidos presenta una homologa del 40 % con la albmina. Se sintetiza inicialmente en
el saco vitelino y posteriormente en el hgado fetal. Su concentracin en sangre fetal
aumenta hasta alcanzar un mximo de 300 mg/100ml a las 10-13 semanas de embarazo. A
partir de este momento, disminuye progresivamente a menos de 100 mg/100ml a trmino y
sigue disminuyendo hasta los 5 mg/100ml a los 2 aos de edad, permaneciendo en estos
niveles hasta la vida adulta.

La AFP puede migrar de la circulacin fetal a la materna a travs de dos mecanismos:


1. Difusin transplacentaria 2. Difusin transamnitica desde la orina fetal.

La concentracin en sangre materna es cinco veces inferior a la fetal y va aumentando


durante el segundo trimestre debido al incremento de la permeabilidad placentaria.
Podemos encontrar por lo tanto aumento de la AFP srica materna por defectos de la
barrera feto-amnitica ( Defectos abiertos del tubo neural , defectos abiertos de la pared
abdominal , higroma qustico , teratoma fetal , amputaciones fetales , muerte fetal ,
sndrome nefrtico fetal ) o por defectos de la barrera placentaria ( Hemorragia fetomaterna , tumores o infartos placentarios, placentas hipertrficas o qusticas ). Cualquier
mnimo compromiso de la integridad placentaria produce repercusiones importantes en los
niveles maternos de AFP, debido al gran gradiente de concentracin existente entre el
suero materno y el fetal.
La causa menos frecuente de elevacin de la AFP en suero materno, es la secundaria a la
disminucin de la eliminacin renal fetal por patologa obstructiva o displsica.
Los valores de AFP en suero materno estn disminuidos en pacientes portadoras de fetos
con Sndrome de Down. Existen diversas etiologas acerca de la causa de esta disminucin
siendo la ms aceptada la propuesta por Cukcle y colaboradores quienes abogan por una
disminucin de la sntesis heptica fetal.
La concentracin de AFP en el suero de fetos con trisoma 13, 18 y monosoma X, es ms
baja que la de fetos cromosmicamente normales. Los valores bajos de AFP en suero
materno no son exclusivos por tanto del Sndrome de Down.

TAMIZAJE BIOQUIMICO DE CROMOSOMOPATIAS.

12.- Una paciente de 25 aos con una historia de contactos sexuales con distintas parejas
en los ltimos meses, consulta por fiebre elevada, cefalea, adenopatas inguinales y
ulcerosas dolorosas en vulva. Cul es el diagnstico ms adecuado?

a) Vaginitis candidisica.
b) Enfermedad plvica inflamatoria por Chlamydia.
c) Infeccin por Citomegalovirus.
d) Primoinfeccin por Herpes simple tipo 2.

Los virus Herpes simplex (VHS) tipos 1 (VHS-1) y 2 (VHS-2), dos alfaherpesvirus
estrechamente relacionados, causan el herpes genital, un problema de salud pblica global,
de rango epidmico, cuya verdadera dimensin empezamos a advertir. En nuestro entorno,
el herpes es la tercera enfermedad de transmisin sexual (ETS) por orden de frecuencia y
la primera causa de lceras anogenitales. Se estima que en el mundo hay alrededor de 90
millones de individuos que sufren la enfermedad crnica recurrente sintomtica y, lo que
resulta ms inquietante, el nmero de infectados que desconocen serlo podra exceder esa
cifra. En Espaa, al igual que en otros pases de la Europa mediterrnea, la prevalencia de la
infeccin por VHS-2 en la poblacin general se sita en torno al 5-10% y alcanza el 25-40%
cuando se considera nicamente a individuos atendidos en clnicas de ETS.
El VHS-2 causa la mayora de los herpes genitales, si bien, durante los ltimos aos, se ha
producido un incremento del nmero de casos debidos al VHS-1, particularmente en
occidente, en adolescentes y adultos jvenes; este hecho es atribuible al aumento de los
adolescentes susceptibles al VHS-1 y a un cambio sustancial en el modelo de conducta
sexual de la poblacin.
La primoinfeccin genital por ambos VHS es, frecuentemente, asintomtica o "subclnica".
En su forma caracterstica, el herpes anogenital sintomtico cursa con lesiones vesiculosas
y ulcerativas dolorosas situadas, generalmente, en los labios y el introito vaginales, glande y
prepucio, adenopatas regionales, y, especialmente en la mujer, con disuria; no es
infrecuente que se acompae de sntomas generales tales como fiebre y cefalea. La
infeccin previa por el VHS-1 suele aminorar la sintomatologa asociada a la primoinfeccin
por el VHS-2. Las reactivaciones son asintomticas en ms del 80% de los casos,
independientemente del VHS causal, y los episodios de recurrencia sintomticos son, en
general, menos llamativos clnicamente que los vinculados a las primoinfecciones.
Los VHS-1 y VHS-2 difieren en cuanto a su capacidad de causar lesiones anogenitales
recurrentes; el VHS-2 las genera con mayor frecuencia que el VHS-1, particularmente
durante el ao siguiente a la primoinfeccin, en una proporcin de 5:1. Es de gran inters
epidemiolgico, por otra parte, el hecho de que mientras que el VHS-1 se excreta
infrecuentemente en ausencia de lesiones, el VHS-2 lo hace habitualmente, de forma
intermitente (entre 4-75% de los das del ao) e impredecible (no hay manera de prever
quines lo harn, en qu medida y con qu frecuencia), de modo que la mayora de
infecciones de novo por el VHS-2 son consecuencia de contactos sexuales con individuos
que excretan el virus de forma "subclnica" o asintomtica; no hay duda de que la poblacin
sexualmente activa que ignora estar infectada por el VHS-2 es el reservorio ms
trascendente del virus y la fuente de diseminacin de la infeccin.

BIBLIOGRAFA:

Ashley R, Wald A. Genital herpes: review of the epidemic and potential use of
type-specific serology. Clin Microbiol Rev 1999; 12:1-8.

Aurelian L. Herpes simplex viruses. En: Specter S, Hodinka RL, Young SA (eds).
Clinical Virology Manual, 3 ed. Washington DC: ASM Press, 2000; pp 384-409.
Guerry SL, Bauer HM, Klausner JD, et al. Recommendations for the selective use
of herpes simplex virus type 2 serological tests. Clin Infect Dis 2005; 40:38-45.
Kimberlin DW, Rouse DJ. Genital herpes. N Engl J Med 2004; 350:1970-1977.
Patel R. Progress in meeting today`s demands in genital herpes: an overview of
current management. J Infect Dis 2002; 186 (Supl 1):S47-S56.
Ramaswamy M, Smith M, Geretti AM. Detection and typing of herpes simplex DNA
in genital swabs by real-time polymerase chain reaction. J Virol Methods 2005;
126:203-206.
Slomka MJ. Current diagnostic techniques in genital herpes: their role in
controlling the epidemic. Clin Lab 2000; 11: 591-607.
Stevenson J, Hymas W, Hillyard D. Effect of sequence polymorphisms on
performance of two real-time PCR assays for detection of herpes simplex virus. J
Clin Microbiol 2005; 43:2391-2398.

13.- Masculino de 18 aos que desde la infancia padece anemia, esplenomegalia e ictericia.
El frotis de sangre perifrica muestra eritrocitos pequeos con palidez central y una
fragilidad osmtica muy incrementada. Ud. sospecha de:
a) Esferocitosis hereditaria
b) Anemia perniciosa
c) Talasemia mayor
e) Leucemia linfoblstica

La esferocitosis hereditaria (EH) es una enfermedad caracterizada por anemia hemoltica


de severidad variable, con presencia de esferocitos en sangre perifrica y una respuesta
clnica favorable a la esplenectoma. Con el desarrollo de nuevas tcnicas se encontraron las
primeras alteraciones bioqumicas de las protenas de la membrana eritrocitaria, y

posteriormente, se han podido precisar las alteraciones moleculares mediante las tcnicas
del ADN recombinante. La EH es una enfermedad muy heterognea que se produce por un
defecto intrnseco del glbulo rojo, y existen otras alteraciones secundarias a esta
afeccin. La prueba ms utilizada para el diagnstico de la EH es la fragilidad osmtica del
glbulo rojo. Se ha demostrado que esta enfermedad es producida por defectos de las
protenas que intervienen en las interacciones verticales entre el esqueleto de la membrana
y la bicapa lipdica. El tratamiento de eleccin en la EH es la esplenectoma, ya que es el

ms efectivo en el control de la anemia, aunque la sobrevida de los glbulos rojos


permanece acortada y los esferocitos no desaparecen. Este proceder se indica en
pacientes con anemia hemoltica severa o en individuos moderadamente asintomticos pero
que presentan litiasis vesicular.

Formas Clnicas
La EH es una enfermedad muy heterognea desde el punto de vista clnico. Se puede
observar desde el portador asintomtico hasta pacientes que presentan una anemia
hemoltica crnica con grandes requerimientos transfusionales.24,25.
Dependiendo de la severidad del cuadro clnico, de las cifras de hemoglobina, los niveles de
bilirrubina y el conteo de reticulocitos, esta enfermedad se clasifica en 4 formas:
portador asintomtico, EH ligera, EH tpica y EH severa.26,27.

Portador asintomtico. En algunas familias se ha sealado un patrn de herencia


autosmico recesivo. En estos casos, los padres de un paciente afectado no presentan
ninguna alteracin. En ocasiones la afectacin es muy leve, como ligero incremento de las
cifras de reticulocitos, escasos esferocitos en periferia o fragilidad osmtica incubada
alterada y puede no ser detectada por los exmenes de rutina. Debe tenerse en cuenta
tambin que pueden ocurrir nuevas mutaciones dentro de una familia aparentando un
patrn de herencia autosmico recesivo, por lo que siempre es importante un estudio
minucioso de todos los miembros de la familia.3,4.
EH ligera. Comprende entre el 20 y 30 % de todos los pacientes con EH autosmica

dominante, los que pueden presentar una hemlisis ligera compensada.3,27 Los individuos
son frecuentemente asintomticos y algunos casos son difciles de diagnosticar, ya que la
anemia y la esplenomegalia son muy ligeras y en ocasiones pueden estar ausentes.28
Muchos de estos pacientes se diagnostican durante estudios familiares o cuando en la
etapa adulta aparece el ctero y la esplenomegalia. Episodios hemolticos pueden
presentarse en el curso de algunos procesos infecciosos como mononucleosis, parvovirus o
citomegalovirus, as como durante el embarazo, por esfuerzos fsicos intensos o por
sangramientos.27-30.

EH tpica. Entre el 50 y 60 % de los pacientes con EH autosmica dominante tienen esta

forma clnica. Presentan una hemlisis compensada incompleta y una anemia de ligera a

moderada. El ctero es comn en nios, aunque se puede ver tambin en los adultos y est
asociado con infecciones virales ligeras, debido a la estimulacin reticuloendotelial y a un
aumento de la hemlisis. Los requerimientos transfusionales son espordicos. La
esplenomegalia est presente en el 50 % de los nios y en el 75 % de los adultos.2,3,31,32.

EH severa. Estos pacientes (5-10 %) evolucionan con una hemlisis severa y presentan

frecuentes requerimientos transfusionales. La mayora de estos casos tienen una forma


autosmica recesiva de la enfermedad. Pueden presentar crisis aplsticas, retardo del

crecimiento y de la maduracin sexual. La esplenectoma es el tratamiento de eleccin en


esta forma clnica.33,34 Generalmente la enfermedad debuta al nacimiento con ictericia y
hemlisis y se requiere, en muchas ocasiones, de exanguinotransfusin.30,32.

REFERENCIAS BIBLIOGRFICAS:
1. Berga L, Vives-Corrons JLl, Feli E, Woessner S, Rozman C. Hemorreologa. Bases
tericas y aplicaciones clnicas. Barcelona: Salvat, 1983.
2. Delaunay J. Genetic disorders in the red cell membrane. Crit Rev Oncol Hematol
1995;19:79-110.
3. Iolascon A, Miraglia del Giudice E, Perrotta S, Alloisio N, Morle L, Delaunay J.
Hereditary spherocytosis: from clinical to molecular defects. Haematologica 1998; 83:24057.
4. Scriver SR, Beaudet AL, Sly WS, Valle D. The metabolic and molecular bases of
inherited disease.
Philadelphia: McGraw-Hill Company, CD-ROM, 1997.
5. Yu J, Steck TL. Isolation and characterization of band 3, the predominant polypeptide
of the human erythrocyte membrane. J Biol Chem 1975;250:9170-6.
6. Kay MMB, Folwers N, Goodman J. Alterations in membrane protein band 3 associated
with accelerated erythrocyte ageing. Proc Nall Acad Sci USA 1989;86:5834-8.
7. Marsh WL. Molecular defects associated with McLeod blood group phenotype. En.
Salmon E, ed.
Blood groups and other red cells surface markers in health and disease 1982:17-82.
8. Tse WT, Lux SE. Red blood cell membrane disorders. Br J Haematol 1999;104:2-13.
9. Speicher DW, DeSilva TM, Speicher KD, Ursitti JA, Hembach P, Weglarz L. Location of
human red cell spectrin tetramer binding site and detection of a related closed hairpin
loop dimer using proteolytic footprinting. J Biol Chem 1993;268:4227-31.
10. Jordan C, Puschel B, Koob GR. Identification of a binding motif for ankirin on the a
subunit of Na+ K+ ATPase. J Biol Chem 1995;270:29971-4.

14.- Masculino de 34 que acude por fatiga y palidez. Su Hto es del 32% y su Hb de
10.3g/100mL. El frotis de sangre perifrica revela macrocitosis. Su nivel de vitamina B12
srica es 90 pg/mL (normal 170-940). Su diagnstico es:

a) Leucemia linfoctica
b) Anemia microctica
c) Prpura tromboctiopnica
d) Anemia megaloblstica

La anemia megaloblstica es un tipo de anemia en donde existe una disminucin de la


sntesis del ADN con detencin de la maduracin que compromete las tres lneas celulares
de la mdula sea (glbulos rojos, glbulos blancos y plaquetas). Las causas que la producen
son numerosas, pero aproximadamente el 95% de los casos es consecuencia de una
deficiencia de vitamina B 12 y/o de cido flico. Las manifestaciones clnicas y
hematolgicas son similares en ambos casos, pero las manifestaciones neurolgicas se
presentan slo en los casos de deficiencia de vitamina B12. El tratamiento est ligado a la
causa que la produce.
Hemograma:
En la serie roja:
- Macrocitosis con un Volumen corpuscular medio > 100 fL, y generalmente la hemoglobina
corpuscular media est elevada (12)
- Ovalositos, dacriositos, cuerpos de inclusin: en el frotis de sangre perifrica (HowellJolly y anillos de Cabot)
- Incremento del ndice de anisocitosis.
En la serie blanca:
- Leucopenia en casos severos
- Un signo precoz de megaloblastosis carencial es la hipersegmentacin de los
Neutrfilos.
- Recuento de plaquetas: no suele alterarse pero puede haber trombocitopenia severa (4,
12).
El aspirado de mdula sea: es hipercelular, con aumento relativo de los precursores
eritroides, ncleos de aspecto inmaduro y citoplasma
hemoglobinizado. Los precursores granulocticos de gran tamao (metamielocitos gigantes)
as como megacariocitos(12).
Bioqumica: La determinacin de cobalamina en suero debe ser menor de 200 pg/ml, valor
que debe demostrarse en al menos dos determinaciones separadas (13) (Normal: 150- 900
pg/ml).
BIBLIOGRAFIA:

1. De Paz R, Hernndez- Navarro F. Manejo, prevencin y control de la anemia perniciosa.


Nutr Hosp. Scielo
Chile [en lnea] 2005 [fecha de acceso 12 de Febrero de 2007]; 20 (6): URL
2. Forrellat Barrios M, Gmis Hernndez I, du Dfaix
Gmez HG. Vitamina B 12: metabolismo y aspectos clnicos de su deficiencia. Rev Cubana
Hematol Inmunol
Hemoter. Biblioteca virtual de salud [en lnea] 1999 [fecha de acceso 12 de Febrero de
2007]; 15 (3): URL.
3. Brees MH, Berlow RW. El Manual Merk de diagnstico
y tratamiento. 10 ed. Espaa: Elsevier Espaa SA,
1999; 869- 873.

4. Ruiz Franco O. Sndromes anmicos (Primera parte)


Anemias carenciales. Rev Diagnstico. Fundacin instituto
Hiplito Unanue [en lnea] Abril- Junio de 2005
[fecha de acceso 13 de Febrero de 2007]; 44 (2); URL.

15.- Acude masculino de 6 aos de edad al departamento de Urgencias debido a que


presenta un cuadro de dificultad respiratoria. Sus padres refieren que por la maana
estaba bien y al regresar de jugar en el jardn presenta dificultad respiratoria. A la
exploracin fsica se encuentra letrgico, taquipneico, diafortico; y existen sibilancias y
estertores en ambos campos pulmonares, con abundantes secreciones, y pupilas reactivas
de 1mm. Signos vitales: FC 70 por minuto, FR 50 por min, TA de 90/ palpable, Temp. 37.5
C, peso de 25 Kg. Qu clase de exposicin txica presenta este paciente?

a) Hidrocarburos
b) Organofosforados
c) Anticolinergicos
d) Salicilatos

Los efectos agudos aparecen inmediatamente o poco tiempo despus de la exposicin: En la


absorcin drmica los signos y sntomas se manifiestan a las 2-3 horas despus de la
exposicin. Sin embargo es posible observar los efectos dentro de l/2-1 hora, dependiendo
de las circunstancias de la intoxicacin. Algunos compuestos pueden retenerse en el tejido
graso resultando en un retraso de los sntomas hasta por 24 horas.
Los sntomas de intoxicacin se dividen en:

1. Efectos muscarnicos que incluyen diaforesis, pupilas puntiformes, sialorrea, epfora,


broncoconstrccn y aumento de la secrecin de las glndulas bronquiales, espasmo
abdominal con vmito y diarrea, bradicardia.
2. Efectos nicotnicos que incluyen taquicardia, fasciculaciones musculares o contracciones
espasmdicas de los msculos finos, en los casos ms severos del diafragma y msculos
respiratorios.
3. Las manifestaciones en el sistema nervioso central incluyen cefalea, fatiga, vrtigo,
ansiedad, confusin, convulsiones, depresin del centro respiratorio, coma.
El inicio y la intensidad de los sntomas vara dependiendo del compuesto (inhibidor
directo/indirecto), la ruta y el nivel de exposicin.

Los primeros sntomas son usualmente nausea, cefalea, fatiga, vrtigo, visin borrosa muchas veces descrita "como un velo sobre los ojos"- y constriccin pupilar. Dependiendo
de la severidad de la intoxicacin estos sntomas pueden agravarse agregndose vmito,
dolor abdominal, diarrea, diaforesis y sialorrea. El empeoramiento progresivo se
caracteriza por espasmos musculares que usualmente inician en la lengua y los prpados,
progresando a crisis convulsivas y finalmente parlisis. Tambin hay broncoconstriccin e
hipersecrecin bronquial y en la etapa final se observa parlisis, convulsiones, depresin
respiratoria y coma. En la intoxicacin fatal por organofosforados la causa inmediata de
muerte es generalmente la asfixia como resultado de depresin respiratoria.
Para la mayora de los plaguicidas organofosforados, la exposicin dermal y la
subsecuente absorcin a travs de la piel intacta representa la ruta de entrada ms
importante en caso de exposicin ocupacional.
La ruta oral es importante en caso de ingestin accidental o con fines suicidas. La
ingestin ocupacional accidental ocurre como resultado de un deficiente adestramento en
el trabajo y la falta de higiene personal.
La ruta respiratoria es generalmente la menos importante. La inhalacin de
organofosforados depende de la volatilidad del compuesto, del tipo de formulacin y de la
tcnica de aplicacin.

16.- Mujer de 25 aos presenta epistaxis, sangrado transvaginal, equimosis y petequias.


Refiere que ha presentado hemorragias en las encas. Desde hace tres das ha tenido
artralgias leves. A la exploracin fsica, se encuentran huellas de sangrado en la nariz y no
hay visceromegalias. El diagnstico m+as probable es:

a)
b)
c)
d)

Enfermedad de von willebrand.


Prpura trombocitopnica autoinmune.
Prpura trombocitopnica trombtica.
Prpura trombocitopnica idioptica.

La EVW es el trastorno hemorrgico hereditario ms comn en los seres humanos. La


caracterstica central de todos los tipos de EVW es la presencia de cantidades reducidas
de FVW o de formas anormales del FVW en el torrente sanguneo.
Papel del factor von Willebrand en la hemostasia:
Facilita la adhesin plaquetaria a la pared del vaso sanguneo lesionado.
Participa en la agregacin plaquetaria.
Es la protena portadora del factor VIII.

ENFERMEDAD DE VON WILLEBRAND: INTRODUCCIN PARA MDICOS DE


ATENCIN PRIMARIA
David Lillicrap
Department of Pathology and Molecular Medicine
Queens University, Kingston, Canad
Paula James
Department of Medicine
Queens University, Kingston, Canad
TRATAMIENTO DE LA HEMOFILIA
MARZO DE 2009 NO 47

17. - The following conditon is commonly seen in the magnesium-amonium-phosphate


(struvite) stones:
a)
b)
c)
d)

Recurrent P. mirabilis infection


Double J stent placement
Resorptive hypercalciuria
Renal azotemia

Esta bacteria de colonias redondeadas tiene la habilidad de producir grandes niveles de


ureasa. La ureasa hidroliza urea a amonaco, (NH3) y eso hace a la orina ms alcalina. Y al
subir la alcalinidad puede liderar la formacin de cristales de estruvita, carbonato de
calcio, y/o apatita. Esta bacteria puede encontrarse en clculos, y esas bacterias
escondidas all, pueden reiniciar una infeccin post tratamientos antibiticos. Al
desarrollarse los clculos, despus de un tiempo pueden seguir creciendo ms y causar
obstruccin dando fallas renales.

Proteus tambin puede producir infecciones de heridas, septicemia y neumonas, sobre


todo en pacientes hospitalizados.

Esipov, Sergei E. and J. A. Shapiro (1998). Kinetic model of Proteus mirabilis


swarm colony development. Journal of Mathematical Biology 36 (3).
doi 10.1007/s002850050100.
Frnod, Emmanuel (2006). Existence result for a model of Proteus mirabilis
swarm.
Differential and integral equations 19 (6):
pp. 697-720.
http://arxiv.org/abs/math.FA/0702761.

18.- En la enfermedad por reflujo los siguientes pacientes tiene una indicacin ms clara
de tratamiento quirrgico:

a) Paciente de 68 aos de edad con esofagitis erosiva y antecedentes de infarto de


miocardio hace 2 aos con insuficiencia cardiaca.
b) Paciente de 79 aos con hernia hiatal de mediano tamao y pirosis frecuente que
responde bien a 20 mg/da de omeprazol en 6 semanas.
c) Femenino de de 45 aos con molestias epigstricas tipo flatulencia y pirosis intermitente que responde slo ligeramente al tratamiento con omeprazol 20 mg.

d) Masculino de 56 aos con pirosis diaria diurna y nocturna de ms de 10 aos de


evolucin
que durante el tratamiento con 40 mg de esomeprazol
se presenta
asintomtico y a la supresin o disminucin de dosis reinciden de forma inmediata los
mismos sntomas.

QU ENFERMOS DEBEN SER ENVIADOS A TRATAMIENTO QUIRRGICO?


1. Aquellos con sintomatologa persistente, que afecta su calidad de vida y que dependen de
la ingesta constante de medicamentos.
2. Falta de apego al tratamiento mdico.1
3. Pacientes que para el control de sus sntomas requieren de un aumento progresivo de las
dosis habituales de omeprazol (20 mg), rabeprazol (20 mg), lansoprazol
(30 mg), pantoprazol (40 mg) o esomeprazol (40 mg).
4. Esofagitis erosiva que requiere tratamiento a largo plazo y con riesgo de complicaciones.
5. El enfermo con sntomas recurrentes y que decida de manera expresa la ciruga, siempre
que no exista una contraindicacin.

La ciruga antirreflujo es una alternativa de tratamiento a largo plazo en enfermos bien


seleccionados.
Es indispensable establecer con absoluta certeza el diagnstico de ERGE con o sin hernia
hiatal. El fundamento del tratamiento quirrgico es aumentar la eficacia de la barrera
antirreflujo
Las complicaciones de la enfermedad como: estenosis, lcera o esfago de Barrett no son
indicaciones absolutas de ciruga.

Los predictores de buena respuesta al tratamiento quirrgico son:


1. Respuesta adecuada a tratamiento farmacolgico.
2. Pacientes con pHmetra anormal. En los enfermos con pHmetra normal y ERGE se han
informado malos resultados en comparacin con los que son llevados a funduplicatura con
pHmetra anormal.
3. Personas menores de 50 aos.2,3
La efectividad de la ciruga se ve reducida en sujetos con reflujo refractario a tratamiento
mdico, enfermedad documentada de manera deficiente.

Enfermedad por reflujo no erosiva (ERNE)

En los casos de ERNE no se ha determinado bien el papel de la ciruga antirreflujo. Aunque


podra considerarse en caso de algunos sntomas extraesofgicos a nivel larngeo o
pulmonar, o en el caso de un defecto anatmico grande como hernia hiatal.4
El grupo consider que la ciruga antirreflujo laparoscpica representa un avance
teraputico significativo, sin embargo, los resultados dependen sustancialmente de la
experiencia del cirujano. No hay en la actualidad estudios que comparen el costoefectividad de la Terapa utica mdica con la quirrgica. Por lo tanto, la eleccin depender
de la preferencia del paciente en el caso de contar con un grupo quirrgico entrenado.

(Nivel de Evidencia 2, Grado de Recomendacin B).

1. Armstrong D, Marshall J, Chiba N, Enns R, Fallone C, et al. Canadian Consensus


Conference on the management of gastroesophageal reflux disease in adults. Update 2004.
Can J Gastroenterol 2005; 19: 15-35.
2. De Vault K, Castell D. Update Guidelines for the diagnosis and treatment of
Gastroesophageal reflux disease. AJG 2005: 100: 190-200.
3. Catarci M, Gentileschi P, Papi C, Carrara A, Marrese R, et al. Evidence- Based Appraisal
of Antireflux Fundoplication. Ann Surg 2004; 239: 325- 37.
4. Westcot C, Hopkins B, Bach K, Postman G, Belafisky P, et al. Fundoplicationfor laringofaringeal reflux disease. Am J Coll Surg 2004; 199:23-30.
5. Brooks DC, Rattner D. Patient satisfaction following laparoscopic andopen antireflux
surgery. Arch Surg 1995; 130: 289-94.
6. Catarci M, Gentileschi P, Papi C, Carrara A, Marrese R, et al. Evidence- Based Appraisal
of Antireflux Fundoplication. Ann Surg 2004; 239: 325-37.

7. Zoring C, Strate U, Fibbe C. Nissen vs Toupet laparoscopic fundoplication. Surg End


2002; 16: 758-66.
8. Freston JW, Triadafilopoulos G. Review article: approaches to the longterm management
of adults with GERD-proton pump inhibitor therapy, laparoscopic fundoplication or
endoscopic therapy. Aliment Pharmacol Ther 2004; 19(Suppl. 1): 35-42.

19.-Femenino de 34 aos la cual labora en lavandera, hace varios aos presenta


enrojecimiento de los pliegues proximales de las uas de varios dedos de las manos, que
ocasionalmente le supuran. Cul es el diagnstico ms probable?
a) Dermatitis de contacto.
b) Paroniquia candidisica crnica.
c) Liquen plano.
d) Dermatoficia.

Infeccin por Candida del lecho ungueal que se presenta con mayor frecuencia como una
onicolisis asociada con paroniquia, aunque tambin se observa la destruccin completa del
lecho ungueal y la erosin de la zona distal y lateral de las uas de los dedos, sin distrofia
ungueal total. La perionixis candidisica se trata de la inflamacin con eritema, edema,
dolor y salida ocasional de pus blanquecino escaso y espeso del reborde ungueal de uno o
varios dedos de la mano. Con frecuencia, se acompaa de onicopata (discoloracin,
onicolisis, distrofia total, etc.).
Predisponentes: Todas las labores manuales que generen humedad. (muy frecuente en amas
de casas, trabajadores de restaurantes, lavanderas, etc.)

BIBLIOGRAFA:
1. Crespo Erchiga V. Protocolo diagnstico de contaminantes. En "Micologa para
dermatlogos" Ed. Janssen, Madrid, 1994, pp:49-70.
2. Crespo V, De Lus B, Delgado V, Crespo A y Vera. Espectro etiolgico de las onicomicosis
en nuestro medio. CO7. II Congreso Nacional de Micologa. Santiago de Compostela. Junio,
1994.

3. Crespo Erchiga V, Delgado Florencio V y Martnez Garca S. Micologa dermatolgica. Ed.


M.R.A. Barcelona, 2006.
4. Daniel III CR. The Diagnosis of Nail Fungal Infection. Editorial Arch Dermatol
1991;127:1566-1567.
5. Delgado Florencio V. Protocolo de identificacin de dermatofitos. En "Micologa para
dermatlogos". Ed. Janssen, Madrid, 1994, pp:27-41.
6. Delgado Florencio V. Estrategia en el diagnstico y tratamiento de las micosis
superficiales. Ed. Aula Mdica, Madrid, 1994.
7. Delgado V, Abad Romero-Balmas J, Armijo Moreno M y Dulanto F. Scopulariopsis
brevicaulis como agente de onicomicosis. Actas Deermo-Sif. 1976; 9-10:693-700.
8. English MP. Nails and fungi. Br J Dermatol 1976; 94:697-701.
9. Fevilhade de Chauvin M. Onicomicosis. Dermatologa prctica. 1994; 9:1-2.

20.-Masculino de 12 aos que inicia con dolor intraescrotal intenso de aparicin brusca al
encontarse realizando actividad fsica, se tiene la sospecha de una torsin de testculo,
Cul es, entre las siguientes, la prueba diagnstica de eleccin?

a) Ecografa-doppler.
b) Ecografa.
c) Radiografa escrotal.
d) Palpacin bimanual.

Ecografa eco-doppler color: Es la prueba diagnstica de eleccin al permitir la visualizacin


de la vascularizacin testicular pudiendo distinguir si el flujo circulatorio es normal,
ausente o aumentado. En la torsin lo normal es que est reducido o ausente. Tiene una
especificidad del 100% y una sensibilidad del 80%, aunque tiene sus limitaciones en las
subtorsiones.
Gammagrafa isotpica testicular con TC 99: Es tambin un mtodo sensible y especfico,
pero del que no siempre se puede disponer en urgencias. Se ver un rea de captacin
disminuida o ausente, rodeado de un rea de captacin aumentada por la hiperemia
reactiva.

Bibliografa:
1) Sesions AE et al: Testicular torsin: direction, cadem, duration and desinformation. J.
Urol
2003, 169: 663-665.
2) Cummings JM el al: Adult testicular torsin. J.Urol. 202, 167: 2109, 167: 2109-2110
3) Bedos F et al: Manual de Urologa. Ed Masson SA. Barcelona 1996. 297-335.

21.- De las dermatosis reaccionales, el padecimiento que se caracteriza afeccin de piel y


mucosas con ampollas en blanco de tiro, signo de Nikolski positivo y en la biopsia de piel se
observa ampolla con necrosis epidrmica es:

a)
b)
c)
d)

Psoriasis
Sndrome de Stevens-Johnson
Eritema Multiforme
Pnfigo Vulgar

La necrlisis epidrmica txica (NET) y el sndrome de Stevens-Johnson (SSJ) constituyen


un espectro de la misma enfermedad, compartiendo aspectos etiolgicos, patogenticos,
histolgicos y teraputicos. Casi todos (si no todos) los casos son inducidos por frmacos.
NET y SSJ pueden distinguirse clnicamente del eritema multiforme, que debe
considerarse una enfermedad distinta. La patogenia de la NET y del SSJ es poco conocida,
pero se acepta que intervienen reacciones inmunolgicas y un mecanismo final de apoptosis
masiva de queratinocitos epidrmicos. El tratamiento consiste en la retirada del frmaco
causal y medidas de soporte, evitando la administracin de corticosteroides. Se han
descrito tratamientos que pretenden detener la evolucin del cuadro, entre ellos
ciclofosfamida, ciclosporina, plasmafresis, pentoxifilina e inmunoglobulinas i.v.

El cuadro tpico de NET, descrito por Lyell (8), se caracteriza por la aparicin brusca, tras
un prdromo catarral, de lesiones eritematosas, dolorosas, inicialmente distribuidas de
forma simtrica en cara y tronco, aunque luego se puedan extender hacia las partes acras.
Las lesiones comienzan siendo mculas eritematosas, pero pronto aparece el signo
caracterstico de la enfermedad: la necrosis y desprendimiento de la epidermis que
produce ampollas flcidas y signo de Nikolsky con despegamiento de amplias zonas
epidrmicas que dejan erosiones exudativas. La extensin de este despegamiento es uno de
los factores pronsticos principales. Un pequeo porcentaje de enfermos presenta slo
eritema confluente y zonas erosivas, pero el 90% tienen adems lesiones aisladas,
salpicadas en la proximidad de las erosiones. stas son mculas de borde mal definido, con
forma irregular, y pueden tener un centro ms oscuro o ampolloso.
Casi todos los pacientes presentan lesiones mucosas, incluyendo erosiones dolorosas orales
y farngeas, lesiones oculares (que conllevan un riesgo elevado de secuelas) y genitales. El
dao de otros epitelios, como el respiratorio o digestivo, y las complicaciones de la
insuficiencia cutnea aguda agravan el cuadro
Signo de NIKOLSKY: desprendimiento de las capas de la piel, aparentemente sana, por
efecto de la presin tangencial del dedo, con una ventosa o con un esparadrapo.

1. Avakian R, Flowers FP, Arajo OE, Ramos-Caro FA. Toxic epidermal necrolysis: a review.
J Am Acad Dermatol 1991;25:69-79.
2. Roujeau J-C, Chosidow O, Saiag P, Guillaume J-C. Toxic epidermal necrolysis (Lyell
syndrome). J Am Acad Dermatol1990;23:1039-58.
3. Champion RH. Disorders of blood vessels. En: Champion RH, Burton JL, Ebling FJG, eds.
Textbook of dermatology, 5.a ed. Oxford: Blackwell Scientific Publications; 1992. p. 18.3418.38.
4. Fritsch PO, Elias PM. Erythema multiforme and toxic epidermal necrolysis. En:
Fitzpatrick TB, Eisen AZ, Wolff K, Freedberg IM, Austen KF, eds. Dermatology in general
medicine, 4.a ed. Nueva York: McGraw-Hill; 1993. p. 585-600.
5. Bttiger LE, Strandberg I, Westernholm B. Drug induced febrile mucocutaneous
syndrome. Acta Med Scand 1975.

22.- Una mujer de 35 aos de edad, afroamericana, acude a consulta debido a que presenta
lesiones de piel acrmicas que han ido desarrollndose en los ltimos aos. Una vez que una
placa se desarrolla, parece ser que nunca vuelve a colorearse normalmente de nuevo, y se
quema fcilmente a la exposicin solar. La EF demuestra 6 placas hipopigmentadas de piel
en cuello y trax que varan en tamao de 1 a 6cm. La piel afectada no presenta otras
alteraciones. Cul de los siguientes es el diagnstico ms probable?
a)
b)
c)
d)

Albinismo
Lentigo
Vitiligo
Hipocroma idiomtica

El vitiligo es una leucodermia adquirida de causa desconocida que se caracteriza por


manchas acrnicas y cuyo tratamiento en ocasiones es difcil. Se debe a la desaparicin
selectiva de los melanocitos epidrmicos en el rea de las lesiones. Tiene una prevalencia
del 1-2% a nivel mundial; en Mxico ocupa el entre el 3 y 5 lugar entre las dermopatas,

sin embargo, el impacto psicosocial es importante en muchos de los casos. Se presenta a


cualquier edad, principalmente entre los 20 y 40 aos, aunque puede verse incluso en nios
de 3-4 aos; tiene un ligero predominio en el sexo femenino. Es polignico, el patrn de
herencia no ha sido bien establecido, sin embargo, existe una elevada incidencia en
parientes de primer grado de pacientes que padecen vitiligo. De manera ocasional se
relaciona con padecimientos internos subyacentes, donde destacan factores hormonales y
autoinmunitarios, tales como tiroideos (hipotiroidismo, hipertiroidismo, enfermedad de
Graves y tiroiditis), que se pueden presentar hasta en el 14% de los pacientes con vitiligo;
la evolucin de ambos padecimientos es independiente. Otros padecimientos relacionados
son: disfuncin poliglandular, diabetesmellitus, anemia perniciosa. Se desconoce la etiologa
del vitiligo pero se tienen diversas hiptesis tales como la neural, la citotxica, la
inmunitaria, la psicosomtica y la bioqumica.
Elementos diagnsticos.
Se caracteriza por mancha acrmicas o hipocrmicas bien delimitadas, que se pueden
presentar en cualquier parte del cuerpo, pero es ms comn en cara (prpados, peribucal),
dorso de manos, muecas, axilas, ombligo, pezones, cintura, regin sacra e inguinal; puede
ser simtrico, aunque sta no es una caracterstica. Pueden aparecer nuevas lesiones en
sitios de presin o secundarias a un traumatismo (fenmeno de Koebner), ste ltimo se
considera como signo de progresin. Las manchas varian en nmero y tamao, la superficie
es lisa.
Otras manifestaciones clnicas son: Nevo en halo o vitiligo perinvico (nevo pigmentado
rodeado por una mancha acrnica), alopecia areata y leucotriquia; esta ltima indica un mal
pronstico para la repigmentacin.
Su evolucin generalmente es lenta, insidiosa, crnica y son asintomticas.

Bibliografa:
1) Luger T, Paul C. Potential new indications of topical calcineurin inhibitors. Dermatology.
2007;215 Suppl 1:45-54.
2) Lotti TM. Vitiligo: problemas y soluciones. DermatologaCMQ 2003;3:
3) Njoo MD, Spuls PI, Bos JD et al. Nonsurgical repigmentation therapies in vitiligo. Arch
Dermatol 1998;134:1532-1540.
4) Drake LA, Dinehart SM, Farmer ER et al. Guidelines of care for vitiligo. American
Academy of Dermatology. J Am Acad Dermatol 1996;35:620-626.

23.- Se trata de paciente masculino de 66 aos con antecedentes de tabaquismo positivo


consumiendo ms de 30 cigarrillos al da desde hace mas de 40 aos, hipertensin arterial
descontrolada, acude a consulta por presentar hematuria franca y alteraciones en la
funcin renal, a la exploracin fsica se aprecia masa abdominal en flanco derecho se

realiza una TAC demostrando masa slida de aproximados 9 cm de dimetro en rin


derecho. Cul es el diagnstico presuncional ms probable?
a) Nefroblastoma.
b) Adenocarcinoma.
c) Carcinoma epidermoide.
d) Liposarcoma.

La incidencia de cncer de rin ha aumentado sustancialmente en el mundo. El carcinoma


de clulas renales (CCR) representa 80 a 85% de todos los tumores malignos de rin.
Varios estudios epidemiolgicos indican que el tabaquismo constituye un importante factor
de riesgo. Un estudio reciente mostr que dejar de fumar se asocia con una reduccin lineal
del riesgo de CCR. El tabaquismo pasivo ha sido relacionado con varias consecuencias sobre
la salud incluyendo cncer (de pulmn, estmago y mama). Adems, recientemente se ha
establecido la asociacin causal entre tabaquismo activo y cncer de rin. El cigarrillo
parece ser el factor de riesgo ms importante para cncer de rin y explica la mayora de
los casos en casi todo el mundo.

Presentacin clnica
La mayora de los casos de carcinoma renal presenta una trada clsica de hematuria, dolor
abdominal en el flanco y una masa palpable. La hematuria gruesa se encuentra en un 45% de
los pacientes. Un 40% de ellos presentan un cuadro clnico llamado sndrome paraneoplsico
que comprende anemia, dolor seo, hipercalcemia, fiebre, prdida de peso.

La evaluacin inicial de un paciente con cncer renal debe incluir un buen historial y examen
fsico, laboratorio de rutina y una tomografa computarizada (CT scan) del abdomen y la
pelvis, as como una radiografa de pecho. Si hay evidencia o sospecha de trombosis de la
vena cava o trombosis tumoral venosa (que ocurre en un 5% a 10% de los casos), se
recomienda hacer una resonancia magntica (MRI).
En general, no se recomienda realizar una biopsia percutnea de la masa renal, a menos que
se sospeche que se trate de una metstasis de otra neoplasia, como cncer de colon o
linfoma, entre otros.
La reseccin quirrgica sigue siendo el tratamiento de eleccin para los tumores localizados,
ya que se sabe que dicha neoplasia es resistente a la quimioterapia y a la radioterapia.

Klein EA, RM Bukowski, JH Finke: Renal Cell Carcinoma. Immunotherapy and cellular Biology.
MARCEL DEKKER, Inc. 1993.
2. RE Lenhard, Jr, RT Osteen, T Gansler: The American Cancer Societys Clinical Oncology.
American Cancer Society, 2001.
3. 2008 Genitourinary Cancers Symposium: A Multidisciplinary Approach. February 14-18
2008.
4. Managing the Complex Journey of Renal Cell Carcinoma. Institute for medical Education
& Research. March 2008.
5. Highlights of the NCCN 13th Annual Conference. Clinical Practice Guidelines & Quality
Cancer Care. March 5-9, 2008.
6. JS Lam, J Bergman, A Breda, P Schulam: Importance of Surgical Margins in the
Management of Renal Cell Carcinoma. Nature Clinical Practice Urology, Medscape.com. May
13, 2008.
7. R. Nelson: Renal cell carcinoma is being increasingly diagnosed at early stages. Medscape
Medical News. Medscape.com. May 22, 2008.

24.- Paciente masculino de 40 aos de edad, empleado en una fbrica, homosexual, con
infeccin por VIH diagnosticada hace 2 aos, acude al servicio de urgencias por presentar
fiebre de 15 das de evolucin, ataque al estado general, prdida de peso, disnea progresiva
y tos productiva. Se realiz tincin de la expectoracin, que report bacilos cido-alcohol
resistente. Su cuenta de CD4 realizada hace 1 mes fue de 100 cl/mL y una carga viral
reciente fue de 250,000 copias. En qu estadio CDC se encuentra el paciente?
a)
b)
c)
d)

B3
C3
B2
A3

El estadio C3 est definido por una cuenta de CD4 menor de 200 cl/mL y la presencia de
una condicin clnica indicativa de Sndrome de Inmunodeficiencia Adquirida, entre las que
se encuentra la infeccin por Mycobacterium tuberculosis en cualquier variedad.

CLASIFICACION DE INFECCION VIH


VIGILANCIA, ADULTOS Y ADOLESCENTES
CDC 1993
A
Asintomtico
Infxn aguda

B
Sintomtico*
(no A o C)

C
Indicador de
SIDA

> 500/mm3

A1

B1

C1

200-499/mm3

A2

B2

C2

< 200/mm3

A3

B3

C3

Categora CD4

* muguet oral, candidiasis vaginal persistente, displasia cervical, fiebre-diarrea


de >1mes, leucoplasia vellosa, zoster (2 episodios o >1 dermatoma), PTI, PID

NO TOMA EN CUENTA LA CARGA VIRAL


5

Kasper DL, Braunwald E, Fauci AS, Hauser SL, Longo DL, Jameson JL. Harrisons Principles
of Internal Medicine. McGraw Hill. 16 Ed. 1076 p.

25.- Se trata de femenino de 22 aos, que presenta lesiones eritematoescamosas, edema


y alguna vescula en la cara, escote, dorso de las manos y antebrazos. Las lesiones tienen 12
horas de evolucin y han aparecido tras una escursin al campo. Entre los antecedentes
personales destaca acn vulgar en tratamiento con retinoides tpicos y doxiciclicina oral. El
diagnstico ms probable es:

a) Erupcin lumnica poliforma.

b) Eritrodermia por frmacos.


c) Urticaria solar.
d) Reaccin fototxica.

DEFINICIN
Enfermedades cutneas que se producen por el aumento de capacidad de reaccin de la piel
a las radiaciones lumnicas tras la administracin de una sustancia fotosensibilizante. Se
conocen como reacciones de fotosensibilidad y pueden desencadenarse tanto por contacto
como por la administracin sistmica del agente fotosensibilizante. Si existe implicacin
inmunolgica se denomina dermatitis fotoalrgica y si no dermatitis fototxica.

Dermatitis fototxica

No existe un mecanismo inmunolgico, puede afectar a muchas personas siempre que exista
dosis elevada de irradiacin y cantidad suficiente de sustancia qumica. Las lesiones
aparecen tras la primera exposicin, son monomorfas, y se caracterizan por eritema
intenso, edema y vesiculacin en reas de piel fotoexpuestas, marcando claramente los
bordes de las zonas descubiertas, y onicolisis ungueal . Formas particulares de fototoxia:
fitofotodermatitis (dermatitis de los prados, apio) (Fig. 3), dermatitis de Berloque,
fotosensibilidad en tatuajes (sulfuro de cadmio), frmacos (tetraciclinas, AINEs,
amiodarona (color azulado), clorpromacina (color gris).

Figura 3. Fitofotodermatitis

1. Litt Jz. Drug eruption reference manual 2001. New York: Parthenon, 2001.
2. Sullivan JR, Shear NH. Drug eruptions and other adverse drug effects in aged skin.
Clinics in geriatric medicine 2002;18(1).
3. Lim HW, Gigli. Complement-derived peptides in phototoxic reaction. En: Daynes RA,
Spikes JD, editors. Experimental and clinical photoimmunology. Boca Raton: CRC Press,
1983:81-93.
4. Torinuki W, Tagami H. Role of complement in chlorpromazine-induced phototoxicity. J
Invest Dermatol 1986;86:142-4.
5. Hearst JE, Issacs ST, Kanne D, Rapoport H, Straub K. The reaction of the psoralens
with deoxyribonucleic acid. Q Rev Biophys 1984;45:891-5.
6. Athar M, Elmets CA, Bickers DR, Mukhtar H,. A novel mechanism for the generation of
superoxide anions in hematoporphyrin derivative-mediated cutaneous photosensitization.
Activation of the xantine oxidase pathway. J Clin Invest 1989;83:1137-43.
7. Matsuo I, Inukai N, Fujita H, Ohkido M. Possible involvement of oxidationmof lipids in
inducing griseofluvin photosensitivity. Photodermatol Photoimmunol Photomed 1990;7:2137.
8. Harber LC, Bickers DR. Photosensitivity diseadses. Principles of diagnosis and
treatment. Ontario: BC Decker Inc, 1989:160-202.
9. Kockevar IE. Phototoxicity of nonsteroidal inflammatory drugs. Coincidence or specific
mechanism?. Arch Dermatol 1989;125:824-6.

26.- El diagnstico ms probable en un paciente masculino de 17 aos de edad que


desconoce sus antecedentes familiares. Desarrolla dolor abdominal progresivo, posterior a
un traumatismo directo en la regin lumbar; se documenta un hematoma en el msculo psoas
izquierdo. La Bh con 11 gr. de HB, reticulocitos 35, LEUCOCITOS DE 13500, 350 mil
plaquetas, TTP: 60/35, TP: 12/12, TT: 26/26 fibringeno 350 MG/dl. Cul es su
sospecha diagnstica?

a)
b)
c)
d)

Enfermedad de Von Willebrand


Hemofilia
Trombocitopata
Deficiencia de factor XII

Cules son los sntomas de la hemofilia?


El sntoma ms comn de la hemofilia es la hemorragia incontrolable y excesiva por causa
del factor de coagulacin que falta o est en bajos niveles en la sangre. Puede producirse
una hemorragia incluso cuando no haya ninguna lesin. La mayora de las veces se produce en
las articulaciones y en la cabeza.
A continuacin, se enumeran los sntomas ms frecuentes de la hemofilia. Sin embargo,
cada individuo puede experimentarlos de una forma diferente. Los sntomas pueden incluir:

Equimosis (Moretones)
Pueden producirse moretones por pequeos accidentes, que pueden producir un
hematoma grande (una acumulacin de sangre debajo de la piel que causa
inflamacin).
Sangra con facilidad
La tendencia a sangrar por la nariz, la boca y las encas por un traumatismo sin
importancia, cepillarse los dientes y, o trabajo dental es a menudo una indicacin de
hemofilia.
Hemorragia en una articulacin
La hemartrosis (hemorragia en una articulacin) puede producir dolor, inmovilidad y,
con el tiempo, deformacin si no se trata mdicamente de la forma adecuada. sta
es la zona ms comn de complicaciones debido a la hemorragia por hemofilia. Estas
hemorragias de las articulaciones pueden producir la artritis crnica y dolorosa,
deformaciones y parlisis si se repiten.
Hemorragia en los msculos
La hemorragia en los msculos puede causar hinchazn, dolor y enrojecimiento. La
hinchazn por el exceso de sangre en estas zonas puede producir un aumento de la
presin en los tejidos y nervios de la zona, provocando dao y, o deformacin
permanente.

Hemorragia por lesiones o hemorragia cerebral


La hemorragia por lesin o espontnea en el cerebro es la causa ms comn de
muerte en los nios que tienen hemofilia y la complicacin hemorrgica ms grave.

Otras fuentes de hemorragia


La sangre en la orina o en las heces puede ser tambin un sntoma de hemofilia.

Los sntomas de la hemofilia pueden parecerse a los de otros trastornos de la sangre o


problemas mdicos. Siempre consulte a su mdico para el diagnstico.

Roberts HR. Hemophilia A and Hemophilia B. In: Lichtman, MA, et al., eds. Williams
Hematology. 7th ed. The McGraw-Hill Companies, Inc.; 2006:chap 115.

27.- Femenino de 45 aos de edad que se encuentra en terapia intensiva por manejo de
spsis abdominal. Lleva dos das con tratamiento a base de ceftriaxona, metronidazol y
amikacina, el cual provee de una cobertura conveniente para los microorganismos
probablemente involucrados. Despus de revisar al paciente que ha tenido una evolucin
favorable, usted sugiere que se aplique el aminoglucsido en monodosis ya que:

a)
b)
c)
c)

De esta forma aumenta su potencia.


Como tiene un efecto post antibitico corto su efecto es mejor.
Disminuyen sus efectos adversos.
Hace sinergismo con el Metronidazol.

Una caracterstica farmacolgica de los aminoglucsidos es su actividad dependiente de


concentracin, por lo que su aplicacin en bolos, con lo que se logran niveles pico ms altos,
es de mayor conveniencia para aprovechar este fenmeno. Adems, el importante efecto
post-antibitico que tiene, permiten que la actividad dure ms all del tiempo en que sus
concentraciones se encuentran por arriba de la mnima inhibitoria, favoreciendo una
posologa cmoda cada 24 horas. Por otra parte, se ha visto que los efectos adversos
(nefrotoxicidad y ototoxicidad), se encuentran ms relacionados con los niveles valle.

Estos se mantienen a niveles bajos cuando se utiliza el esquema en monodosis, disminuyendo


la probabilidad de la presentacin de estos efectos adversos.

Kasper DL, Braunwald E, Fauci AS, Hauser SL, Longo DL, Jameson JL. Harrisons Principles
of Internal Medicine. McGraw Hill. 16 Ed. 789-806 pp.

28.- Un varn de 18 aos presenta cefalea persistente y fiebre, y despus de cinco das, un
cuadro de crisis convulsiva tnica focal. Una tomografia computadorizada de la cabeza
muestra una lesin con reforzamiento anular en el lbulo frontal derecho y un nivel
hidroareo en el seno frontal del mismo lado. Lo ms probable es que un aspirado
neuroquirrgico de la lesin muestre:

a)
b)
c)
d)

Clulas mononucleares pequeas sugestivas de linfoma de Burkitt.


Quistes de Toxoplasma gondii y taquizotos (trofozotos)

Escherichia coli y Bacteroides fragilis.

Estreptococo hemoltico a y mezcla de anaerobios.

El absceso cerebral es una patologa que se da entre personas jvenes o de edad media
(entre la segunda y cuarta dcadas de la vida), con una inexplicable predileccin por el sexo
masculino (Doble de frecuencia).
El

25%

de

los

abscesos

cerebrales

ocurren

en

nios

menores

de

15

aos.

De forma rara se presentan en neonatos pero tienen una alta morbi-mortalidad (Wu, Lai et
al.2006).

Localizacin:
La localizacin de los abscesos est muy relacionada con la etiologa de la infeccin, siendo
la predileccin de los abscesos de origen otgeno por el lbulo temporal o cerebelo, los de
senos paranasales por el lbulo frontal, o la frecuente multiplicidad de los de origen
hematgeno.

Etiologa:
El absceso cerebral se inicia cuando los grmenes alcanzan el parnquima cerebral, lo cual
se produce a travs de tres vas fundamentales: por inoculacin directa o fstulas, en el
caso de traumatismos o ciruga; por una infeccin en contigidad (bien por invasin directa,
bien por tromboflebitis de las venas emisarias); o por embolismos spticos procedentes de
infecciones a distancia. Tradicionalmente las infecciones crnicas del odo y de los senos
paranasales han sido la causa ms frecuente de abscesos cerebrales, pero su incidencia ha
disminuido con la mejora del tratamiento de estas infecciones, producindose as un
relativo incremento de la infeccin de origen hematgeno.

Microorganismos ms frecuentes: Estreptococo, Estafilococo, Enterobacterias.


Los microorganismos ms frecuentes son los estreptococos, 33%-50% anaerobios o
microaerfilos. Generalmente, se obtienen muchos microorganismos en 10%-30% de los
cultivos, pero esta cifra puede aproximarse a 80%-90%, y suelen hallarse cepas anaerobias
(son frecuentes las Bacteroides sp.)
En los abscesos secundarios a sinusitis frontoetmoidal, se observan streptococcus milleri y
Streptococcus anginosus. Cuando son secundarios a otitis media, mastoiditis o abscesos
pulmonares, por lo general, se hallan varios tipos de microorganismos: estreptococos
anaerobios, Bacteroides, enterobacterias (proteo o proteus).Los abscesos postraumticos
suelen deberse a S. aureus o a enterobacterias.

En los pacientes inmunodeprimidos transplantados (tanto de mdula sea como de rganos


slidos), son frecuentes las infecciones fngicas, la mayora por Aspergillus fumigatus a
partir de una infeccin pulmonar primaria.
Lactantes: son frecuentes los bacilos gramnegativos porque la IgM no pasa a la placenta.
Microorganismos que pueden generar abscesos en pacientes inmunodeficientes:
1.toxoplasmosis.
2.nocardia.

RM de un absceso multilobulado

Bibliografa:
1. Awad I, Bay JW, Petersen JM: Nocardial osteomyelitis of the spine with epidural spinal
cord
compression--a
case
report.
Neurosurgery
15:254-256,
1984.
2. Byrne E, Brophy BP, Perrett LV: Nocardia cerebral abscess: New concepts in diagnosis,
management, and prognosis. J Neurol Neurosurg Psychiatry 42:1038-1045, 1979.
Tonon, E., P. G. Scotton, et al. (2006). "Brain abscess: clinical aspects of 100 patients." Int
JInfectDis10
(2):103-9.
3.Tseng, J. H. and M. Y. Tseng (2006). "Brain abscess in 142 patients: factors influencing
outcome
and
mortality."
Surg
Neurol
65(6):
557-62;
discussion
562.
4.Visani, P., E. Schmutzhard, et al. (2006). "Subcortical deficit pattern after brain
abscess:
a
neuropsychological
study."
Eur
J
Neurol
13(6):
599-603.
5.Wandroo, F., P. Stableforth, et al. (2006). "Aspergillus brain abscess in a patient with
acute myeloid leukaemia successfully treated with voriconazole." Clin Lab Haematol 28(2):
130-3.

29.- Femenino que desde hace varias semanas le han aparecido de forma eruptiva unas
mculas y placas eritematosas en el tronco, refiere que hubo una lesin ms grande que
precedi a las dems. Las lesiones presentan una descamacin fina en la periferia y son discretamente pruriginosas. No existe afectacin palmoplantar. La serologa lutica es
negativa. Qu tipo de pitiriasis es la ms probable?

a) P. liquenoide crnica.
b) P. rubra pilaris.
c) P. rosada.
d) P. versicolor

La pitiriasis rosada es una enfermedad exantemtica autolimitada de causa desconocida


que cursa con lesiones maculopapulosas de aspecto asalmonado en reas proximales de las
extremidades y en tronco, donde se distribuyen de forma paralela a las lneas de la
hendidura (Stulberg DL, 2004).
Existen algunas caractersticas que sugieren una etiologa vrica (prdromos, predominio
estacional, evolucin variable, ausencia de recadas, hallazgos histolgicos...) que algunos
autores relacionan con el herpes virus humano 7, aunque otros lo desmienten (Chuh A,
2004).

Afecta con ms frecuencia a nios mayores y jvenes adultos (10-35 aos) y es ligeramente
ms frecuente en mujeres (Chuang TY, 1982).
El cuadro tpico se inicia con una lesin redondeada u oval, de 4-8 cm. nica, generalmente
situada en el tronco, de color asalmonado, discretamente descamativa y en ocasiones
ligeramente pruriginosa, llamada madre. Puede acompaarse de sntomas generales leves:
astenia, anorexia, odinofagia, cefalea y artralgias.
En 1-2 semanas esta lesin se torna ms escamosa y clara en el centro al tiempo que
aparecen otras de menor tamao que se diseminan por las extremidades (reas proximales)
y por el tronco distribuyndose simtricamente en forma arbrea a lo largo de las
hendiduras costales. En la espalda es ms evidente (signo del rbol de Navidad) (Stulberg
DL, 2004)
La evolucin de las lesiones es similar a las de la lesin madre desapareciendo en 5-6
semanas, aunque en algunos casos puede permanecer ms tiempo y en otros verse reas
hiper o hipo pigmentadas durante algn tiempo. La tasa de recurrencia a los 5 aos es
inferior al 2% (Chuang TY, 1982).

Pitiriasis Rosada
Chuh A, Chan H, Zawar V. Pityriasis rosea--evidence for and against an infectious
aetiology. Epidemiol Infect. 2004 Jun;132(3):381-90.
Chuh AA, Chan HH. Prospective case-control study of chlamydia, legionella and
mycoplasma infections in patients with pityriasis rosea.
Chuh AA, Dofitas BL, Comisel GG, Reveiz L, Sharma V, Garner SE, Chu F.
Interventions for pityriasis rosea. Cochrane Database Syst Rev. 2007 Apr
18;(2):CD005068.
Chuh AA. Quality of life in children with pityriasis rosea: a prospective case
control study. Pediatr Dermatol. 2003 Nov-Dec;20(6):474-8.
Hsu S, Le EH, Khoshevis MR. Differential diagnosis of annular lesions. Am Fam
Physician. 2001 Jul 15;64(2):289-96] Eur J Dermatol. 2002 Mar-Apr;12(2):170-3.
Miranda SB, Lupi O, Lucas E. Vesicular pityriasis rosea: response to erythromycin
treatment. J Eur Acad Dermatol Venereol. 2004 Sep;18(5):622-5

30.- Recibe el siguiente perfil de hepatitis viral: anticuerpos contra Hepatitis C negativos
(-); Hepatitis B: Antgeno de superficie positivo (+), Anticuerpos contra antgeno de
superficie negativos (-), inmunoglobulina G contra Antgeno core positiva (+), Antgeno e
(Anti-HBe) positivo (+). Con lo anterior diagnostica?
a)
b)
c)
d)

Infeccin aguda C.
Infeccin antigua C.
Infeccin aguda por hepatitis B.
Infeccin crnica por hepatitis B, con replicacin viral.

La infeccin aguda de hepatitis B se caracteriza por la presencia de inmunoglobulina M


contra antgeno del core, que no tiene este paciente. La presencia de inmunoglobulina G
contra ste mismo antgeno la caracteriza como crnica, y la presencia de antgeno e
determina que existe replicacin activa.

CRITERIOS DIAGNSTICOS

1. Determinar la presencia de infeccin crnica por el VHB, que vendr dictada por la
positividad del HBsAg durante ms de 6 meses.
2. Determinar la existencia de replicacin viral activa, caracterizada por la presencia en
suero del ADN-VHB. Existen distintos mtodos para determinar el ADN-VHB srico; si se
utilizan mtodos poco sensibles, como la hibridacin molecular, se pueden obtener
resultados falsamente negativos y por el contrario, tcnicas muy sensibles, como la
reaccin en cadena de la polimerasa, pueden ofrecer resultados positivos en portadores del
VHB en los que la replicacin del virus no tiene repercusin clnica relevante.
3. Determinar la existencia de enfermedad heptica y conocer su estado. La elevacin de
los valores de transaminasas constituye el marcador ms sencillo de la existencia de
enfermedad heptica activa. En tales casos, la realizacin de una biopsia heptica permitir
confirmar el diagnstico de hepatitis crnica, conocer su grado de actividad, as como el
estadio de fibrosis existente.
4. Descartar otras causas de enfermedad heptica. Otros virus de la hepatitis (VHD o
VHC), o incluso otras causas de enfermedad heptica pueden ser los responsables, en
asociacin o no con el VHB, de la lesin heptica en portadores crnicos de este virus. El
conocimiento de estas otras posibles causas de enfermedad heptica es imprescindible
para planificar el tratamiento oportuno.
La infeccin crnica por el VHB es un proceso dinmico, sujeto a la interaccin entre el
propio virus y el sistema inmune, que tiene como resultado la existencia de distintas fases,
con datos virolgicos y clnicos diferenciales:

HEPATITIS CRNICA POR VIRUS DE LA


HEPATITIS B
(Tabla 1).
Tabla 1. Fases de la infeccin crnica por el VHB
HBsAg
HBeAg
Histologa
Inmunotolerancia Positivo
Normal

ADN-VHB

Transaminasas

Positivo

Positivo

Normales

Positivo

Positivo
HBeAg+

Positivo

Elevadas

Hepatitis crnica Positivo


Hepatitis HBeAg crnica

Negativo

Portador sano
Positivo
Normal
del VHB
hepatitis "residual"

Negativo

Cambios
Mnimos
Hepatitis crnica
Hepatitis
crnica

Positivo

Negativo

Elevadas

Normales

BIBLIOGRAFA
Kasper DL, Braunwald E, Fauci AS, Hauser SL, Longo DL, Jameson JL. Harrisons Principles
of Internal Medicine. McGraw Hill. 16 Ed. 1835 p.
Benhamou Y, Bochet M, Thibault V et al. Long-term incidence of hepatitis B virus
resistance to lamivudine in human immunodeficiency virusinfected patients. Hepatology
1999; 30: 1302-1306.
Chayama K, Suzuki Y, Kobayashi M et al. Emergence and takeover of YMDD motif mutant
hepatitis B virus during long-term lamivudine therapy and re-takeover by wild type after
cessation of therapy. Hepatology 1998; 27: 1711-1716.

Chien R-N, Liaw Y-F, Atkins M. Pretherapy alanine transaminase level as a determinant for
hepatitis B e antigen seroconversion during lamivudine therapy in patients with chronic
hepatitis B. Hepatology 1999;
30: 770-4.
Dienstag JL, Schiff ER, Mitchell M et al. Extended lamivudine retreatment for chronic
hepatitis B: manteinance of viral suppression after discontinuation of therapy.
Lai C-L, Chien R-N, Leung NWY et al. A one year trial of lamivudine for chronic hepatitis B.
N Engl J Med 1998; 339: 61-8.

31.- Una nia de 2 aos de edad presenta lesiones rojas, con costras melicricas en la cara,
cuero cabelludo, rea de paal y extremidades, desde aproximadamente los 2 meses de
edad. El evitar sustancias irritantes no ha funcionado en el tratamiento. La nia se rasca
constantemente las reas afectadas. Hay una historia familiar positiva para asma y fiebre.
Cul es el diagnstico ms probable?
a)
b)
c)
d)

Dermatitis atpica
Celulitis
Dermatitis de contacto
Liquen cimple crnico

La dermatitis atopica (DA) (Wise-Sulzberger, 1993), llamada neurodermatitis diseminada,


por las escuelas Europea, es un estado reaccional de la piel, intensamente pruriginosa, ms
frecuente en los nios, multifactorial, en la que combinan factores constitucionales y
factores ambientales, por lo tanto de difcil tratamiento, muy frecuente en la consulta
diaria del pediatra y del dermatlogo.

DATOS EPIDEMIOLOGICOS
La enfermedad originalmente conocida como prrigo de Besnier y eccema constitucional,
es ubicua, afecta a todas las razas y existe en todos los pases. Se seala su existencia
hasta en el 2% de la poblacin general y en el 14% de los nios. La enfermedad se inicia
antes del primer aos de la vida en ms del 60% de los casos, la curva desciende hacia los
12 aos de la cifra de inicio es apenas del 5%-y es excepcional que la enfermedad se inicie
en la edad adulta. La enfermedad sufre exacerbaciones en las temporadas de calor o de
fro cuando hay sequedad de la atmsfera.
CUADRO CLNICO
Tradicionalmente se han descrito tres etapas que en la actualidad ya no se presentan pues
se suman unas a otras debido a los tratamientos que reciben los pacientes desde su inicio.
Etapa de lactante. La enfermedad suele iniciarse en los primeros meses de vida, a veces
casi desde el nacimiento. Las lesiones afectan la cara: mejillas, frente (respetando su
centro); afectan tambin la piel cabelluda, los pliegues, las nalgas y el dorso de los pies. Las
lesiones son de tipo eccematosos (piel llorosa): eritema, vescular y costras melicricas con
costras hemticas como signo del rascado. Estas lesiones altamente pruriginosas
evolucionan por brotes y en general si el paciente no es yatrognicamente tratado, tienen
tendencia a involucionar al ao de edad.
Fase del escolar. Se inicia hacia los 3 a 7 aos, cuando el nio empieza a ir a la escuela. Las
lesiones son preferentemente flexurales: Cuello, pliegues de codo y huecos poplteos y la
morfologa corresponde a la de una dermatitis crnica: zonas de eritema y liquenificacin
(ndice de rascado crnico) y costras hemticas peridicamente sufren un proceso de
eccematizacin sobre todo por los tratamientos indebidos. La enfermedad evoluciona por
brotes, hay temporadas en que no existen lesiones aun cuando persiste a veces una piel
xertica (seca) y pigmentada con intenso prurito.
Etapa del adulto. Se vea con poca frecuencia y ahora es habitual en las consultas diarias
debido a los malos tratamientos. En este caso adems de las zonas flexurales, se presenta
lesiones periorbitarias; y peribucales y lesiones vesiculosas en las manos. Son lesiones tanto
liquenificadas como eccematosas, muy pruriginosas, que alternan con periodos
asintomticos.

Con el uso inmoderado de los corticoesteroides no hay diferenciacin entre las etapas de la
enfermedad y se ven casos que arrastran su enfermedad casi desde que nacen hasta la
edad adulta.

DIAGNSTICO
Es clnico y relativamente sencillo, aunque no toda dermatitis flexural es necesariamente
de origen atpico. Deben tomarse en cuenta los antecedentes del propio enfermo y los
familiares. En los lactantes debe diferenciarse de la dermatitis seborreica que afecta
sobre todo la piel cabelluda y el centro de la cara. En ocasiones hay mezcla de las dos
dermatitis: la atpica y la seborreica y es difcil diferenciarlas. La dermatitis del paal
predomina en los lactante; en las zonas glteas y genitales, se presenta con eritema y
lesiones vesculopapulosas muy pruriginosas. La pitiriasis alba corresponde a los clsicos
jiotes de los nios: manchas hipocrmicas cubiertas de fina escama en la cara.
Estos procesos pueden coincidir con manifestaciones de DA, pero nunca se ha demostrado
la relacin de causa efecto.
TRATAMIENTO
Medidas generales. Es conveniente una explicacin amplia al paciente y su familia
sealando la naturaleza y evolucin de la enfermedad y lo que se espera del tratamiento
que vamos a indicar.
Evitar el sol excesivo, el agua clorada de las albercas, el uso de jabn (indicar jabones
neutros o sustitutos de jabn), prohibir el uso de pomadas y remedios caseros as como los
cosmticos que irritan a la piel.
Las dietas restrictivas han sido y siguen siendo un tema muy controversial, pues mientras
los alergistras siguen insistiendo sin muchas bases en el beneficio de estas dietas, los ms
hemos comprobado su inutilidad. Existe un grupo muy limitado en que se puede demostrar
que una dieta restrictiva de huevo, leche, fresas, etc. puede mejorar los brotes de DA y en
tales casos (slo en esos casos) se aconsejara la supresin de tales medicamentos; en lo
general se permite al paciente que coma de todo. En la actualidad se estn limitando por los
pediatras ciertos alimentos como el huevo, las fresas, el pltano y la leche de vaca en el
primer ao de vida por ser muy alergnicos y se discute la utilidad de la leche materna en
estos nios.
Es necesario hacer ver que estos pacientes requieren de una atencin ms personal, que
sienten la necesidad de cario por lo que es de recomendarse que el nio sea atendido
personalmente por la madre.

Tratamiento tpico. Depende del estado de la piel; si est eccematosa debe secarse antes
de aplicar cualquier pomada que ser rechazada por la piel llorosa. El uso de fomentos con
agua de manzanilla o suero fisiolgico es til. Los fometos con agua de vgeto (subacetato
de plomo) al 20% son potentes antiexudativos, pero deben limitarse a reas restringidas y
no usarse en nios. Estando la piel ya seca, se usan pasta inertes que llevan xido de zinc y
calamina en una base de vaselina y lanolina que son protectoras a la vez que antiprutiginosas.
Si la piel est muy seca y liquenificada, los fomentos y baos sern emolientes, con almidn
y aceites seguidos de cremas o pomadas ms grasosas que llevan vaselina, coldcream y
xido de zinc. En el comercio existen numerosos preparados humectantes que ayudan a
mantener el manto cido grasa-agua, que se pierde en la DA. Si hay dermatitis por contacto
o imptigo hay que tratar primero estas complicaciones con sulfato de cobre al 1: 1000 y
pomadas con vioformo o mupirocn. En el caso de eritrodermia, los baos emolientes y el uso
del petrolato (vaselina) ser lo indicado.
Cuando hay mucha liquenificacin, pueden usarse cremas con alquitrn de hulla al 3% en
base de coldcream por tiempo y zonas limitadas.
Los corticoesteroides tpicos son los medicamentos ms usados en esta enfermedad y
muchas veces causa de las complicaciones que se presentan. Estos medicamentos no curan
nada, solo engaan al paciente y al mdico hacindoles creer que la enfermedad va curando
cuando slo se oculta y modifica. Al pasar el efecto de estos medicamentos,
invariablemente se presenta el rebote y ms tarde la corticodependencia, haciendo a la
enfermedad incontrolable. Nunca deben usarse los corticoesteroides fluorinados en nios y
en la cara y zonas genitales o en los pliegues por su posibilidad de absorcin. La
hidrocortisona es de baja potencia, hace menos dao, pero tambin es menos efectiva;
algunos la recomiendan.
Tratamiento sistmico. El uso de antihistamnicos sobre todo de la primera generacin que
son sedantes como la clorfeniramina y la hidroxicina ayudan a mejorar el prurito; el
ketotifeno y la oxotamida por su accin dual: inhiben la produccin de histamina por los
mastocitos y bloquean los receptores Hl, tambin son de ayuda, al igual que los sedantes
suaves tanto para el paciente como para la madre que est en perenne angustia que
transmite al pequeo paciente. La talidomida ha mostrados ser de ayuda en casos de DA
conticoestropeada, en su fase eritrodrmica, no tanto en los nios, a dosis de 100 mg al da.
Los antibiticos tipo dicloxacilina, sern necesarios cuando haya infeccin o simplemente
eccematizacin por el papel que tiene el estafilococo dorado.

1. Criterios Diagnstico para Dermatitis Atpica (DA), MedicalCriteria.com.


2. Dr. Luciano Domnguez-Soto, Dr. Amado Sal Cano, Dermatitis atpica o
neurodermatitis diseminada, Dermatologa, Parte C, Libro ) "

32.- Masculino de 48 aos de edad, presenta erupcin cutnea descamativa asintomtica,


con afectacin del cuero cabelludo, cejas, pestaas, eminencias malares y pliegues
nasolabiales. En la exploracin se pone de manifiesto una descamacin seborreica sobre una
piel moderadamente eritematosa en las reas afectadas. En la regin anterior del trax se
observa un moderado enrojecimiento con descamacin. El diagnstico ms probable es:
a) Dermatitis seborreica.
b) Psoriasis.
c) Lupus eritematoso.
d) Tia corporis.

CLNICA
La dermatitis seborreica se puede clasificar en: 1) Dermatitis Seborreica en la infancia: a)
Costra lctea. B) Dermatitis seborreica infantil. C) Falsa tia amiantcea. D) Eritrodermia
descamativa de Leiner-Mousses. 2. Dermatitis Seborreica en el adulto: Pitiriasis cpitis. B)
Pitiriasis esteatoide.
C) Dermatitis seborreica facial, mediotracica y medioescapular y pbica. 3. Dermatitis

Seborreica en enfermos inmunocomprometidos.


La costra lctea se manifiesta en los primeros meses de vida y se manifiesta mediante

placas escamosas, untuosas al tacto, de color grisceo que localizan en cuero cabelludo (Fig.
1). Tambin pueden existir lesiones eritemato-descamativas centrofaciales y
mediotorcicas. La falsa tia amiantcea se manifiesta como gruesas escamocostras en
cuero cabelludo, de color blanco-grisceo que al despegarse dejan zonas hmedas. La
eritrodermia descamativa de Leiner Mousses comienza de forma repentina, a los pocos
meses de vida, con lesiones en cuero cabelludo, evolucionando de forma rpida a lesiones
eritematosas y descamativas que ocupan casi todo el tegumento.
Suele acompaarse de candidiasis de pliegues, infecciones locales y sistmicas, diarreas. A
veces el pronstico es malo. Las manifestaciones clnicas de la dermatitis seborreica
infantil o clsica del adulto slo se manifiestan en la edad de presentacin.
Conforman lesiones eritemato-descamativas que localizan en la zona de insercin de cuero
cabelludos, pliegues retroauriculares, conducto auditivo externo, surcos nasolabiales y
nasogeniamos, cejas, glabela, zona mediotorcia e interescapular, axilas, ingles, ombligo,
pubis . Suele ser pruriginosa. En pacientes HIV (+) las manifestacionesclnicas de la
dermatitis seborreica son mucho ms intensas.
DERMATITIS SEBORREICA
Dres. E. Herrera y G. Ruz del Portal.

33.- Femenino de 52 aos con antecedente: Dx. De sndrome anmico sin causa aparente.
Exmenes de Laboratorio Reportan: Hb de 8,5 g/dl, VCM de 75 fl, y HCM de 25 pg. El
examen de laboratorio que conforma el origen del diagnstico de esta paciente es la
determinacin de:
a)

Sideremia.

b)

Saturacin de la transferrina.

c) Hemoglobina A2.
d) Ferritina.

La anemia por dficit de hierro es un proceso que aparece de forma rpida o de


forma ms bien lenta?
La anemia por falta de hierro se va estableciendo de forma gradual, cuando el hierro que se
pierde supera al hierro aportado por la dieta; lo primero que ocurre es que el organismo
empieza a utilizar las reservas de hierro. En esta etapa la ferritina empieza a descender,
pero el hierro srico y la TIBC generalmente permanecen inalterados no establecindose
todava anemia. En el momento en que el dficit de hierro empeora, el hierro srico
disminuye mientras que la transferrina y la TIBC aumentan, empezando las clulas de la
serie roja (hemates) a palidecer (hipocromia) y a disminuir de volumen (microcitosis),
existiendo todava un nmero suficiente de clulas de la serie roja. Si la deficiencia de
hierro contina o empeora aparece entonces la anemia.
Mourey L. Manual de procedimientos del Laboratorio Clnico. Mxico:
IMSS, 1978: 158.

34.- Femenino acude a su consultorio con diagnstico de e anemia microctica e


hipocrmica, Resultados de laboratorio reportan: hierro srico y ferritina bajos, aumento
de la capacidad de fijacin del hierro y de la protoporfirina eritrocitaria, con hemoglobina
A2 disminuida. La entidad responsable de los resultados de laboratorio de esta paciente
es:
a) Anemia por deficiencia de hierro.
b) Anemia de los trastornos crnicos.
c) Anemia sideroblstica.
d) Talasemia beta.

Frotis de sangre perifrica: hipocroma, microcitosis, policromatofilia o punteado


basfilo(eventualmente). Reticulocitos: Generalmente normales. Si estn aumentados,
investigar prdidas por hemorragia o posibilidad de otro diagnstico Plaquetas: normales.
Si estn elevadas, investigar prdidas por hemorragia. Leucocitos: normales. ndices
hematimtricos: volumen corpuscular medio (VCM) y concentracin de hemoglobina
corpuscular media (CHCM) disminuidos. Los lmites inferiores normales (X 2 DE) que se
deben considera a distintas edades Pruebas que evalan el hierro del
Compartimiento funcional: Sideremia: disminuida (< 60 g/dl). Capacidad total de saturacin
del Hierro (TIBC): aumentada, a meno que coexista proceso infeccioso, inflamatorio o
tumoral. Porcentaje de saturacin: disminuido (< 16%). Protoporfirina libre eritrocitaria:
aumentada (> 70 g/dl). Receptores solubles de transferrina: aumentados (> 30 nMol/l). c3.
Pruebas que evalan el hierro del compartimiento de depsito: Ferritina srica: Disminuida
(< 12 ng/ml), excepto que coexista proceso infeccioso o inflamatorio. Medulograma: hierro
de depsito (SRE) ausente.
Anemia ferropnica. Normas de diagnstico y tratamiento / 163
Arch.argent.pediatr 2001; 99(2) / 162

35.- Masculino de 28 aos, acude a consulta con resultado de exmenes de laboratorio:


microcitosis, hemates 5280000/microl, hemoglobina 12,3 g/dl, Hcto 36,6%, VCM 69,4 fl,
HCM 23,3 pg, serie roja microctica e hipocroma. Sideremia 78 microg/dl, transferrina 312
microg/dl, IST 25%, ferritina 71 ng/ml, HbA2 2,0%, HbF 0,6%, Antecedentes: 3 aos
antes, de melenas, hematoquecia por hemorroides externas. Tratado con hierro.
El diagnstico ms probable en este paciente es:

a) Anemia ferropnica parcialmente tratada.


b) Anemia sideroblstica adquirida.
c) Portador de beta talasemia.
d) Rasgo alfa talasmico.

Talasemia
Talasemia es el nombre genrico con el que se conoce a un grupo de enfermedades
hereditarias de la sangre que incluyen anomalas en la hemoglobina, el componente de los

glbulos rojos encargado de transportar el oxgeno. La hemoglobina est compuesta


principalmente por dos clases de protenas denominadas globina alfa y globina beta. Las
personas que padecen talasemia no producen suficiente cantidad de una de estas protenas

(y, en ocasiones, de las dos). En consecuencia, sus glbulos rojos pueden ser anormales y no
estar en condiciones de transportar suficiente oxgeno por todo el cuerpo.
Los dos tipos principales de talasemia se denominan talasemia alfa y talasemia beta. Las
personas afectadas por talasemia alfa no producen suficiente cantidad de globina alfa y las
afectadas por talasemia beta no producen suficiente cantidad de globina beta. Existen
distintos tipos de talasemia alfa y beta, con sntomas que van de leves a graves.
La talasemia se encuentra entre los trastornos genticos ms comunes en todo el mundo.1
Cada ao nacen en todo el mundo ms de 100,000 bebs afectados con formas graves de
talasemia.2 Esta enfermedad ocurre con mayor frecuencia en personas de ascendencia
italiana, griega, asitica y africana, as como en aquellas de familias oriundas del Medio
Oriente.3
Qu es la talasemia alfa?

Existen al menos cinco tipos principales de talasemia alfa, que afectan principalmente a
personas de ascendencia hind, africana, del sur de China, de Medio Oriente y del sudeste
asitico.4 Los genes que controlan la produccin de globina alfa son cuatro. La gravedad de
la enfermedad se determina de acuerdo con el nmero de genes anormales o faltantes.

El portador silencioso, la forma ms leve de la enfermedad, tiene un solo gen de


globina alfa anormal o faltante. Por lo general, las personas afectadas no presentan
sntomas pero pueden transmitir la anomala gentica a sus hijos.

Las personas con talasemia alfa menor (tambin denominada rasgo talasmico alfa)
tienen dos genes de globina alfa anormales o faltantes. Por lo general, las personas
afectadas no presentan sntomas o presentan una anemia leve pero pueden
transmitir la enfermedad a sus hijos.

La enfermedad de la hemoglobina H es causada por tres genes de globina alfa


faltantes o anormales (las personas afectadas tienen un gen de globina alfa normal).
La enfermedad trae como consecuencia anomalas en los glbulos rojos y la rpida
destruccin de los mismos. La mayora de las personas afectadas padece anemia de
leve a moderada y puede llevar una vida relativamente normal. La anemia puede
empeorar temporalmente cuando la persona afectada contrae infecciones virales o
cuando se trata con ciertos medicamentos (tales como medicamentos sulfa).5
Algunas personas afectadas pueden desarrollar complicaciones con el tiempo como,
por ejemplo, el agrandamiento del bazo y clculos biliares.5 Las personas que
padecen la enfermedad de la hemoglobina H deben recibir atencin mdica regular
para detectar y tratar estas complicaciones. Algunas incluso pueden necesitar
transfusiones de sangre ocasionalmente.6

La hemoglobina H-Constant Spring es una forma ms grave de la enfermedad de


hemoglobina H. Las personas afectadas tienen un solo gen de globina alfa normal y
una mutacin especfica denominada Constant Spring en uno de sus tres genes
anormales. Las personas afectadas por esta enfermedad generalmente padecen

anemia de moderada a grave y a menudo desarrollan complicaciones, tales como el


agrandamiento del bazo. En algunas oportunidades, algunas necesitan recibir
transfusiones de sangre, por ejemplo, cuando desarrollan una enfermedad con
fiebre, mientras que otras necesitan recibir transfusiones con mayor frecuencia.5,6

La talasemia alfa mayor, el tipo ms grave, se da cuando no hay genes para la


produccin de globina alfa. Los fetos afectados padecen anemia grave, insuficiencia
cardaca y acumulacin de lquido. Por lo general, nacen sin vida o mueren pocas
horas despus del parto. En casos poco frecuentes, han sobrevivido bebs a los que
se diagnostic la enfermedad antes de nacer y que recibieron tratamiento mediante
transfusiones de sangre. Estos bebs necesitan recibir transfusiones de sangre de
por vida.4,5

Qu es la talasemia beta?
Existen tres formas principales de talasemia beta, que afectan principalmente a personas
con ascendencia griega, italiana, africana, de Medio Oriente, del sudeste asitico y del sur
de China.4 El control de la produccin de globina beta est a cargo de dos genes y las
mutaciones en uno o ambos genes pueden provocar este trastorno. La gravedad de la
enfermedad depende de si uno o ambos genes de globina beta son portadores de una
mutacin y de la gravedad de la mutacin.

La talasemia menor (tambin denominada rasgo talasmico) es causada por una


mutacin en un gen de globina beta. La mayora de las personas afectadas no
presenta sntomas aunque algunas padecen anemia leve. Las personas afectadas
pueden transmitir el gen anormal a sus descendientes.

La talasemia intermedia es resultado de anomalas en ambos genes de globina beta.


Por lo general, estas anomalas genticas son menos graves que las que causan la
talasemia mayor. Por lo general, los nios afectados padecen anemia de leve a
moderada y pueden desarrollar algunas de las complicaciones que se observan en la
talasemia mayor, como agrandamiento del brazo y anomalas en los huesos. Muchas
de las personas afectadas necesitan, ocasionalmente o con mayor frecuencia,
transfusiones de sangre para reducir las complicaciones.1

La talasemia mayor, la forma ms grave, es resultado de mutaciones graves en


ambos genes de globina beta. Tambin se denomina anemia de Cooley, como
homenaje al mdico que la describi por primera vez en el ao 1925. La mayora de
los nios afectados parecen saludables al nacer. No obstante, durante el primer o el
segundo ao de vida se vuelven plidos e irritables y pierden el apetito. Su
crecimiento es lento y a menudo tienen ictericia (su piel y sus ojos adquieren un

color amarillento). Sin el tratamiento adecuado, estos nios desarrollan un bazo y


un hgado de mayor tamao, huesos dbiles que se quiebran con facilidad, huesos
faciales anormales, infecciones frecuentes y problemas cardacos y mueren dentro
de los primeros diez aos de vida. Los nios afectados requieren transfusiones de
sangre peridicas desde la lactancia.

Referencias:
1. Rund, D. and Rachmilewitz, E. Medical Progress: Beta-Thalassemia. New England Journal
of Medicine, volumen 353, nmero 11, 15 de septiembre de 2005, pgs. 1135-1146.
2. New York Academy of Sciences. Cooleys Anemia Eighth Symposium. Publicado 22 de
julio de 2005, consultado 2 de mayo de 2008, www.nyas.org/cooleys.
3. National Heart, Lung and Blood Institute. Thalassemias. Publicado
www.nhlbi.nih.gov/health/dci/Diseases/Thalassemia/Thalassemia_All.html.

1/08,

4.
Cooleys
Anemia
Foundation.
http://www.cooleysanemia.org/.

2007,

About

Thalassemia.

Actualizado

5. Northern California Comprehensive Thalassemia Center. Alpha Thalassemia. Consultado


2 de mayo de 2008, www.thalassemia.com/alpha_thal.html.
6. Cohen, A.R., et al. Thalassemia. Hematology 2004, American Society of Hematology,
pgs. 14-34.
7. Food and Drug Administration (FDA). FDA Approves First Oral Drug for Chronic Iron
Overload. FDA News, 9 de noviembre de 2005.

36. - A 19 year old man with acute non lymhocytic leukemia is admitted to he hospital 2
weeks after hi first round of chemotherapy. His temperature is 39.2 C, and physical
examination shows a no localized abnormalities. Chest radiograph shows a Hickman
catheter with its tip in the right atrium. The white blood cell is 300/uL with no
polymorphonuclear or band cells in the differential count. Blood cultures are obtained. The
next step is to

a) Initiate antistaphylococcal treatment for the possibility of Hickman catheter


related bacteremia.
b) Administer broad spectrum antibiotics with excellent activity for enteric
gramnegative rods ad Pseudomonas aeruginosa.
c) Await results of blood cultures and other diagnostic tests because infection could
be caused by almost any microorganism.

d) Administer parenteral antifungal therapy.

Pseudomonas aeruginosa es un patgeno oportunista que causa infecciones del aparato


urinario y respiratorio, de los tejidos blandos, endocarditis y una variedad de infecciones
sistmicas, particularmente en pacientes quemados, con cncer, fibrosis qustica o
compromiso del sistema inmune. Pseudomonas aeruginosa ha emergido como uno de los ms
importantes patgenos hospitalarios causantes de infecciones graves. Las infecciones
asociadas a catteres son una importante causade morbi-mortalidad. Caractersticas en
pacientes con infeccin relacionada a catteres: Infeccion local; fiebre de origen
desconocido en paciente con catter de ms de 3 das; con hemocultivos positivos sin otro
foco probable; normalizacin de la temperatura luego de la retirada del dispositivo.

IDSA Guidelines for the management of intravascular catheter related infections. Clinical
Infectious Diseases 2001; 32:1249.
Centers for Disease Control/ Hospital Infection Control Practices Advisory Committee.
Guidelines for Prevention of Intravascular
Device-Related Infections American Journal of Infection Control 1996: 24: 262-293.

37.- Es una de las enfermedades tiroideas autoinmunes organo-especfica ms frecuentes.


Es ms comn en la mujer, posee una asociacin directa con otras enfermedades
autoinmunes y, por lo general, se presenta con hipertiroidismo, bocio difuso, oftalmopata,
adems, en algunos casos coincide con mixedema pretibial:
a)
b)
c)
d)

Carcinoma basocelular
Enfermedad de Graves
Adenoma txico
Sndrome del eutiroideo enferm

Introduccin
La enfermedad de Graves-Basedow (EGB) constituye la entidad ms importante, por su
frecuencia, entre las enfermedades que producen hipertiroidismo. Se caracteriza por la
presentacin de la trada sintomtica constituida por hipertiroidismo, bocio difuso y
oftalmopata (50%). Es ms frecuente en la 3 y 4 dcada de la vida y afecta con ms

frecuencia a mujeres (7/1 en zonas no bocigenas y 3/1 a partir de los 45 aos en zonas de
bocio endmico) 1.

Etiopatogenia
Aunque no se conocen exactamente los factores que inician y mantienen la enfermedad, hay
elementos que permiten encuadrarla en el marco de la patologa autoinmunitaria, como la
presencia de autoanticuerpos antitiroglobulina y antitiroperoxidasas (TPO), as como la
presencia de inmunoglobulinas antireceptor de la TSH (TRAb) que son estimuladoras de la
funcin y del crecimiento del tiroides.
Para el desarrollo de la enfermedad parece que es necesaria la intervencin de factores
genticos y ambientales (exceso de Yodo) 2.

Clnica
La enfermedad se caracteriza por la presencia de sntomas de hipertiroidismo (nerviosismo
99%, sudoracin 91%, intolerancia al calor y palpitaciones 89%, prdida de peso 85%,
aumento de apetito 69% y sntomas oculares 55%) y signos clnicos (aumento uniforme del
tamao de la glndula 97%, oftalmopata 60%, taquicardia en reposo 90%, temblor 70% y
retraccin palpebral 38%) relacionados con el aumento de receptores adrenrgicos que
determinan las hormonas tiroideas 1,2.
La historia natural de la enfermedad se caracteriza por fases cclicas de exacerbacin y
remisin, de duracin y presentacin imprevistas; aunque, hoy en da, est artefactada por
los tratamientos utilizados. En aproximadamente el 25% de los pacientes, especialmente
en aquellos con una forma leve de enfermedad, el proceso se autolimita al ao o ms,
regresando espontneamente a un estado eutiroideo 3.
En cuanto a la oftalmopata, hay una forma no infiltrativa o plpebro-retrctil (afecta ms
frecuentemente al sexo femenino, afecta simtricamente a ambos ojos y que evoluciona
paralelamente al hipertiroidismo) y una forma infiltrativa (infiltracin del tejido
retroorbitario por mucopolisacridos y linfocitaria con protusin ocular asimtrica)1.
Otras manifestaciones menos frecuentes son la aparicin de mixedema pretibial o
dermopata infiltrativa y, la acropaquia tiroidea 1.

Referencias
1.- Foz M. Enfermedades del tiroides. En Rozman C (ed): Farreras-Rozman Medicina interna.
Editorial Doyma. Barcelona, 1992:1997.

2.- Wartofsky L. Diseases of the thyroid. En Braunwald E, Isselbacher KJ, Wilson JD,
Martin JB, Fauci AS, Kasper DL (eds): Harrisons principles of internal medicine. Editorial
McGraw-Hill. EEUU, 1994: 1930.

3.- De Groot. Graves diseases and the manifestations of thyrotoxicosis. En De Groot LJ,
Reed Larsen P, Hennemann G.(eds): The thyroid and its diseases. 1996: 371.
4.- Haynes RC, Murad F. Drogas tiroideas y antitiroideas. En Goodman LS, Gilman A (eds):
Las Bases farmacolgicas de la teraputica. Editorial Panamericana. Mxico, 1982: 1376.
5. - Feliciano DV. Everything you wanted to know about Graves disease. Am J Surg 1992,
(164): 404.

38.- Se trata de masculino de 70 aos que consulta por pirosis, disfagia leve ocasional y
episodios de regurgitacin nocturna desde hace 15 das. Comenta que desde hace 2 aos
viene presentando por ocasiones pirosis y regurgitacin. Se reporta endoscopia alta con
esofagitis erosiva grave. Qu tratamiento farmacolgico entre los siguientes, es el ms
adecuado?
a) Antagonistas de los receptores H2.
b) Sucralfato.
c) Inhibidores de la bomba de protones.
d) Tratamiento combinado con anti-H2 y sucralfato.

Se ha demostrado que existe una relacin directa entre la duracin de la supresin del
acido gstrico y la menor acidez del esfago.
Los inhibidores de la bomba de protones (IBP) son los agentes preferidos para la curacin
de las lesiones agudas y para la mantencin de remisin. Los bloqueadores H2 presentan
como nica ventaja su rapidez de accin, pero su potencia es menor. Su uso de rutina
asociado con los IBP no se recomienda, ya que los IBP actan sobre las bombas de cido
activadas y cualquier inhibicin de la secrecin por otro agente retarda su mximo efecto
Los anticidos son tiles para sntomas ocasionales y su uso no se contrapone con los IBP.
De no existir factores significativos modificables en los hbitos o en la anatoma, la ERGE
debe considerarse frecuentemente una patologa crnica. No existe evidencia de peso que
contraindique el uso crnico de IBP. Los pacientes con esofagitis erosiva son los ms
susceptibles a desarrollar complicaciones que los en GERD (endoscopy negative

Gastroesophageal reflux disease).Todos los IBP son tiles pero no son necesariamente
iguales. Puede haber diferencias tnicas en el nmero de clulas parietales o polimorfismo
del citocromo p450 con diferente metabolizacin de los IBP.
La infeccin por Helicobacter pylori confunde cualquier intento de comparar IBP. La
gastritis de predominio antral puede producir hipergastrinemia e hipersecrecin. Si es de
predominio corporal puede disminuir la produccin de cido. Los estudios comparativos
deben hacerse en individuos sin esta infeccin.
La duracin del efecto de los IBP es importante. Muchas publicaciones demuestran la
mayor duracin de la accin de la accin del esomeprazol, pero clnicamente doble dosis de

los otros IBP pueden tener resultados comparables. Katz y cols compararon los 5 IBP
disponibles en pacientes con ERGE, todos resultaron en pH > 4 por al menos 8 a 10 h, pero
la duracin de accin del esomeprazol fue superior en el rango de los pH intragstricos
entre 2 a 6.
En los pacientes con enGERD el efecto de los IBP no es tan dramtico, puede deberse a que
no hay tantos elementos objetivos de juicio como la curacin de las erosiones.
Los IBP pueden usarse una vez al da, en la maana y antes del desayuno. Los pacientes con
dao extraesofgico o cuadros severos obtienen mejor efecto con doble dosis fraccionada
(Antes de desayuno y cena). Esto mejora el control de la acidez nocturna.
En los pacientes cuya pHmetra demuestre que persiste escape nocturno algunos favorecen
agregar un antagonista H2 al acostarse. Rackoff y cols demostraron que 74% de los
pacientes presentaban mejora de los sntomas nocturnos. Una nueva formulacin de
liberacin y absorcin rpida de omeprazol tambin podra ser til en estos casos.
La ausencia de sntomas no significa ausencia de dao. Los casos graves (y por supuesto el
Barrett) deben tener seguimiento endoscpico para asegurar su evolucin. Un porcentaje
significativo de pacientes sin Barrett continan presentando RGE patolgico y bajo pH
intragstrico a pesar de IBP bien llevado con total remisin de sntomas.
El primer objetivo es la remisin de los sntomas de reflujo clsico. Esto debe lograrse en
la primera semana si la prescripcin es adecuada. Las manifestaciones extraesofgicas
tardan varios meses en controlarse, pero deben exhibir una mejora rpida en las primeras
semanas que confirme la buena orientacin de las medidas indicadas.
La estrategia teraputica de comenzar el tratamiento en forma poco agresiva y escalarlo si
no hay respuesta, me parece poco adecuada.
Al paciente se le debe insistir que los frmacos son una parte importante del tratamiento
pero no la nica y que los cambios de hbitos y costumbres, la baja de peso, el ejercicio
Etc, son claves para el xito.

Bibliografa:
1.- Miner P, Katz P, Chen Y, Sostek M. Gastric acid control with esomeprazole,
lansoprazole, omeprazole,
pantoprazole, and rabeprazole: A five-way crossover study. Am J Gastroenterol 2003; 98:
2616-20.

2. Katz P, Miner P, Chen Y, Sostek M. Effects of 5 marketed proton pump inhibitors on


acid suppression relative to a range of pH thresholds. Am J Gastroenterol 2004; 99: S34.
3.- Rackoff A, Agrawal A, Hila A, et al. Histamine-2 receptor antagonists at night improve
GERD symptoms for patients on proton pump inhibitor therapy. Am J Gastroenterol 2004;
99: S18.
4.- Castell D, Goldlust B, Morelli G, et al. Omeprazole immediate-release oral suspension is
more effective than pantoprazole delayed-release capsules in reducing nighttime gastric
acidity in GERD patients. Am J Gastroenterol 2004; 99: S39.
5. Milkes D, Gerson L, Triadafilopoulos G. Complete elimination of reflux symptoms does
not guarantee normalization of intraesophageal and intragastric pH in patients with
gastroesophageal reflux disease(GERD). Am J Gastroenterol 2004; 99: 991-6.

39.- Se trata de paciente masculino de 72 aos que cursa con evento vascular cerebral
isqumico El tratamiento de eleccin en este padecimiento es?

a) Antiagregantes, hemorreologicos, anticonvulsivantes.


b) Vasodilatadores, anticonvulsivantes, derivados sanguneos.
c) Medidas generales, hipoglucemiantes, analgsicos.
d) Vasodilatadores, analgsicos, sedacin.

Tratamiento:
z

Fase aguda ataque isqumico menor


* Estabilizacin general
- 02 por puntas nasales
- Control de la TA y alteraciones cardiacas (FA)
- Correcin de variantes metablicos (ES, glucosa, etc.)
* Con o sin crisis convulsivas
- DFH 750 mg. IV impregnacin con 125 mg. IV c/8 hr.
- Carbamacepina 200 mg. VO cada 8 hrs
* Mejorar estado circulatorio
- Pentoxifilina 400 VO c/ 8 hrs
- ASA 125 mg VO c/ 24 hrs

Atacar causa subyacente:


- Control adecuado de la TA y la glicemia
- Control de hiperlipidemia e hipercolesterolemia
- Rehabilitacin temprana
- Eliminar tabaquismo

* Vigilancia tomogrfica de la evolucin


* Limitar el dao y prevenir su extensin
* Educacin al paciente y a la familia

1.Aronson A y cols. Examen clnico neurolgico, 3 Edicin. La Prensa Mdica Mexicana,


Mxico, 1995.
2. Uribe CS, Arana A, Pombo PL. Neurologa, 5 Edicin. Corporacin para investigaciones
biolgicas. Colombia, 1996.
3.-Adams R, Vctor M. Principles of Neurology. Mc Graw Hill. 7th ed. USA 2001.
4.-Bradley W.G. Neurology in clinical practice. Butterworth Heinemann. 4th ed.

40.- Se trata de femenino de 31 aos, con un cuadro clnico de sangrado menstrual


irregular, dismenorrea, dispareunia, y una esterilidad de 3 aos de evolucin, con
resultados de laboratorio hormonal normal y un estudio ecogrfico transvaginal que informa
de un tero normal y sendas formaciones qustica ovricas bilaterales de 4 cms. con signos
ecogrficos de sospecha. Cul sera la orientacin diagnstica?
a) Hemorragia uterina disfuncional.
b) Sndrome del ovario poliqustico.
c) Endometriosis.
d) Quistes dermoides bilaterales.

Causas y sntomas de endometriosis:


Las causas de la endometriosis an no se conocen. Las clulas del revestimiento
interno del tero de alguna manera se desplazan hasta zonas externas al mismo y
siguen creciendo. Este desplazamiento podra quizs deberse a que pequeos
fragmentos del revestimiento uterino, desprendidos durante la menstruacin,
retrocedan hacia las trompas de Falopio en direccin a los ovarios hasta entrar

en la cavidad abdominal, en lugar de salir con el flujo menstrual a travs de la


vagina.
La endometriosis causa dolor en la parte inferior del abdomen y la zona plvica,
irregularidades menstruales (como manchar antes de la menstruacin) e
infertilidad. Algunas mujeres con endometriosis grave no presentan sntomas,
mientras que otras con la enfermedad en grado mnimo sufren un dolor
invalidante. Con frecuencia, el dolor menstrual debido a la endometriosis no
aparece hasta aos despus de desarrollar la enfermedad. En algunos casos, se
constata dolor durante el coito (dispareunia), antes o durante la menstruacin.
El tejido endometrial adherido al intestino grueso o a la vejiga urinaria puede
provocar hinchazn abdominal, dolor durante las deposiciones, hemorragia rectal
durante la menstruacin o dolor en la parte inferior del abdomen durante la
miccin.

As mismo, cuando el tejido se localiza en un ovario o una estructura cercana


puede dar lugar a la formacin de una masa llena de sangre (endometrioma). En
ocasiones, el endometrioma se rompe bruscamente o se escapa algo de su
contenido, lo que causa un agudo y repentino dolor abdominal.

41.- Masculino de 40 aos recin llegado del estado de Chiapas, Mx. Acude a consulta por
referir presentar fiebre elevada de tres das de tres das de evolucin refiere artralgias
francas y cefalea intensa. El mismo da de su visita a urgencias comenz a presentar un
exantema maculo-papuloso pruriginoso. El examen de la sangre mostr los siguientes datos:
Valor hematocrito 38%, Leucocitos 3600 p.mm3 con 82% neutrfilos y 12% linfocitos.
Plaquetas 115.000 p.mm3. Placa de trax normal. El paciente fue dado de alta con el
diagnstico de cuadro viral, en tratamiento con paracetamol y antihistamnicos para el picor.
A las 48 horas volvi a urgencias con muy mal aspecto: estaba afebril, tena confusin
mental, se observaban petequias en antebrazos y piernas, edema en pies, TA 85/70. Pulso
110 l.p.min. de amplitud pequeo. En los nuevos exmenes de la sangre destacaban: Valor
hematocrito 46%, leucocitos 3600 p.mm3 sin cambios en la frmula y plaquetas 65.000
p.mm3. Glucosa 106 mg.p. dl. Creatinina 1,8 mg.p.dl. Sodio 126 mEq/l. Potasio 4,2 mEq/l. La
placa de trax mostraba un pequeo derrame pleural bilateral. Cul es, entre los
siguientes, el diagnstico ms probable?

a) Dengue.
b) Meningoencefalitis bacteriana.
c) Fiebre tifoidea.
d) Neumona por Legionella Neumophila.

El dengue es actualmente la ms importante arbovirosis que afecta al hombre. Su agente


etiolgico son los 4 serotipos del virus del dengue (D1-4). Se transmite entre humanos
mediante la picadura del mosquito Aedes aegypti. Se estima que el 40% de la poblacin
mundial vive en reas de riesgo de esta entidad (1,2).

La forma clnica ms grave, la fiebre hemorrgica de dengue/sndrome de choque del


dengue (FHD/SCD), fue casi exclusiva del Sudeste Asitico y el Pacfico Occidental hasta
1981, cuando una gran epidemia de dengue, la mayor de las Amricas, que incluy ms de
10.000 casos de FDH/SCD, ocurri en Cuba. Esta constituy la primera epidemia de DH en
la regin (3). Posterior a este brote y hasta la actualidad la FHD contina presentndose de
forma endmica en diferentes pases, ocurriendo epidemias de forma frecuente (4).

Cuadro clnico
El curso de la enfermedad, desde el punto de vista clnico, se puede dividir en cuatro fases:
Inicial, crtica, de recuperacin y de convalecencia.
Fase inicial: En ella el enfermo tiene un sndrome febril sin localizacin. Es muy
sintomtica y adems de la fiebre predominan las manifestaciones generales como cefalea,
artromialgias, dolor retroocular y malestar, puede aparecer rash.
Suele durar alrededor de 3 das, tras lo cual la fiebre cede y algunos casos de dengue
clsico comienzan a presentar manifestaciones hemorrgicas leves, pero la mayora tienden
a mejorar. Otro grupo menor de pacientes desarrollar la FHD/SCD. Los llamados signos de
alarma que preceden al choque pueden comenzar en esta fase.

Fase crtica: Transcurre entre el 4to y 7mo da de la enfermedad, en ella se presentan los
sntomas que definen al DH, como son la extravasacin de plasma, la trombocitopenia, y las
manifestaciones hemorrgicas. Algunos pacientes desarrollan el sndrome de choque por
dengue que es la forma ms severa de la enfermedad. Las manifestaciones hemorrgicas
pueden ser tan leves que solo se hacen evidentes a travs de la prueba del torniquete, o tan
severas como sangramientos digestivos graves con compromiso hemodinmico.
Los signos de extravasacin de lquidos (derrame en serosas, hemoconcentracin) son
indispensables para hablar de FHD/SCD, ya que es la fuga de lquido la que casi siempre
lleva el paciente al choque y no las hemorragias. Este hecho es de gran importancia pues
influenciado por el nombre de fiebre hemorrgica de dengue, en no pocas ocasiones los
mdicos asistentes esperan las grandes hemorragias que nunca llegan y el paciente cae en
choque por la extravasacin de plasma.

Fase de recuperacin: Se inicia cuando cesa el escape de lquido y las manifestaciones de


sangrado comienzan a disminuir. Pueden aparecer edemas o agravarse los derrames serosos
producto de la sobre hidratacin. Tambin se puede apreciar un rash tardo asociado a
prurito intenso. Se recuperan el apetito y el nmero de plaquetas.
Fase de convalecencia: Puede prolongarse hasta ms de 6 meses, se caracteriza por
cefalea discreta, cansancio fcil y artromialgias. No se presenta en la totalidad de los
casos.

Resumimos los signos de alarma de la siguiente manera:


1. Signos clnicos tempranos del choque.
Cada brusca de la fiebre.
Dolor abdominal intenso.
Irritabilidad, somnolencia u otras alteraciones mentales.
Fatiga extrema.
Lipotimias.
Dolor torcico.
2. Signos clnicos que agravan el choque.
Vmitos frecuentes.
Diarreas frecuentes.
3. Signos de laboratorio e imagenolgicos.

Aumento progresivo del hematcrito y


disminucin progresiva del conteo de plaquetas.
Engrosamiento de la pared de la vescula biliar.

Referencias:
1. Guzman MG, Kouri G. Dengue-an update. The Lancet Inf Dis 2002;2:33-42.
2. Gubler DJ, Clark CG. Dengue/dengue hemorrhagic fever: the emergence of a global
health problem. Emerging Infectious Diseases. Atlanta USA. 1995;1:55-57.
3. Kour GP, Guzmn MG, Bravo JR, Triana C. Dengue haemorrhagic fever/ dengue shock
syndrome: lessons from the Cuban epidemic. 1981. Bull World Health Organ 1989;87:37580.
4. Organizacin Panamericana de la Salud. 2003: Number of reported cases of dengue and
dengue hemorrhagic fever (DHF), region of the Americas (by country and subregion).
[citado del 25 de agosto de 2003].

5. Valdes L, Guzman MG, Kouri G, Delgado J, Carbonell I, Cabrera MV, Rosario D, Vazquez
S. La Epidemiologa del Dengue en Cuba en 1997. Rev Panam Salud Publica/Pan American
Journal of Public Health 1999;6:16-25.
6. Pelaez O et al. Havana dengue 3 epidemic, 2001. Enviada a Emerging Infection Diseases.
7. George R, Lum LCS. Clinical spectrum of dengue infection. In: Gubler DJ, Kuno G, eds.
Dengue and dengue hemorrhagic fever. London: CAB International, UK, 1997; 89-113.
8. Kour G, Guzmn MG, Bravo J. Why dengue haemorrhagic fever in Cuba? 2: An integral
analysis. Trans R Soc Trop Med Hyg 1987;81:821-23.

42.- Paciente de 4 aos de edad tiene lesiones eczematosas crnicas en flexuras de brazos
y piernas que producen intenso picor, asociadas a una queilitis descamativa de labios. Cul,
entre los siguientes, es el diagnstico ms probable?
a) Una dermatitis atpica.
b) Un eczema seborrico.
c) Un prrigo nodular.
d) Un eczema microbiano.

La dermatitis atpica llamada comnmente eccema (atpico), es una enfermedad que


consiste en un estado reaccional de la piel caracterizado por erupciones pruriginosas y con
aspecto de escamas, ms frecuente en nios, multifactorial, en la cual intervienen factores
tanto ambientales como constitucionales. Las personas con eccema a menudo tienen
antecedentes de condiciones alrgicas como asma, fiebre del heno o eccema. La dermatitis
atpica fue originalmente conocida como prrigo de Besnier y eccema constitucional,
actualmente tambin es llamada neurodermatitis diseminada, por las escuelas europeas.

Clnica
Las manifestaciones clnicas tpicas de la dermatitis atpica se dividen en tres etapas, que
suelen denominarse del lactante, infantil y del adulto. Junto a ellas se encuentran otras,
con frecuencia llamadas atpicas, a pesar de que muchas, como la xerosis, son muy
constantes.

Dermatitis atpica del lactante:


Suele empezar hacia los cinco meses de vida, pero puede hacerlo antes. Algunos nios
desarrollan lesiones de eccema seborreico, que de forma gradual va adquiriendo el aspecto
de la dermatitis o eccema atpico.
La localizacin ms habitual es en la cara, respetando las zonas alrededor de los ojos, la
nariz y la boca (. Son tambin frecuentes en el cuero cabelludo, las orejas, el dorso de las
manos y las zonas de extensin de las extremidades.
Las lesiones suelen ser ppulas o placas eritematosas y edematosas, muchas veces con
erosiones, exudacin y costras.
Es muy raro que se aprecien las vesculas caractersticas del eccema. El prurito es un
sntoma constante.

Dermatitis atpica infantil

Este periodo suele considerarse con un inicio hacia los dos aos y un final entre los siete
aos y la pubertad. Las lesiones caractersticas se observan sobre todo en las flexuras, en
especial en los codos y las rodillas (Fig. ), pero pueden aparecer en otras zonas.
En esta fase es ms fcil ver lesiones eccematosas con vesculas, pero el intenso prurito
hace que enseguida se transformen en erosiones, con exudacin y formacin de costras.

Bibliografa:

1. Bielsa Marsol I. Eccemas (II). En: Ferrndiz C, ed. Dermatologa Clnica. Madrid, Mosby/
Doyma Libros 1996, 113-124.
2. Fernndez Vozmediano JM y cols. Dermatitis atpica. Madrid, Jarpyo 1994.
3. Fernndez Vozmediano JM, Armario Hita JC. Tacrolimus. Piel 2001;16:48-54.
4. Fonseca Capdevila E. Dermatitis atpica. Protocolo teraputico. (En lnea) (14.02.2001).
Disponible en www.especialistasdermatologia.com.
5. Fonseca E. Dermatitis atpica en la infancia. Salud Rural 1997;14:92-105.
6. Guerra Tapia A. Dermatitis atpica. En: Fonseca Capdevila E, ed. Dermatologa
Peditrica.
Madrid, Aula Mdica 1999, 83-180.

43.- Un hombre de 55 aos acude a consulta por una historia de 2 meses de dificultad para
tragar. En un principio, la dificultad era nicamente con bocados grandes de alimentos
slidos, pero ahora tiene problema hasta con los lquidos. Tiene una historia de uso de
alcohol y tabaco. La esofagoscopa demuestra una masa polipoide grande e irregular que
ocluye casi completamente el tercio superior del esfago. Cul de los siguientes es el tipo
histolgico de este tumor?

a)
b)
c)
d)

Linfoma de clulas gigantes


Carcinoma de clulas pequeas
Linfoma de clulas pequeas
Carcinoma de clulas escamosas

Hay varios subtipos, principalmente adenocarcinoma (aproximadamente 50-80% de todos


los cnceres del esfago) y el cncer de clulas escamosas. El cncer de clulas escamosas
surge de las clulas que recubren la parte superior del esfago. El adenocarcinoma se
deriva de las clulas glandulares que estn presentes en la unin del esfago y el estmago.
Los tumores de esfago por lo general llevan a la disfagia (dificultad para tragar), dolor.

Clasificacin:

Los cnceres de esfago son generalmente carcinomas que surgen a partir del epitelio o
revestimiento de superficie, del esfago. La mayora de los cnceres del esfago caen en
una de dos clases: Los carcinomas de clulas escamosas, que son similares a cncer de
cabeza y cuello en su apariencia y su asociacin con el tabaco y el consumo de alcohol, y los
adenocarcinomas, que se asocian a menudo con antecedentes de enfermedad por reflujo
gastroesofgico y esfago de Barrett. Una regla del pulgar general es que un cncer en la
parte superior de dos tercios es un carcinoma de clulas escamosas y uno en el tercio
inferior es un adenocarcinoma.

REVISTA ESPAOLA DE PATOLOGA


Vol. 37, n. 4, 2004
Protocolo e informacin sistematizada para los estudios histopatolgicos relacionados con
el carcinoma esofgico
Francisco Colina, Guadalupe Lpez Alonso, Carolina Ibarrola.

44.- Masculino de 7 meses de edad que inicia con rechazo al alimento, irritabilidad, llanto
constante, inquietud, delirio, convulsiones y estado epilptico los datos clnicos del menor
son compatibles con intoxicacin por:
a) Capuln tullidor.
b) T de ans estrella.
c) Papaver somniferum
d) Picadura de alacrn.

La intoxicacin se presenta en forma crnica. Ms frecuente en recin nacidos y lactantes.


La sintomatologa por intoxicacin de t de ans de estrella se caracteriza por el siguiente
cuadro clnico.
Gran irritabilidad
Llanto constante
Inquietud
Delirio
Convulsiones
Rechazo al alimento
Estado epilptico
El tratamiento consiste en:

Estabilizar signos vitales


Ayuno
Soluciones parenterales
Anticonvulsivos:
Benzodiacepinas barbitricos
Estabilizar signos vitales
Ayuno
Soluciones parenterales
Anticonvulsivos:
Benzodiacepinas barbitricos
1.- Montoya-Cabrera MA. Intoxicaciones y envenenamientos en nios. Mxico,
Intersistemas, 2000.

2.-Montoya CMA. Toxicologa clnica. 2. Ed., Mxico, Mndez Editores.

45.- Su frecuencia es de 1/8,000 bajo peso al nacer, occipucio prominente, micrognatia,


puos cerrados con cabalgamiento caracterstico de los dedos, pies en mecedora y
Divertculo de Meckel:

a)
b)
c)
d)

Trisoma 13
Trisoma 18
Trisoma 8
Trisoma 11

TRISOMIA 18 (SINDROME DE EDWARD)


Tiene una incidencia de 1/8000 recin nacidos vivos con predominio para el sexo femenino.
Es la trisomia ms comn entre los recin nacidos muertos con malformaciones.
Manifestaciones Clnicas: En mas del 50% de los casos presenta: Retraso mental y
pondoestatural, hipertonia, occipucio prominente, orejas bajas y malformadas, micrognatia,
cuello corto, implantacin distal y retroflexin del pulgar, hipoplasia de uas, defectos
articulares de los dedos, dedos de manos montados, pie equinovaro, calcaneo prominente
(pie en mecedora), abduccin limitada de caderas, esternn corto, cardiopatia y
criptorquidea. En menos del 50% de los casos se acompaa de hipotonia, epicantus,

microftalmia, defectos oculares, fisura labial y palatina, hemangiomas capilares, polidactilia,


sindactilia, hernias y malformaciones renales.
Desde el punto de vista anestsico se debe predecir una difcil intubacin y tener especial
precaucin por las drogas con excrecin renal. El pronstico es muy grave ya que la
supervivencia es corta, por el conjunto de malformaciones viscerales graves asociadas ; el
50% fallecen antes de los dos meses, y el 75% antes de los tres meses. En las
supervivencias prolongadas se pone de manifiesto el intenso retraso psicomotor y los
defectos esquelticos posturales. En los casos de trisomas parciales por lo general la
clnica se atena y la supervivencia es prolongada.

BADGWELL J : Clinical pediatric anesthesia. Common and uncommon coexisting diseases.


Philadelphia. 399-401, 1997.
CRUZ M : Tratado de pediatria. Cromosopatias. Barcelona. 7a edicin, 305-314, 1994.
MENEGHELLO J : Pediatria. Fundamentos de genetica clinica en pediatria. Buenos aires, 4a
edicion, 2004-2005, 1991.
HARLEY EH : Neurologic sequelae secondary to atlantoaxial instability in down syndrome.
Arch otolaringology, 120(2), 159-165, 1994.

46.- Paciente femenino de 10 aos de edad que la llevan sus padres por conducta rara.
Actualmente se est recuperando de un cuadro diarreico. No presenta rigidez de nuca,
nusea, vmito u otra alteracin. Se presenta combativa, hiperreflxica, midritica,
mucosas secas y rubicunda. Signos vitales: FC 140 por min, FR 20 por min, TA 110/70
mmHg, Temp. 40C, peso 35 Kg. Esta paciente presenta un toxndrome, cual sera?

a)
b)
c)
d)

Intoxicacin por organofosforados.


Intoxicacin por hongos.
Intoxicacin por custicos.
Intoxicacin por anticolinrgicos.

ANTICOLINRGICOS
El grupo de frmacos de esta clase actan en el nivel uno es decir a nivel del rgano y son
los llamados frmacos anticolinrgicos o antimuscarnicos y a estos frmacos se los define
como aquellos que bloquean la accin muscarnica de la Acetilcolina (Ach) es decir son los
que inhiben la funcin del sistema parasimptico.

ANTICOLINRGICOS

Agitacin

Atropina

Alucinaciones

Antidepresivos tricclicos

Midriasis

Fenotiacidas

Boca Seca

Antihistamnicos

Taquicardia, arritmias

Hongos

Retencin urinaria
Calor
Piel seca
Movs. Extrapiramidales

47.-Un hombre de 29 aos con historia de actividad homosexual hace 5 aos y HIV-1
positivo con tratamiento con HAART , el cual hace 8 meses suspende el medicamento por
sentirse bien y que ha presentado hace dos meses una neumona por neumocictis carinii , en
caso de presentar una infeccin del SNC por papovavirus tiene un riesgo de producir :

a)
b)
c)
d)

Adrenoleucodistrofia.
Esclerosis mltiple.
Panencefalitis subaguda esclerosante. (SSPE)
Leucoencefalopatia multifocal progresiva. (PML)

La leucoencefalopata multifocal progresiva (LMP) es una enfermedad de etiologa viral


(virus JC, Papovavirus), que afecta de forma subaguda o crnica la sustancia enceflica
provocando una desmielinizacin progresiva de sta.
Se observa en pacientes jvenes, portadores de enfermedades inmunosupresoras
(infeccin por VIH etapas finales, enfermedades linfoproliferativas, tratamientos
inmunosupresores, etctera).

El diagnstico de LMP se plantea ante un paciente inmunodeprimido, que presenta un cuadro


de deterioro cognitivo o dficit focales (hemiparesia, alteraciones campimtricas,
alteraciones de la coordinacin y del equilibrio, etctera), o ambos, de curso progresivo, en
cuestin de semanas o pocos meses, infrecuentemente acompaado de cefaleas y sin
fiebre. Este cuadro lleva inexorablemente a la muerte, aunque se han reportado casos
anecdticos de detencin y reversibilidad de la enfermedad (7). La tomografa
computarizada (TC) enceflica puede ser normal al inicio de la enfermedad, o mostrar
lesiones hipodensas de la sustancia blanca, a menudo confluentes, ms frecuentemente
ubicadas en las regiones frontales y parietooccipitales que no se realzan con el contraste,
no producen efecto de masa y respetan la sustancia gris cortical.

Leucoencefalopata multifocal progresiva


Rev Med Uruguay 2003; 19: 78-82.

Bibliografa
1 . Corradi H. Leucoencefalopata multifocal progresiva. In: Salamano R (coord). Temas de
Neuroinfectologa. Montevideo: Oficina del Libro-AEM, 1998: 105-8.
2 . Astrom KE, Mancall EL, Richardson EP Jr. Progressive Multifocal Encephalopathy.
Brain 1958; 81: 930.

3 . Cavanagh JB, Greenbaum D, Marshall AHE, et al. Cerebral


Demyelination associated with disorders of reticuloendothelial system. Lancet 1959; 2:
525.
4 . Richardson EP Jr. Progressive Multifocal Leukoencephalopathy. N Engl J Med 1961;
265: 815.
5 . Padgett BL, Walker DL, Zu Rheim GM, Eckroade RJ, Dessel BH. Cultivation of
papova-like virus from human
brain with progressive multifocal leucoencephalopathy. Lancet 1971; 1: 1257-60.

48. Acude a consulta un hombre de 35 aos de edad, soltero, previamente sano. Sus
familiares refieren que en los 5 das previos ha presentado fiebre, cefalea, cambios del
estado de nimo, y en el ltimo da, somnolencia que se ha convertido en estupor. A la
exploracin fsica no encuentra alteraciones. Realiza una puncin lumbar y encuentra como
nica alteracin del lquido cerebroespinal la presencia de eritrocitos. Se solicit una
resonancia magntica de crneo. Usted esperara encontrar para apoyar su diagnstico:

a) Afeccin de ganglios basales


b) Afeccin de lbulo frontal

c) Afeccin de lbulo temporal


d) Lesin intraaxial ocupativa en puente.

El cuadro clnico y los hallazgos del lquido cerebroespinal sugieren fuertemente infeccin
por virus del herpes simple. Los cambios en el estudio de imagen ms caractersticos, sobre
todo en resonancia magntica, se observan en los lbulos temporales.
Kasper DL, Braunwald E, Fauci AS, Hauser SL, Longo DL, Jameson JL. Harrisons Principles
of Internal Medicine. McGraw Hill. 16 Ed. 2471-2490 p.

49.- Paciente femenino de 5 aos quien llega al servicio de urgencias por presentar fiebre,
cefalea, nusea, vmito y variabilidad en el estado de despierto. A la exploracin fsica
confirma la fiebre en 39C, FC 110x, y presenta lesiones purpricas en extremidades.
Usted sospecha meningitis por meningococo y hace una puncin lumbar. Cul de las
siguientes opciones sera compatible con su diagnstico:
a) Aspecto de agua de roca, glucorraqua normal y celularidad aumentada con predominio
de linfocitos.
b) Aspecto de agua de roca, glucorraqua normal, celularidad aumentada con predominio de
linfocitos y algunas clulas de aspecto neoplsico.
c) Aspecto turbio, glucorraqua normal y celularidad aumentada con predominio de
polimorfonucleares.
d) Aspecto turbio, glucorraqua disminuda y celularidad aumentada con predominio de
polimorfonucleares.

El cuadro clnico es compatible con una meningitis bacteriana. La primera opcin es ms


compatible con meningitis asptica, de origen viral, la segunda con una meningitis
neoplsica, la tercera son poco probables encontrarlas en la realidad.
Kasper DL, Braunwald E, Fauci AS, Hauser SL, Longo DL, Jameson JL. Harrisons Principles
of Internal Medicine. McGraw Hill. 16 Ed. 2471-2490 p.

50.- Durante su servicio social le llevan en la noche a un paciente de 3 das de nacido, cuyo
parto fue atendido en el domicilio de su madre por una partera emprica. El nio ha
rechazado la alimentacin al seno materno en las ltimas 24 horas, y se ha mostrado
hipoactivo. Al revisarlo encuentra que presenta la fontanela con tensin aumentada y
fiebre. Se le pasa por la mente el diagnstico de meningitis. Uno de los microorganismos
ms probables es:
a)
b)
c)
d)

Meningococo.
Neumococo.
Estreptococo del grupo B (S agalactiae)
Listeria monocytogenes

Los microorganismos ms frecuentes en este grupo de edad son el estreptococo del grupo
B y la Escherichia coli, con los que se colonizan algunos recin nacidos por su paso a travs
del canal del parto. Listeria aparece en pacientes de mayor edad, lo mismo que meningo y
neumococo.
Kasper DL, Braunwald E, Fauci AS, Hauser SL, Longo DL, Jameson JL. Harrisons Principles
of Internal Medicine. McGraw Hill. 16 Ed. 2471-2490 p.

51.- Femenino de 62 aos refiere hace unos das dolor brusco en espalda tras un intento
de levantar un bulto de aproximados 15 kg. Menopausia a los 50 aos. Fumadora.
Exploracin fsica normal, salvo dolor a la percusin sobre apfisis espinosas de D5-D6.
Calcemia en lmite normal alto; fsforo, fosfatasa alcalina y niveles de vitamina D normales.
Urea y creatinina en lmite superior de la normalidad. Hb 11 g/dl. Protenas totales 9 g/dl
con albmina 3,8 g/dl. VSG de 90 en primera hora. Proteinuria de 1,2 g (24 horas).
Radiologa lateral de columna dorsal: acuamiento posterior de D6, sugerente de
osteoporosis generalizada. La paciente tendr ms probablemente:
a)
b)
c)
d)

Osteoporosis postmenopusica.
Osteomalacia.
Mieloma mltiple.
Hiperparatiroidismo.

Definicin

OMS:

Tumor maligno, que habitualmente muestra compromiso seo difuso o mltiple, y que se
caracteriza por la presencia de clulas redondas del tipo de las clulas plasmticas pero
con
diversos
grados
de
inmadurez,
incluyendo
formas
atpicas.
Presentacin clnica:
Ms frecuente en los hombres 2:1. El 75% de los casos entre 50 y 70 aos de edad.
Se localiza, preferentemente, en los huesos donde existe mdula sea roja: vrtebras,
costillas, esternn, pelvis, crneo y huesos largos (tercio proximal fmur y hmero).
Aproximadamente el 80% de pacientes tienen como queja principal el dolor seo con
sensibilidad difusa de hueso, particularmente sobre el esternn, pelvis y columna vertebral.
Posteriormente pueden aparecer sntomas radiculares.
La sensacin de tumor es menos frecuente, ya que a veces los ndulos mielomatosos son
pequeos o asientan en huesos profundos, a la palpacin suele ser de dos a tres cm. de
dimetro
El crecimiento de clulas plasmticas interfiere con la produccin de glbulos rojos,
glbulos blancos y plaquetas, por lo que se desarrolla anemia, susceptibilidad a infecciones
y tendencia al sangrado.
Tambin pueden parecer sntomas constitucionales, fiebre, anemia, trombocitopenia, y
fracaso
renal.
Las fracturas patolgicas de columna o fmur pueden ser el primer sntoma. Se producen
con
un
traumatismo
mnimo
o
incluso
sin
trauma.
El rango de duracin de los sntomas es tan corto como pocas semanas hasta ms de 2 aos

Mieloma Mltiple
Mieloma mltiple en estadio I
En estadio I se presenta con:
1. Estructura sea normal.
2. Calcio srico normal.
3. Hemoglobina superior a 10 g/dL.
4. Produccin baja de protena M indicada por:
a. IgG inferior a 5.0 g/dL
b. IgA inferior a 3.0 g/dL
c. kappa o lambda de la orina
inferior a 4 g/24 horas.
Masa estimada de las clulas del mieloma: inferior a 0.6 billones de clulas por metro
cuadrado (carga baja)
Se usa la siguiente subclasificacin de estadios:
A. creatinina inferior a 2.0 mg/dL
B. creatinina superior o igual a 2.0 mg/dL
La insuficiencia renal empeora el pronstico independientemente del estadio.
Mieloma mltiple en estadio II
En este estadio II el mieloma mltiple se presenta con:
Masa estimada de clulas del mieloma: 0.6 a 1.2 billones de clulas por metro
cuadrado (carga intermedia)
Se usa la siguiente subclasificacin de estadios:
A. creatinina inferior a 2.0 mg/dL
B. creatinina superior o igual a 2.0 mg/dL
La insuficiencia renal empeora el pronstico independientemente del estadio.
Mieloma mltiple en estadio III

En estadio III significa que existen uno o ms de los siguientes:


1. Hemoglobina inferior a 8.5 g/dL
2. Calcio srico superior a 12.0 mg/dL
3. Ms de 3 lesiones osteolticas.

4. Produccin alta de protena M indicada por:


a. IgG superior a 7.0 g/dL
b. IgA superior a 5.0 g/dL
c. kappa o lambda de la orina
superior a 12.0 g/24 horas
Masa estimada de clulas del mieloma: mayor de 1.2 billones de clulas por metro
cuadrado (carga alta)
Se usa la siguiente subclasificacin de estadios:
A. creatinina inferior a 2.0 mg/dL
B. creatinina superior o igual a 2.0 mg/dL
La microglobulina beta-2 srica ha mostrado ser un indicador confiable para el pronstico.
Puesto que la gran mayora de pacientes con mieloma sintomticos son clasificados como
estadio III por los criterios de Durie/Salmon, este sistema de clasificacin no ha probado
ser muy til para identificar a los pacientes con pronstico precario e intermedio. Muchos
investigadores estn a favor de un sistema ms simple que utilice solamente las
concentraciones de microglobulina beta-2 y albmina para determinar la estadio de los
pacientes. La insuficiencia renal empeora el pronstico independientemente de la estadio.
Las anormalidades de los cromosomas 13 y 11q y la morfologa plasmablstica parecen
haber pronosticado un resultado precario en una poblacin de pacientes cuyo mieloma fue
tratado con quimioterapia de dosis elevadas y rescate con clulas madre.

De National Cancer Institute


Solitary plasmacytoma of bone and extramedullary plasmacytomas: A clinicopathologic and
immunohistochemical study.
JM Meis et al. Cancer. Vol 59. 1987. p 1475-1485;

52.- Una mujer de 54 aos de edad es llevada a urgencias despus de un accidente


automovilstico. Al llegar, se encuentra respirando adecuadamente. Presenta miltiples
abrasiones en el trax y mltiples sitios de dolor a la palpacin sobre las costillas. LA RX
muestra mltiples fracturas costales, pero el parnquima se observa sin alteraciones y
ambos pulmones se encuentran expandidos. Dos das despus, presenta dificultad
respiratoria y sus pulmones se emblanquecen en la RX. Cul de los siguientes es el
diagnstico ms probable?

a)
b)
c)
d)

Contusin miocrdica
Contusin pulmonar
Ruptura traumtica de la aorta
Hemotrax

Diagnstico
Generalmente se lo hace con base en los datos clnicos, el conocimiento del mecanismo de la
lesin y la confirmacin mediante la radiografa de trax y la Tc.
Datos clnicos.- No existen signos fsicos especficos de contusin, los datos ms tpicos
son taquipnea, si la contusin es severa y est presente un trax inestable, o si hay lesiones
asociadas como neumotrax, el distrs respiratorio puede ser extremo. La cianosis es rara
excepto en casos de lesin de todo el trax cuando una decoloracin ciantica de cabeza y
cuello frecuentemente en asociacin con petequias es caracterstico.3
Radiologa.- La contusin pulmonar se muestra como un infiltrado pulmonar de densidad
variable con bordes pobremente definidos en el rea de mximo impacto. En el trauma
contuso la densidad es perifrica y tiende a disminuir hacia el hilio. (Fig. 4 Y 5) Estas
lesiones tpicamente aparecen 4 a 6 horas despus del trauma, clsicamente empeoran en
aproximadamente 24 horas y luego tienden a resolverse desapareciendo en 3 a 4 das,
aunque la cicatrizacin posiblemente se complete a los 7 a 10 das.I-3-?
Existe una interrelacin entre la extensin de las anormalidades vistas en la radiografa de
trax y la severidad de los datos clnicos. Cuando sobreviene neumona, frecuentemente
esta se desarrolla en el segmento que estuvo contuso y se superpone en el sitio lesionado
de forma tal que el infiltrado nunca se aclara totalmente en la placa radiogrfica. (Fig. 6)
Las fracturas costales generalmente estn presentes alrededor del rea de contusin
sealando el rea de mximo impacto.
En el trauma penetrante la contusin corresponde a un cilindro de tejido lesionado
alrededor del tracto de la herida o del tejido directamente lesionado cuando hay mltiples
fragmentos. Estas imgenes pueden estar ligeramente alteradas debido a hemorragia
dentro del trayecto de la herida.
Hemoneumotrax universalmente est presente, en contraste con el trauma contuso en
donde a menudo est ausente.
Tanto en el trauma penetrante o contuso las lesiones no tienen bordes anatmicos
definidos, en caso de presentarse, es probable que la lesin no corresponda precisamente a
una contusin.

Fig. 5. Contusin pulmonar. Imagen obtenida de la misma paciente luego de 8h del trauma.

Ntese las fracturas costales alIado izquierdo y la imagen de condensacin compatible con
contusin pulmonar.
Bibliografa
l. Cohn SM. Contusion pulmonary. Review of the Clinical Entity. J Trauma 1997; 42:973979.
2. Velmahos GC. Traumatismos contusos de trax. En: Naude GP, Bongar
FS, Demetriades D. Secretos del traumatismo. Editorial McGraw-Hill Interamericana.
Mxico DF-Mxico 1999:97-101.
3. Lewis FR. Pulmonary Co~tusion. In: Callaham MI.. Current Therapy in Emergency
Medicine. BC. Decker Inc. Toronto - Philadelphia. 1987: 123-126.
4. Thomson FG. Notes on penetrating chest wounds. BMJ 1940; 1 :44.
5. Wintermark M., Schnyder P. The Macklin Effect. A frequent Etiology
forPneumomediastinum in Severe Blunt Chest Trauma. Chest 2001; 120:543-547.
6. Hooker DR. Physiological effects of air concussion. Am J Physiol 1924; 67:219
7. Moseley RV, Vemick JJ, Doty DE. Response to blunt chest injury: a new experimental
model. J Trauma 1970; 10:673.

53.- A un lactante de 8 meses le fue administrada metoclopramida en forma repetida por


haber presentado vmitos es llevado al servicio de urgencias porque presenta irritabilidad,
espasticidad y opistotonos, es este momento se le debe de administrar:
a) Bicarbonato de Sodio
b) Difenhidramina
c) Atropina
d) Naloxona
e) Diazepam

METOCLOPRAMIDA: Es un medicamento con acciones colinrgicas y antidopaminrgicas. A


nivel gastrointestinal, la metoclopramida incrementa el tono del esfnter esofgico inferior,
mejora el tono gstrico, incrementa los movimientos peristlticos, mejora la coordinacin
antro-duodenal y relaja el esfnter pilrico. Gracias a estos efectos farmacolgicos, la
metoclopramida acelera el vaciamiento gstrico y disminuye el tiempo de trnsito intestinal
a travs del duodeno, yeyuno e leon. Por otra parte, la metoclopramida es un frmaco que
cruza la barrera hemato-enceflica y por consiguiente tiene acciones a nivel del sistema
nervioso central, principalmente como antagonista a nivel de los receptores dopaminrgicos
ATROPINA (ATROPINA)
Se utiliza en la intoxicacin por agentes colinrgicos, los cuales actan por inhibicin de la
acetilcolinesterasa. El acmulo de acetilcolina resultante provoca diversos efectos que
constituyen el sndrome colinrgico: muscarnico, nicotnico y central.
Existen diversos agentes colinrgicos, entre los que se incluyen: insecticidas tipo
carbamatos y organofosforados (OP) y agentes nerviosos OP.
Mientras que los carbamatos se unen de forma reversible a la acetilcolinesterasa, la unin
de los organofosforados se hace irreversible con el tiempo.
La atropina, antagonista competitivo de la acetilcolina, revierte los efectos muscarnicos
(salivacin, lagrimeo, incontinencia urinaria, diarrea, sudoracin, vmitos, broncorrea,

broncoconstriccin, bradicardia y miosis); sin embargo, no contrarresta los efectos


nicotnicos (debilidad muscular, fasciculaciones, parlisis, hipertensin arterial, taquicardia
y midriasis), ni los centrales (delirio, convulsiones, coma). La sintomatologa nicotnica de los
OP se trata con pralidoxima (oxima), que reactiva la acetilcolinesterasa. En la intoxicacin
por carbamatos, la pralidoxima no est indicada, ya que la enzima se regenera de forma
rpida y espontnea.
Antdotos ms utilizados en intoxicaciones peditricas 55

Indicaciones:

1. Intoxicaciones sintomticas por insecticidas y herbicidas (OP o carbamatos).


2. Intoxicaciones por OP incluidos en agentes nerviosos usados en guerras qumicas (sarin,
soman, tuban, VX).
3. Intoxicaciones por sustancias colinrgicas: setas, fisostigmina, metacolina,
neostigmina, pilocarpina.
4. Conduccin auriculoventricular alterada por: digital, beta-bloqueantes, antagonistas del
calcio.
Dosis iv/im: 0,05-0,1 mg/kg/dosis (mn. 0,1 mg, mx. 5 mg), seguida de dosis repetidas
cada 2-10 minutos o en infusin continua (0,025 mg/kg/h), hasta que aparezcan los signos
de atropinizacin: desaparicin de la broncorrea y del broncoespasmo, taquicardia, piel seca
y rubicunda.
La midriasis no es un signo confiable de atropinizacin, ya que su aparicin no es constante.
Pueden necesitarse varias horas de tratamiento.
Se utilizan dosis ms elevadas en las intoxicaciones ms graves.
Efectos secundarios: midriasis, taquicardia, fiebre, visin borrosa, sequedad de boca, leo,
retencin urinaria. Adems, si se administra va iv, puede provocar arritmias en el paciente
hipxico, por lo que se aconseja corregir la hipoxia antes de su administracin o si no es
posible, administrarla va im.
El tratamiento de la intoxicacin por agentes colinrgicos se basa en la administracin de
atropina (para proteger los receptores muscarnicos), oximas (para acelerar la reactivacin
de la AChE inhibida, en la intoxicacin por OP) y benzodiazepinas (para evitar convulsiones).
La administracin de fosfotriesterasas (PTE, enzimas implicadas en la detoxificacin de OP
por hidrlisis) ha demostrado ser un tratamiento muy eficaz frente a intoxicaciones por
insecticidas OP y agentes nerviosos de guerra OP.

54.- A la probabilidad de que una persona que en realidad padece la enfermedad de inters
tenga un resultado positivo en su prueba diagnstica se le conoce como:

a)
b)
c)
d)

Sensibilidad
Especificidad
Valor predictivo positivo
Valor predictivo negativo

La sensibilidad se define como la capacidad que tiene una prueba diagnstica para clasificar
correctamente al enfermo como enfermo o como la probabilidad de tener un resultado
positivo dado que se est enfermo.

Ruiz M. A. Epidemiologa Clnica, Panamericana, 1. Ed. 2004; pg: 116

55.-El diagnstico mas probable en un paciente femenino de 23 aos que presenta en una
radiografa con datos de tumoracin multiloculada en la zona epifisiaria del extremo distal
de fmur izquierdo, es el siguiente:
a) Sarcoma osteognico.
b) Tumor de clulas gigantes.
c) Sarcoma de Ewing.
d) Mieloma mltiple

INTRODUCCIN:
El tumor de clulas gigantes es un tumor raro, constituye el 5% de los tumores seos
primarios (1). Ocurre en pacientes entre los 20 y 40 aos, siendo raro en los menores de 10
y en mayores de 50 aos. Tiene un ligero predominio del sexo femenino y es ms frecuente
en pases orientales que occidentales (2).
Habitualmente se localizan en huesos largos, con afectacin de la epfisis y de localizacin
excntrica, pueden propagarse a metfisis, provocar destruccin cortical y extensin
eventual a tejidos blandos y al espacio articular. Los tres sitios ms habituales de
localizacin (3), por orden de frecuencia, son:
1 Extremo distal del fmur
2 Extremo proximal de tibia
3 Extremo distal del radio.
Clnicamente, el dolor es el signo ms frecuente de presentacin, tanto si se asocia o no a
fractura patolgica (4).
Localmente la piel puede estar hiperrmica, eritematosa y si el tumor crece puede
aparecer circulacin colateral con presencia de masa palpable. La clnica sistmica es
infrecuente y si el TCG tiene localizacin raqudea o sacra presenta signos y sntomas
neurolgicos (5).
El diagnstico del TCG suele realizarse por las manifestaciones clnicas y radiogrficas
(tabla I), teniendo siempre en cuenta los posibles diagnsticos diferenciales (tabla II).
Como pruebas complementarias, la RMN es el mtodo ms provechoso para determinar la
extensin y el estadiaje, la Gammagrafa es utilizada para detectar TCG multicntricos que
son raros (< 1%) (6) y los parmetros de laboratorio suelen ser normales.

Tabla I: Caractersticas radiolgicas del


TCG
Lesin osteoltica, excntrica y
epifisaria
Bordes generalmente bien definidos pero no esclerosos
Afectacin parcial metafisaria
Expansin a hueso subcondral, adelgazamiento y abombamiento cortical (ruptura) y
expansin a tejidos blandos
Mnima o ninguna trabeculacin

Pals, S.D., and Wilkins, R.M.:Giant cell tumor of bone treated by curettage, cementation,
and bone grafting. Orthopedics.
1992. 15: 703-708

56.-Femenino de 22 aos, que presenta una tumoracin de 2 cm de dimetro en el


cuadrante nfero-externo de la mama izquierda, indolora, de consistencia firme, superficie
lisa, forma ovoidea, mvil y bien delimitada del parnquima vecino, sin antecedentes de
derrame por el pezn, sin piel de naranja ni retraccin del pezn, cul sera su
diagnstico de presuncin?:

a)
b)
c)
d)

Fibroadenoma.
Carcinoma.
Ectasia de los conductos mamarios.
Quiste solitario.

FIBROADENOMA MAMARIO
Tumor benigno ms frecuente en las mujeres entre los 20 y 35 aos.
ETIOLOGIA
Existen mltiples teoras siendo la ms aceptada la hormonal, generalmente son nicos, solo
el 20% son mltiples o bilaterales. De tamao variable hasta de 10 cm. Ocupa el 13.6% de
la patologa mamaria benigna.
CUADRO CLNICO
Lesin nodular de consistencia dura, de larga evolucin y no dolorosa. Normalmente llegan a
los 3 cm. De dimetro. Durante la fase tarda del ciclo menstrual el tumor suele presentar
un leve aumento de tamao. Durante la menopausia presentan regresin hasta la
calcificacin (signo de palomitas de maz).
DIAGNOSTICO

Es clnico, se presenta como un tumor bien delimitado, desplazable, no adherido a piel ni a


planos profundos, liso o multilobulado en ocasiones. Se localiza frecuentemente en
cuadrantes externos.
EXAMENES DIAGNOSTICOS:
ULTRASONIDO MAMARIO .- Identifica un ndulo slido, bien delimitado de bordes
regulares .
TRATAMIENTO:
Conservador con vigilancia estrecha dependiendo del tamao y en caso de ser necesario
exresis del ndulo para estudio histopatolgico

hospitalgeneral.salud.gob.mx
BIBLIOGRAFIA
1. Snchez BC. Tratado de Enfermedades de la glndula mamaria. Ed. Manual Moderno. Cap.
13- 15.
2.- De Vita V. Cancer of the Breast. In Cancer: Principles and Practice of Oncology: Fifth
Ed. Philadelphia: Lippincott-Raven, Chapter 36; pp: 1521-1616.
3.-Consenso Nacional Acerca del Tratamiento de Cncer de Mama. En Tumores de mama:
Diagnstico y Tratamiento. 2 Ed. McGraw-Hill Interamericana; pp: 119-126.
4.-Eberlein T. Current management of carcinoma of the breast. Ann Surgery 1994; 220:
121-136.
5. Encyclopedie Medico. Chirurgicale Praxis Medica, Editions Techiques de Mxico, tomo 5,
ao 2005.

57.- Se presenta a consulta paciente de 25 aos de edad refiere que desde hace varios
das ha presentado flujo vaginal cuyas caractersticas son: blanquecino, grumoso, sin mal
olor, muy irritante El agente causal ms probable en esta patologa es?
a) Gardnerella
b) Tricomonas
c) Cndida albicans
d) Gonococo

Leucorreas micticas: Los hongos dan un flujo abundante, blanco, con grumos (aspecto como
de quesillo cortado) sin mal olor y que es muy irritante de la piel de la regin genital y por
lo tanto genera gran ardor vaginal y prurito (picazn). Son muy frecuentes y por lo general
son producidas por el hongo Candida albicans.

Tabla I. Caractersticas del flujo vaginal segn la causa


Cantidad

Consistencia

Olor

Blancoamarillento

Grumosa

Indiferente

Tricomonas Aumentada

Amarilloverdoso

Espumosa

Maloliente

Vaginosis

Blancogrisceo

Homogneoadherente

Maloliente

Candidiasis

Escasamoderada

Color

Moderada

Speroff Leon and Fritz Marc A. Clinical Gynecologic endocrinology and infertility. 7 ed.
Philadelphia: Lippincott Williams and Wilkins, 2005. p. 25 44.

58.- Se trata de masculino de 39 aos que sufre colisin automovilstica de frente, poli
contundido y con fx expuesta de fmur llama la atencin una equimosis periorbitaria por
el trauma facial, este signo traduce fractura de:

a)
b)
c)
d)

Piso posterior
Piso medio
Piso anterior
Macizo facial

En la valoracin ocular se deben tener en cuenta las heridas de los tejidos blandos de
prpado, crnea y conjuntiva. El signo de mapache (equimosis periorbitaria bilateral) se
encuentra frecuentemente asociado a las fracturas de la base anterior del crneo.
(Figura 3).

Figura 3. Equimosis periorbitaria luego de trauma facial contundente.


En las heridas del prpado la localizacin es de vital importancia y es necesario anotar
cuidadosamente el sitio, si se compromete el borde libre o si se lesiona el lugar de paso
de la va lagrimal.
El tono ocular ayudar a definir si hubo herida abierta del ojo.
La presencia de enoftalmo (hundimiento del globo ocular) alertar sobre la posibilidad de
una herida abierta del ojo o una fractura de las paredes orbitarias. El hipoftalmos
(descenso del ojo en el eje vertical) puede estar relacionado con fractura del piso de la

rbita o con fractura en el sitio donde se insertan los ligamentos suspensorios del globo
ocular (tubrculo de Whitnall, ligamento de Lookwood).
H. Ric Harnsberger. 2004. ISBN 848174753x Translation of PocketRadiologist Head & Neck: Top 100 Diagnoses.

59.- Un varn de 42 aos con antecedente de trastornos convulsivos experimenta una crisis
de gran mal. Sus pruebas de laboratorio, tomadas poco despus, revelan lo siguiente: Sodio
srico a 140 meq/L; potasio srico, 4.1 meq/L; cloruro srico, 97 meq/L; bicarbonato
plasmtico (HCO3-), 16 meq/L; pH arterial, 7.15 y Paco2, 46 mmHg. Cul de los siguientes
describe mejor la alteracin acidobsica?
a)
b)
c)
d)

Acidosis respiratoria
Acidosis metablica ms acidosis respiratoria
Acidosis respiratoria ms alcalosis respiratoria
Alcalosis mixta

Allen R. M. MMS Medicina Interna. 5. Edicin. National Medical Series. Mc. Graw Hill.
2006. (captulo 6, parte II: IV B-D). El paciente tiene acidemia importante, (bajo pH
arterial), relacionada con bajo Hco3- srico; por tanto, debe haber acidosis metablica.
Este trastorno se debe a acumulacin de lactato debida a actividad convulsiva. Otro dato
es la gran brecha aninica. Adems, el paciente tambin tiene acidosis respiratoria, como
se manifiesta por PaCO2 alta. Con frecuencia, la hiperventilacin acompaa a convulsiones
de gran mal.

60.- Los anti inflamatorios no esteroideos son frmacos muy utilizados, se absorben
fcilmente en el tracto gastrointestinal, una consecuencia idiosincrsica importante que
puede ocurrir an en pacientes previamente sanos, que es reversible si se detecta a tiempo
y que debe considerarse con ms frecuencia en el caso del ibuprofeno es:

a) Elevacin de AST y ALT.


b) Cefalea.
c) Vitiligo.

d) Nefrtis intersticial aguda.

Ruiz A, lvaro-Gracia J. Ma. Manual S.E.R. de las enfermedades reumticas. Sociedad


Espaola de Reumatologa. Editorial Panamericana 2006. Pgs. 183 188. Los anti
inflamatorios no esteroideos poseen vida media y potencia distinta, esas caractersticas
son expresiones de cada una de las formas moleculares, aquellos de vida media corta actan
menos de seis horas, los de mayor tiempo de accin superan esta cifra. Entre las
reacciones adversas ms frecuentes se encuentran las gastrointestinales, seguidas por
aquellas en hgado, rin, piel y sistema nervioso central. La nefritis intersticial aguda
acompaada de proteinuria masiva corresponde a una reaccin idiosincrsica que es ms
frecuente con el uso de fenoprofeno.

61.-Un varn de 30 aos presenta dolor y tumefaccin del testculo derecho. Su mdico
solicita un ultrasonido que revela una masa testicular de 2 x 2.5 cm. Se realizan una
exploracin inguinal y una orquiectoma. El estudio histopatolgico revel un seminoma puro.
Una tomografa computadorizada de trax, abdomen y pelvis mostr dos ganglios
retroperitoneales de 3cm que estn aumentados de tamao. La biometra hemtica, la
qumica sangunea y los marcadores tumorales estn todos dentro de los lmites normales.
Cul de los siguientes sera la mejor conducta?

a) Extirpacin quirrgica de toda la enfermedad.


b) Radioterapia.
c) Quimioterapia.
d) Observacin.

Allen R. M. MMS Medicina Interna. 5. Edicin. National Medical Series. Mc. Graw Hill.
2006. (captulo 4IX G 2). El cncer testicular es el ms comn en varones adultos jvenes
variedades ms frecuentes son seminomas y tumores de clulas germinales no
seminomatosos; ambos sor rabies aun en etapas avanzadas. Los seminomas son muy
sensibles a la radioterapia; por tanto, los pacientes enfermedad de etapa II (la limitada a
testculo y a ganglios por abajo del diafragma) pueden tratarse con < bajas de radiacin.
Debido a la toxicidad de la mdula sea producida por la radioterapia mediastnica, y est
indicada la radiacin profilctica del mediastino. En este contexto, los ganglios linfticos
retroperitoneales aumentados de tamao denotan la existencia de enfermedad metastsica
y est indicado el tratamiento.

62.- En un paciente del sexo masculino de 35 aos de edad con una dermatosis diseminada a
codos y rodillas con placas eritemato-escamosas Cul de los siguientes diagnsticos es el
de mayor probabilidad?
a) Dermatitis atpica
b) Dermatitis seborreica
c) Psoriasis
d) Dermatitis de contacto

La psoriasis es una enfermedad crnica que evoluciona en brotes de causa desconocida, que
se caracteriza por placas eritemato escamosas en diferentes partes de la piel. Se presenta
por igual en hombres y en mujeres, en todas las edades, predominando en jvenes y ms
frecuentemente en personas de piel blanca (parece que la presencia de melanina protege
contra la enfermedad.
Tambin es ampliamente reconocido que la psoriasis es una enfermedad familiar y
hereditaria (ditesis psorisica).
Los sitios de predileccin para que aparezcan las lesiones, son los salientes seos como
codos, rodillas, y la piel cabelluda as como la regin sacrocoxgea.

Christophers E, Krueger G G. Psoriasis. En: Fitzpatrick TB, EISEN AZ, Wolff K.


Dermatologa en Medicina General Buenos Aires: Editorial Panamericana 1988. P. 585-591
Christophers E, Schubert C, Schrder J M. Psoriasis. Dermatologa. 1992; 45.

63.- El encontrar elevacin de a-fetoprotena en sangre de una madre que cursa con
aproximadas 14 semanas de gestacin, nos hace pensar en:

a)
b)
c)
d)

El feto padece un retraso del crecimiento.


El feto es portador de una trisoma 21.
El feto tiene una hernia diafragmtica.
El feto tiene un defecto del cierre del tubo neural.

Los Defectos de Cierre del Tubo Neural (DTN) son problemas congnitos severos en el
cierre del tejido nervioso, del que se forma el cerebro y el cordn espinal. La Anencefalia
es el caso ms extremo de los DTN. La bveda craneana est ausente y slo queda la
porcin basal de los huesos frontal parietal y occipital. Su defecto mximo lo constituye la
Craneorraquisquisis.

El Encefalocele es la hernia del tejido cerebral a travs de un defecto en el crneo (3),


constituyendo el segundo tipo y la Espina Bfida es el DTN menos severo. Consiste en la
existencia de uno o ms arcos vertebrales defectuosos a travs del cual protruye meninges
(Meningocele) o meninges, mdula espinal y nervios (Mielomeningocele). El grado ms leve lo
constituye la Espina Bfida Oculta.

Estas afecciones parecen ser de origen multifactorial. La incidencia vara segn la


localizacin geogrfica, edad materna, caractersticas raciales y tnicas, sexo y el nivel
socioeconmico, adems de la hipertermia y los factores nutricionales. En Estados Unidos
de Amrica, la frecuencia es de 1 por cada 1000 recin nacidos vivos aproximadamente.

Diagnstico prenatal
1.

La elevacin de alfafetoprotenas, tanto sricas como en LCR, es un indicador


precoz.
2. Ecografa, permite planificar conducta y ofrecer la mejor opcin en resguardo de la
vida del binomio madre hijo; en aquellos pacientes que presenten mielomeningocele
no sindromtico, se contacta con los neurocirujanos previo a interrupcin del
embarazo que en este caso se planificar por va alta.
3. Estudio citogentico, en algunos pacientes el DTN corresponde a manifestaciones
de alteraciones cromosmicas, como trisoma 18, que deben ser evidenciadas en el
perodo prenatal y evitar acciones innecesarias al momento del nacimiento; es
fundamental adems para las parejas estar informados del pronstico real.
Referencias
1.

Jones K L , 1997 Smiths recognizable patterns of human malformation , 5th


edition Saunders Philadelpia.
2. Emerys Mueller RF Gentica Mdica 10 Ed Marban S L 2001. Madrid, Espaa
Gentica y anomalas congnitas pg. 223 - 243.
3. Plaguicidas en Chile La Guerra Qumica y sus vctimas Ma. Elena Rosas 1995.
Whittle MJ, Connors JM Prenatal diagnosis in obstetric practice. Blackwell Scientific
Publications, Oxford,1989.

64.-Masculino de 67 aos con cncer de prstata, que presenta hemorragia gingival,


hematomas, no hay pulsos distales, acrocianosis y datos de isquemia miocrdica, por lo que
usted sospecha:
a) Prpura No Trombocitopnica
b) Coagulacin Intravascular Diseminada
c) Prpura Trombocitopnica
d) Hemofilia

La coagulacin intravascular diseminada consiste en una activacin excesiva de la


coagulacin, lo que ocasiona trombosis. Como consecuencia el consumo progresivo de los
factores de coagulacin y plaquetas en las fases finales, produce el fenmeno opuesto, con
hemorragias generalizadas. Las causas son: infecciones sobre todo sepsis por gram
negativos, problemas obsttricos (abruptio, retencin de feto muerto, embolismo de lquido
amnitico, aborto sptico, toxemia) neoplasias (leucemias promielocticas, cncer de
pncreas, prstata, pulmn y esfago), fenmenos autoinmunes y traumas masivos. En la
CID aguda debida a complicaciones obsttricas o traumatismo predominan la ditesis
hemorrgica, mientras que en la CID crnica se manifiesta inicialmente por complicaciones
trombticas.
1.

Cotran R, Kumar V. Robbins Patologa estructural y funcional. 6 edicin. Mc Graw


Hill. Colombia 2003673.

65.-La triada de Beck realizada para el diagnstico de un tamponade cardiaco en un


paciente femenino de 44 aos consiste en:

a)
b)
c)
d)

Hipotensin, PVC elevada, RsCs disminuidos o velados


Hipotensin, PVC elevada, disminucin del tamao del complejo.
Hipotensin, PVC elevada, ingurgitacin yugular.
Hipotensin, PVC elevada, taquicardia.

El taponamiento cardaco se define como la compresin del corazn que resulta de la


acumulacin de lquido en el saco pericrdico y que produce un severo trastorno
hemodinmico. El efecto principal de la compresin cardaca es una alteracin en el llenado
de las cavidades durante la distole, lo cual lleva a una disminucin del gasto cardaco y de

la presin arterial en un espectro variable en el que el paciente severamente comprometido


puede estar en shock cardiognico.

Signos Agudos:-Triada de Beck: Presin arterial baja, elevacin de la presin venosa


central, Ruidos cardiacos apagados (corazn quieto)-La injurgitacinyugular puede estar
ausente debido a hipovolemia.-Paciente estuporoso, agitado-Extremidades fras y hmedasTaquipnea y taquicardia.

1.

Cheitlin MD, Abbott JA. Urgencias cardacas. En: Diagnstico y Tratamiento de


Urgencias. Editado por MT. Ho y CE Saunders. Editotial El Manual Moderno.
Mxico DF, 1991
2. Hancock EW. Cardiac Tamponade.
3. Med Clin North Am 63:1, 1979
4. Herrera Mrquez JH. Pericarditis. En: Cardiologa Fundamentos de Medicina. H
Vlez, J Borrero, J Restrepo editores. Corporacin para Investigaciones Biolgicas.
CIB. Medelln, 1981.

66.- Un varn de 71 aos presenta gota de inicio agudo. Este problema ha recurrido
durante varios aos y por lo comn se manifiesta como artritis monoarticular aguda que
afecta la primera articulacin intertarsal proximal metatarsiana. Tambin tiene
antecedentes de larga duracin de insuficiencia renal crnica con cifras de creatinina
srica de 4 a 6 mg/100 mi en los ltimos cinco aos e hipertensin de mucho tiempo antes
tratada con diversos productos como diurticos y bloqueadores adrenrgicos. En la
exploracin fsica su presin sangunea es de 170/105 mmHg, su pulso es de 72/min, su
frecuencia respiratoria de 15/min y su temperatura de 37C. En la exploracin subsiguiente
se encuentra cardiomegalia moderada con galope con tercer ruido cardaco (S3) y cuarto
ruido cardaco (S4), as como tumefaccin e hipersensibilidad de la primera articulacin
metatarsiana derecha. En los estudios de laboratorio se encuentra lo siguiente: nitrgeno
ureico sanguneo (BUN) de 63 mg/100 mi; creatinina de 5.1 mg/100 mi; sodio srico de 136
meq/L; potasio srico de 5.9 meq/L; cloruro srico de 100 meq/L; C02 de 19 meq/L y cido
rico de 9.3 mg/100 mi. Cul de las siguientes es la causa ms probable del trastorno de
este paciente?

a)
b)
c)
d)

Nefropata crnica por plomo


Exceso de produccin primaria de cido rico
Nefritis intersticial crnica relacionada con abuso de analgsicos
Nefropata hipertensiva

Allen R. M. MMS Medicina Interna. 5. Edicin. National Medical Series. Mc. Graw Hill.
2006. (captulo 6, parte I: XII B 2 b). La nefropata crnica por plomo es la causa ms
probable del trastorno de este paciente; coexiste con afeccin de la excrecin de cido
rico y tambin con sndrome clnico de gota. Si bien sta y la insuficiencia renal crnica de
otras causas pueden coexistir, las prueba; actuales sugieren que la toxicidad por plomo es
la causa de gran parte de los casos en que concurren ambas enfermedades. El abuso de
analgsicos no debe relacionarse con una mayor incidencia de gota, aunque los dato: renales
pueden ser semejantes. La nefropata hipertensiva tambin puede explicar los datos,
aunque no la aparicin de gota en este paciente. Podra haber hipertensin renovascular en
este enfermo, pero se relaciona con hipertensin ms grave cuando hay insuficiencia renal.
Adems, aqulla coexiste con gota. Sin embargo, se necesitan imgenes del rbol vascular
renal para descartar por completo este diagnstico.

67.- Masculino de 46 aos de edad que inicia con dolor intenso en fosa renal izquierda ante
la sospecha de litiasis renoureteral se realizan estudios de laboratorio y gabinetem, los Rx
demuestran clculos radio lcidos, los cuales estn relacionados a:
a)
b)
c)
d)

Calcio
Estruvita
cido rico
Cistina

-Bruce E. Jarrell, R. Anthony Carabasi, Nacional Medical Series for Independent Study.
Wiliams & Wilkins, 3rd Edition: 451-475.
Los clculos que se pueden presentar en la va urinaria son formados por calcio, cido rico
y cistina, los nicos radiolucidos son los de cido rico que representan un reto diagnstico.

68.- Al existir la sospecha de lquido libre en la cavidad peritoneal y despus de analizar los
hallazgos de manera conjunta de las placas de abdomen y en el ultrasonido abdominoplvico, Dnde se debern de buscar los cambios de densidad de manera inicial?
a) Infradiafragmtico
b) Fondos de saco de Douglas y de Morrison
c) Transcavidad de los epiplones

d) Correderas parieto-clicas

Pedrosa C, Casanova R. Diagnstico por imgen. Mc Graw Hill 2001 p. 209, 210. La
diseminacin del lquido intraperitoneal est producida por la gravedad y la presin
hidrosttica secundaria al movimiento diafragmtico.la pelvis es la parte ms dependiente
de la cavidad peritoneal, su capacidad es de 300 cc, a partir de lo cual el lquido asciende
por las correderas parietoclicas. El lquido plvico desplaza las asas intestinales que
rellenan los espacios plvicos, el leon del lado derecho y el colon sigmoides en el izquierdo,
lo que resulta de la paricin de una densidad homognea en al pelvis menor, en contra del
patrn poco homogneo de las asas intestinales que tienen gas y lquido. En la pelvis
femenina el lquido se acumula en los recesos laterales de la vejiga y al tero, y en el
hombre puede verse en la fosa vesicorrectal y los recesos plvicos laterales. En el
ultrasonido el diagnstico de pequeas cantidades de lquido debe hacerse
fundamentalmente en el saco de Douglas, en la bolsa de Morrison y en el receso
yuxtaesplnico.

69.- En la hiponatremia intensa (inferiores 120 mEq/L se debe de tratar con soluciones
salinas hipertnicas mediante el calculo del dficit, reponiendo de la siguiente manera:

a)
b)
c)
d)

Pasar el dficit de sodio en las primeras 6 hr.


Pasar el dficit de sodio en las primeras 12 hr.
Pasar el dficit de sodio, la mitad en las primeras 12-24 hr y se revalora.
Pasar el dficit de sodio en las primeras 3 hr y despus se revalora.

Farreas. Medicina Interna. Cap. 222, Decimoquinta edicin 2004, Pp. 1844-1847. La
hiponatremia con volumen extracelular disminuido se trata con la administracin de
soluciones salinas isotnicas (al 0.9%). La cantidad necesaria de miliequivalentes de sodio se
calcula de acuerdo a la siguiente formula:
Na (mEq)= Na x agua total corporal, es decir
Na (mEq)= (140- Na actual) x (0.6 x peso en Kg)
Donde:
Agua total corporal= 60% del peso = 0.6 x peso en kg

En la practica, suele administrase la mitad de esa cantidad en el transcurso de las primeras


12- 24 h y luego se efectan reevaluaciones correctoras.

70.- Aunque la mayor parte de los casos de hepatitis vrica aguda cede espontneamente,
pueden sobrevenir complicaciones. Cul de las siguientes aseveraciones describe mejor las
complicaciones de la infeccin por virus de hepatitis B (HBV)?

a) El estado de portador crnico coexiste con mayor riesgo de hepatoma


b) Por lo comn la hepatitis persistente crnica provoca deterioro progresivo del
funcionamiento heptico y debe tratarse enrgicamente
c) La hepatitis activa crnica puede diagnosticarse en dos a cuatro semanas de la
infeccin aguda con HBV
d) La hepatitis activa crnica se caracteriza en la biopsia heptica por infiltrado
linfoctico periportal sin fibrosis o extensin extraportal

Allen R. M. MMS Medicina Interna. 5. Edicin. National Medical Series. Mc. Graw Hill.
2006. (captulo 5IX A1 f (1 )-(3)). El estado de portador crnico del virus de hepatitis B
(HBV) coexiste con mayor riesgo de hepatoma y se encuentra en 0.2% de la poblacin
estadounidense. No es posible diagnosticar hepatitis crnica activa hasta por lo menos seis
meses despus de la infeccin aguda con HBV. En la hepatitis activa crnica, la inflamacin
y la fibrosis rebasan el rea portal y por tanto, se correlacionan con deterioro profuso de
la funcin heptica, que puede resultar en cirrosis o insuficiencia heptica. Se presenta
hepatitis fulminante en alrededor de 1 a 2% de los casos de HBV, hepatitis C (HCV) y
hepatitis no A, no B y no C. Esta complicacin rara por lo regular se relaciona con baja de
las concentraciones de transaminasa conforme es destruido el tejido heptico y disminuye
el tamao del hgado.

71.- Se trata de paciente femenino de la tercera edad que acude por presentar durante
las ltimas 3 semanas astenia, febrcula vespertina, cefalea global y, durante los ltimos
das, le han notado confusin intermitente y somnolencia progresiva. A la exploracin se
observa una temperatura de 38C, somnolencia, dudosa rigidez de nuca y paresia de VI par
derecho. La placa de trax muestra un discreto engrosamiento pleural apical derecho. La
biometra hemtica y qumica sanguinea se encuentra normal. Se realiza una puncin lumbar
con los siguiente resultados en LCR: presin de apertura 170mmHg, protenas 140 mg/dL,

glucosa 42 mg/dL, 270 clulas (70% mononucleares), tincin de gram, de Zielhl-Nielsen, as


como investigacin de antgenos bacterianos son negativos. Que diagnstico es el ms
probable?:

a) Meningitis tuberculosa.
b) Encefalitis herptica.
c) Meningitis bacteriana.
d) Meningitis vrica.

Meningitis TB
Causas, incidencia y factores de riesgo
La meningitis tuberculosa es un trastorno muy poco comn, causado por la diseminacin de
la Mycobacterium tuberculosis al cerebro, desde otro sitio en el cuerpo. Generalmente, sus
sntomas se inician gradualmente. Entre los factores de riesgo se pueden mencionar
antecedentes de tuberculosis pulmonar, consumo de alcohol en forma excesiva, SIDA u
otros trastornos que comprometen el sistema inmunitario.
MANIFESTACIONES CLINICAS.
Los fenmenos clnicos evolucionan en varios periodos o etapas cuya duracin y contenido
semiolgico son muy variables, pero que tienen unas caractersticas generales que permiten
definirlos. En 1948 el British Medical Council defini los tres estadios evolutivos de la
enfermedad.(7)
Estadio I: consciente, sntomas inespecficos, ausencia de afectacin neurolgica.
Estadio II: cierto grado de afectacin de la conciencia, aparicin de signos de afectacin
neurolgica.
Estadio III: afectacin profunda del estado de conciencia, convulsiones, signos de
focalidad neurolgica.
La recuperacin total es la regla en los casos en estadio I mientras que las secuelas y
muerte son muy elevadas en los estadios II y III. En esta ltima etapa el porcentaje de
fallecimientos puede ser superior al 25%.(8,9).
Estadio I
En esta fase, los sntomas son muy inespecficos, sin que puedan atribuirse a alteracin del
SNC, salvo por la aparicin de alteraciones del carcter, que si bien no son especficas si
son bastante constantes.

Fiebre, generalmente de escasa entidad. Otros sntomas de este periodo son cefalea
moderada, apata, prdida de la alegra, anorexia, y con mucha frecuencia en el nio dolores
abdominales, acompaados o no de vmitos, estos ltimos sin caractersticas que sugieran
un origen central. Los sntomas de esta etapa en el lactante pueden pasar desapercibidos.
Estadio II
Junto a la persistencia de otros signos, entre ellos la fiebre, se ponen de manifiesto los
signos de afectacin neurolgica, coincidiendo con los signos menngeos.

Al principio predominan la cefalea y los vmitos, estos ltimos sobre todo en menores de
tres aos. Aparece rigidez de nuca, que puede ser muy intensa, y signos de Kernig y
Brudzinski.
En ms de 1/3 de pacientes con meningitis tuberculosa hay afectacin de pares craneales.
El sexto par, es el que se afecta con ms frecuencia, seguido por los pares III y IV, de
forma uni o bilateral. El facial se afecta ms raramente, as como otros pares craneales.
Puede afectarse el nervio ptico, conduciendo a la atrofia ptica y ceguera.
Las convulsiones son en el nio una manifestacin habitual y precoz de la
enfermedad.(10)Los signos de hipertensin endocraneal se van acentuando, aprecindose
una fontanela tensa en el lactante, aumento del permetro craneal, edema de papila en el
nio mayor, diplopia y visin borrosa.
Loa dficits motores que pueden presentarse a lo largo de la evolucin son secundarios a
los fenmenos vasculares descritos, que conducen a isquemia e infarto. Puede verse
hemipleja por afectacin del territorio de la arteria cerebral media o cartida interna.
Sntomas de afectacin medular pueden existir en casos muy evolucionados, con
presentacin progresiva de parapleja.
Durante este estadio las alteraciones del sensorio aun son discretas, y su empeoramiento
es el que define el paso al siguiente periodo.
Estadio III.
Presidido por la alteracin profunda del estado de conciencia, pasando de la apata e
irritabilidad, a la confusin, estupor y coma profundo en los casos avanzados. Los casos
terminales se caracterizan por coma profundo, rigidez de descerebracin, siendo la muerte
inminente.

Encefalopata tuberculosa.
En los nios hay un cuadro, denominado encefalopata tuberculosa por Udani en 1958 (11),
que se caracteriza por la presencia de signos de afectacin difusa con convulsiones estupor
o coma, y signos moderados de afectacin menngea. El coma sobreviene precozmente, y la
muerte sobreviene entre 1 y 2 meses despus del comienzo, aun con teraputica especfica.
Dominan el cuadro los fenmenos de hipertensin intracraneal.

Anatomopatolgicamente el cuadro est constituido por severo edema de la sustancia


blanca, sin presencia de hemorragias. Microscpicamente hay una prdida difusa de mielina
en la sustancia blanca, atribuyndose la causa a fenmenos de hipersensibilidad frente a la
tuberculoproteinas
Es una infeccin de las meninges, las membranas que recubren el cerebro y la mdula
espinal, causada por la bacteria Mycobacterium tuberculosis que produce la tuberculosis.

Signos y exmenes
Para cualquier paciente con meningitis, es importante llevar a cabo una puncin lumbar:

Tincin del lquido cefalorraqudeo (LCR) positiva para Mycobacterium.

LCR con alto nivel de protenas, bajo nivel de glucosa y aumento en el nmero de
linfocitos.

Cultivo de LCR que muestra la proliferacin de Mycobacterium tuberculosis

Reaccin en cadena de la polimerasa (PCR) que muestra LCR positivo para M.

tuberculosis.

Prueba cutnea positiva para tuberculosis.

Biopsia cerebral o menngea que muestra la presencia de M. tuberculosis.

BIBLIOGRAFIA

1. Sudre P,Dam G,Kochi A. La tuberculose aujourd'hui dans le monde.WHO Bull


1992;70:297-308.
2. Dolin PJ,Raviglione MC,Kochi A. Global tuberculosis incidence and mortality during 19902000.WHO Bulletin 1994;72:213-220.
3. March Ayuela P. Trend in tuberculous meningitis in Barcelona in children aged 0-4
years:correlation with the annual risk of tuberculous infection. Tubercle and Lung Disease
1994;75:423-428.
4. Barry R Bloom.Tuberculosis: Pathogenesis, Protection, and Control. Tuberculosis.
American Society for Microbiology. Washington DC 2005.

5. Anggard E. Nitric oxide: Mediator, murderer ,and medicine. Lancet. 1994;343: 1199-206.
6. Jinkins JR,Gupta R,Hyun Chang K,Rodriguez-Carvajal J.MR imaging of cental nervous
system tuberculosis. Radiol Clin North Am 1995;33:771-786.
7. Medical Research Council.Streptomycin in Tuberculosis Trials Committee: Streptomycin
treatment of tuberculous meningitis.Lancet. 1948;1:582-596.

72.- En la utilizacin de medios de contraste iodados por va intravascular. Cul es la


caracterstica que produce reacciones adversas?
a) La ionicidad
b) La hiperosmolaridad
d) La elevacin de azoados
e) La radiopacidad

Snchez AC, Casanova R. Pedrosa. Diagnstico por imgen. Tratado de Radiologa Clnica.
McGraw Hill. Mxico 1997. p 80. Las reacciones adversas al contraste pueden ser de dos
tipos: yodosincrticas o quimiotxicas. La mayora son de origen yodosincrtico, es decir
anafilcticas y ocurren independientemente de la dosis administrada y la concentracin del
producto. Las de origen quimiotxico se deben a mecanismos fisicoqumicos especficos
sobre los rganos que prefunden, directamente dependientes de la dosis y la concentracin
inyectada. Los factores potencialmente responsables del efecto incluyen: la

hiperosmolaridad, concentracin de cationes y capacidad para ligarse al calcio. La


administracin IV de contraste inico de alta osmolaridad produce 5 a 125 de las
reacciones agudas.

73.- Su tutor en el curso de infectologa de pregrado le pide que revise a un paciente al


cual se le ha diagnosticado endocarditis en la vlvula mitral. Como parte de su exploracin
fsica usted revisa el fondo de ojo en busca de:

a) Ndulos de Osler
b) Mancha de Roth
c) Lesiones de Janeway
d) Hemorragias en astilla

Las manchas de Roth son hemorragias retinales (retinianas) con centro plido o de color
blanco compuestas y rodeadas de fibrina coagulada o cmulos de hemates. Estas manchas
pueden observarse en el fondo de ojo mediante el uso de un oftalmoscopio.1
Las manchas de Roth generalmente son causadas vasculitis mediada por inmunocomplejos
debido a la endocarditis bacteriana, tambin pueden ser observadas en leucemia, diabetes,
anemia perniciosa, isquemia y raramente en retinopata por VIH.
Fowler VG Jr, Scheld WM, Bayer AS. Endocarditis and Intravascular Infections. In:
Mandell GL, Bennett JE, Dolin R, eds. Principles and Practice of Infectious Diseases. 7th
ed. Philadelphia, Pa: Elsevier Churchill Livingstone; 2009; chapt 77.

74.- Masculino de 36 aos acude al servicio por presentar fiebre, escalofros, cefalea,
mialgias y esplenomegalia, como antecedentes refiere que realiz un viaje a Chiapas, se
realiza examen de gota gruesa observando formas asexuadas con tincin de Wright.
El laboratorio reporta, leucopenia con presencia de formas jvenes, trombocitopenia y
elevacin de transaminasas. El tratamiento de eleccin en este paciente es:

a)
b)
c)
d)

Cloroquina
Sulfato de quinina y doxiciclina
Mefloquina
Cloroquina y primaquina

Medidas especficas de prevencin y control


9.1 Tratamiento mdico del paludismo. Se emplea la combinacin de dos medicamentos, la
cloroquina que elimina las formas sanguneas del P. vivax y del P. falciparum, excepto los
gametocitos de este ltimo y la primaquina que elimina los hipnozotos del P. vivax y los
gametocitos del P. falciparum. Se ministrarn diferentes esquemas de acuerdo al objetivo
que se persiga.
9.1.1 Recomendaciones Generales. La cloroquina y primaquina son medicamentos que en
personas sensibles causan:
9.1.1.1 Irritacin gstrica. Siempre debern administrarse cuando la persona a tratar
haya ingerido algn alimento y con abundantes lquidos. Si el paciente vomita 30 minutos
despus de recibirlo, debe recibir una segunda dosis completa. Si vomita entre 30 y 60
minutos despus de una dosis, se le debe dar media dosis adicional. En lo sucesivo pueden
fraccionarse las dosis diarias en varias tomas al da.
9.1.1.2 Primaquina. No debe ministrarse a menores de seis meses de edad, mujeres
embarazadas y madres en periodo de lactancia por sus efectos adversos en hgado y la
interaccin con hemoglobinas fetales; en estos casos se emplea solamente cloroquina a
dosis supresivas cada 21 das y una vez que las condiciones antes mencionadas
desaparezcan, se ministrar tratamiento completo.
9.1.1.3 Control mdico. Cualquier tratamiento antipaldico y en particular para menores de
seis meses de edad, mujeres embarazadas, madres en periodo de lactancia, enfermos
hepticos y otros padecimientos intercurrentes graves, queda siempre sujeta al criterio y
vigilancia mdica.

9.1.2 Tratamiento supresivo. Aquel que se administra slo para eliminar el ciclo
eritroctico; utilizar la cloroquina como medicamento de eleccin en caso de infecciones de
paludismo vivax, y cloroquina ms primaquina en regiones con paludismo falciparum.
9.1.2.1 Indicacin. A toda persona sospechosa de padecer paludismo se le ministra al
momento de tomar la muestra hemtica, una dosis nica de cloroquina a dosis de 10 mg por
kg de peso o segn edad y en reas especiales (de falciparum) se agrega primaquina a dosis
de 0.25 mg por kg de peso (a los menores de 6 meses y embarazadas no se les ministra
este ltimo).
9.1.2.2 Tratamiento supresivo en reas con P. vivax. Se ministra slo cloroquina, indicado
para reas de baja transmisin o hipoendmicas, en las mesoendmicas e hiperendmicas el
tratamiento de primera eleccin es la dosis nica, posterior a la toma de muestra hemtica.

Tabla 5
Tratamiento supresivo en reas con transmisin de P. vivax.
GRUPO DE EDAD

No. COMPRIMIDOS
CLOROQUINA 150 mg

< 6 meses

1/4

6 meses a 1 ao

1/2

2 a 5 aos

6 a 12 aos

13 aos y ms,
3
con menos de 60 kg de peso
13 aos y ms,
con ms de 60 kg de peso

9.1.2.3 Tratamiento supresivo en reas P. falciparum. Al momento de tomar la muestra de


sangre, administrar cloroquina y primaquina a las dosis mencionadas en el apartado 9.1.2.1
Indicacin. La dosificacin se refiere en la "Tabla 6".
Tabla 6
Tratamiento supresivo en reas P. falciparum

GRUPO DE EDAD

No. COMP.
CLOROQUINA
DE 150 mg

No. COMP.
PRIMAQUINA
DE 5 mg

No. COMP.
PRIMAQUINA
DE 15 mg

< 6 meses

1/4

6 meses a 1 ao

1/2

1/2

2 a 5 aos

6 a 12 aos

13 aos y ms,
con menos de 60 kg de peso

13 aos y ms,
con ms de 60 kg de peso

9.1.3 Tratamiento de cura radical (TCR). La cura radical se logra aplicando diferentes
esquemas teraputicos y su efectividad vara de acuerdo a lo referido en la "Tabla 7".

Tabla 7
Tratamiento de cura radical

Esquema

Objetivo/Indicacin

Eficacia
estimada

Dosis nica (TDU) con


cloroquina y
primaquina) con
periodicidad mensual

Eliminar fuentes de infeccin para mosquitos. En


distribucin masiva o a los casos conocidos de
los ltimos tres aos y sus convivientes,
disminuye cargas parasitarias en la poblacin.
til en caso de brotes o en reas especiales de
transmisin persistente y focalizada,
actualmente en reas de alta transmisin se
indica como primera eleccin al momento de
tomar la gota gruesa y sustituye al supresivopresuntivo

< 50%

TCR a 5 das
(cloroquina y
primaquina)

Tratamiento en casos detectados en reas hipo,


meso e hiperendmicas. Puede administrarse en
forma masiva o a casos y convivientes pero no
como medida nica de control

80%

TCR a 14 das
(cloroquina y
primaquina)

Tratamiento alternativo para los casos de reas


hipoendmicas y cuando los dos esquemas de
TCR anteriores no han sido suficientes

90%

Dosis nica (TDU con

Su mayor utilidad es en casos repetidores.

95%

cloroquina y
primaquina) 3x3x3 o
seguimiento por 3
aos que equivale a 18
dosis mensuales cada
8 a 21 das durante 8
tomas

Elimina fuentes de infeccin para mosquitos

Los cuatro esquemas empleados para cura radical mencionados en la "Tabla 7" son:
9.1.3.1 Tratamiento en dosis nica (TDU). Se aplica para disminuir, rpida y
drsticamente la densidad de parsitos circulantes en un rea especfica.
Referencia:
NORMA Oficial Mexicana NOM-032-SSA2-2002, Para la vigilancia epidemiolgica,
prevencin y control de enfermedades transmitidas por vector.

75.- Masculino que es ingresado al servicio de urgencias por haber sufrido quemaduras en
el cuerpo, durante las primeras 24 hrs. despus de la lesin la resucitacin de lquidos en
ste tipo de pacientes incluye:

a)
b)
c)
d)

Infusin inicial de 2 ml de coloides / % quemado / Kg. Peso.


Infusin de cristaloides a razn de mantener un gasto urinario de 1 ml / hr. /
Kg. peso en adultos.
Infusin de cristaloides a razn de obtener 45 ml de orina / hr. en paciente de 28 Kg.
Uso de diurticos para tratar oliguria ( 15 ml de orina / 2 hrs.) en ausencia de
hemocromatgenos urinarios.

REANIMACION HIDRICA

Frmula de Parkland
Se utiliza en las primeras 24 hr.
Con cristaloides

4 ML / KG / % SCT Quemada

La mitad del dficit total en las primeras 8 hrs.


La segunda mitad en las 16 hrs restantes.

Calcular volumen circulante 60cc x kg.

Mantener adecuada PVC ( 8 - 10 )


Mantener TAM > 60 mmHg
Gasto urinario 0.5 - 1.0ml / kg / hr
Evitar administracin excesiva de lquidos
Uresis (2ml / kg / hr )
Na srico c 8 hrs
VIGILAR HIPOTERMIA

76.- Un nio de 6 aos con hemofilia tipo A (actividad del factor VIII del 10%) se da un
golpe en el codo y sufre, como consecuencia, inflamacin e hipersensibilidad en el lugar del
traumatismo. Cul es el tratamiento apropiado?

a)
b)
c)
d)

Infusin de desmopresina
Infusin de plasma fresco congelado
Infusin de concentrado de factor VIII
Inyeccin de vitamina K

Paul H. Dwaorkin. Nacional Medical Series for independent study. Pediatrics. Mc Graw Hill.
4th ed. 2000. Capitulo 15. VI D 2 a (2).

Con un nivel de factor VIII del 10%, el paciente tiene una hemofilia A leve. En ausencia de
hemorragia potencialmente letal o de ciruga mayor, estos pacientes pueden tratarse con
desmopresina (dDAVP), que elevara el nivel de factor VIII dentro del mbito teraputico
sin exponer al paciente al riesgo de infeccin vrica transmitida por la sangre. Aunque el
factor VIII recombinante, tambin est libre de contaminantes virales, son mucho ms
caros que la dDAVP. La vitamina K no tendra utilidad alguna en esta situacin, ya que el
factor VIII no es vitamina k- dependiente.

77.- Hipotona muscular, hiperlaxitud articular, braquicefalia, fisuras palpebrales oblicuas,


puente nasal deprimido, macroglosia relativa, clinodactilia del 5 dedo, persistencia de
conducto arterioso son caractersticas clnicas de:

a)
b)
c)
d)

Trisoma 21
Trisoma 13
Trisoma 18
Trisoma 8

CONCEPTO
El Sndrome de Down (SD), tambin llamado trisoma 21, es la causa ms frecuente de
retraso mental identificable de origen gentico. Se trata de una anomala cromosmica que
tiene una incidencia de 1 de cada 800 nacidos, y que aumenta con la edad materna. Es la
cromosomopata ms frecuente y mejor conocida.

CLNICA
Los nios con SD se caracterizan por presentar una gran hipotona e hiperlaxitud
ligamentosa. Fenotpicamente presentan unos rasgos muy caractersticos.
CABEZA y CUELLO: leve microcefalia cuello es corto.
CARA: los ojos son almendrados, y si el iris es azul suele observarse una pigmentacin
moteada, son las manchas de B r u s h f i e l d. Las hendiduras palpebrales siguen una
direccin oblicua hacia arriba y afuera y presentan un pliegue de piel que cubre el ngulo
interno y la carncula del ojo (epicanto). La nariz es pequea con la raz nasal aplanada. La
boca tambin es pequea y la protusin lingual caracterstica.
Las orejas son pequeas con un helix muy plegado y habitualmente con ausencia del lbulo.
El conducto auditivo puede ser muy estrecho.
MANOS Y PIES: manos pequeas y cuadradas con metacarpianos y falanges cortas
(braquidactilia) y clinodactilia por hipoplasia de la falange media del 5 dedo.
Puede observarse un surco palmar nico.
En el pie existe una hendidura entre el primer y segundo dedo con un aumento de la
distancia entre los mismos (signo de la sandalia).

GENITALES: el tamao del pene es algo pequeo y el volumen testicular es menor que el de
los nios de su edad, una criptorqudia es relativamente frecuente en estos individuos.
PIEL y FANERAS: la piel es redundante en la regin cervical sobretodo en el perodo fetl
y neonatal. Puede observarse livedo reticularis (cutis marmorata) de predominio en
extremidades inferiores. Con el tiempo la piel se vuelve seca e hiperqueratsica.
El retraso mental es constante en mayor o menor grado.

BIBLIOGRAFA:
1. Cooley WC, Graham JM. Down syndrome: an update and review for the primary
pediatrician. Clin Pediatr. 1991;30:233-253.
2. American Academy of Pediatrics. Committee on Genetics Health Guidelines for Children
with Down syndrome.. Pediatrics 1994;93:855-859.
3. Cronk C, Crocker Ac, Pueschel SM, et al. Growth charts for children with Down
syndrome: 1 month to 18 years of age. Pediatrics 1988;81:102-10.

78.- Se trata de recin nacido pretermino de 30 semanas de gestacin, peso de 1200 gr.
Curs con enfermedad de membranas hialinas por lo que se manejo con una dosis de
surfactante y manejo con ventilacin convencional por 3 das. Posteriormente inici con
alimentacin enteral por sonda orogstrica con leche humana, sin tolerancia a la misma
(residuo gstrico y distensin abdominal), existe la sospecha sospecha diagnstica de
enterocolitis necrotizante Los signos y sntomas esperados son:

a)
b)
c)
d)

Sangre macroscpica en heces, clico intestinal, diarrea


Sangre microscpica en heces, ictericia, neumatosis intestinal
Distensin abdominal, doble burbuja gstrica
Distensin abdominal, radiografa normal

La enterocolitis necrotizante (ECN) es una enfermedad grave que afecta a recin nacidos,
en especial prematuros, con una incidencia y morbimortalidad elevados. Constituye la
urgencia gastrointestinal ms frecuente en las UCI neonatales.
Se presenta como un sndrome gastrointestinal y sistmico que comprende sntomas
variados y variables, como distensin e hipersensibilidad abdominal, sangre en heces,
intolerancia a la alimentacin, apnea, letargia, y en casos avanzados acidosis, sepsis, CID y
shock.

El sndrome clnico ha sido clasificado en estadios por Bell y col. (1978) y modificado
por Walsh y Klegman (1986) para incluir hallazgos sistmicos, intestinales y
radiolgicos.

A. Estadio I : sospecha de enterocolitis necrotizante

Los hallazgos sistmicos son inespecficos.

Los hallazgos intestinales incluye el residuo gstrico y heces guayaco positivas.

Los hallazgos radiolgicos son normales e inespecficos.

B. Estadio II A: enterocolitis necrotizante leve

Los hallazgos sistmicos son similares al estadio I.

Los hallazgos intestinales incluyen distensin abdominal prominente con


hipersensibilidad a la palpacin o sin ella, ruidos hidroareos ausentes, sangre
macroscpica en materia fecal.

Los hallazgos radiolgicos, leo con asas dilatadas con reas focales de neumatosis
intestinal.

C. Estadio II B: enterocolitis necrotizante moderada

Los hallazgos sistmicos incluyen acidosis leve y trombocitopenia.

Los hallazgos intestinales incluyen edema de la pared abdominal e hipersensibilidad


a la palpacin con una masa palpable o sin ella.

Los hallazgos radiolgicos incluyen neumatosis extensa y ascitis temprana.

Puede haber gas en la vena porta intraheptica.

D. Estadio IIIA: enterocolitis necrotizante avanzada:

Los hallazgos sistmicos incluyen acidosis respiratoria y metablica, ventilacin


asistida por apnea, hipotensin arterial, oliguria, neutropenia y coagulacin intravascular
diseminada.

Los hallazgos intestinales incluyen edema que disemina, eritema e induracin del
abdomen.

Los hallazgos radiolgicos incluyen ascitis prominente y asa centinela persistente


sin perforacin.

E. Estadio IIIB: enterocolitis necrotizante avanzada:

Los hallazgos sistmicos revelan signos vitales e ndices de laboratorio en


deterioro, sndrome de shock y desequilibrio electroltico.

Los hallazgos intestinales y radiolgicos muestran evidencias de perforacin.

Hallazgos radiolgicos en la radiografa de abdomen

Distensin abdominal generalizada

leo paraltico

Neumatosis intestinal quistoide (patognomnico)


La neumatosis intestinal quistoide se la puede visualizar de varias maneras, cmulos de gas
lineales, curvilneos, esponjosos y espumosos.
Este ltimo debe distinguirse de materia fecal o meconio mezclado con aire.
Cualquier lactante con sospecha de enterocolitis necrotizante en el que se encuentren
radiogrficamente colecciones de aire lineales, curvilneas esponjosas o espumosas
debe considerarse que tiene neumatosis intestinal quistoide hasta que se demuestre lo
contrario.
La neumatosis intestinal quistoide suele verse con mayor frecuencia en el colon, pero puede
ocurrir desde estomago hasta recto.

1.

2.

3.

4.
5.
6.

7.

Hartmann G. E., Drugas G. T., Shochat S. J. Post-necrotizing enterocolitis


strictures presenting with sepsis of perforation: risk of clinical observation. J.
Pediatr. Surg. 1988; 23: 562-6.
Kosloske A. M., Burstein J., Bartow S. A. Intestinal obstruction due to colonic
stricture following neonatal necrotizing enterocolitis. Ann Surg. 1980 Aug;192 (2):
202-7.
Schwartz M. Z., Hayden C. K., Richardson C. J., Tyson K. R., Lobe T. E. A
prospective evaluation of intestinal stenosis following necrotizing enterocolitis. J.
Pediatr. Surg. 1982 Dec; 17 (6): 764-70.
Bell M. J., Ternberg J. L., Askin F. B. Intestinal stricture in necroting enterocolitis.
J. Pediatr. Surg. 1976; 11: 319-27.
Pokorny W. J., Harr V. L., McGill, C. W., et al; Intestinal stenosis resulting from
necrotizing enterocolitis. Am J. Surg 1981 42: 721-724.
Schimpl G., Hollwarth M. E., Fotter R., Becker H. Late intestinal strictures
following successful treatment of necrotizing enterocolitis. Acta Paediatr. Suppl.
1994; 396: 80-3.
Btter A., Flageole H., Laberge J. M. The Changing face of Surgical Indication for
Necrotizing Enterocolitis J. Pediatr. Surg. 2002; 37: 469- 499.

Gobet R. , Sacher P. , Schwobel M. G. Surgical procedures in colonic strictures after


necrotizing enterocolitis. Acta Paediatr. Suppl. 1994;396:77-9.

79. - Which one of the following conditions results in prologation of the partial
thromboplastin time (PTT), but not the prothrombin time (PT)?
a)
b)
c)
d)

Varicela hemorrhage as a result of cirrosis


Menorrhagia resulting from von Willebrands disease
Therapy with broad-spectrum antibiotics
Therapy with coumarin for phlebitis

Enf. Von Willebrand

Manifestaciones

9
9
9
9
9
9

Epistaxis

60 %

Hemorragia transvaginal

50 %

Equimosis

40 %

Gingivorragias

35 %

Hematomas

5%

Hemartrosis

3%

EvW
Pruebas de escrutinio
T. Hemorragia

Prolongado

C. Plaquetaria

Normales, excepto 2B

TTPa

Normal o prolongado

TP

Normal

Gpo AB0

25 % bajo en 0

1.-Lee GR, Foerster J, Lukens J, Paraskevas F, Greer JP and Rodgers GM.


Wintrobes clinical haematology; 10th Edition, Lippincott Williams & Wilkins, United
States of America, 1999.

2.-Williams WJ. Manual Williams de hematologa


5a Edicin McGraw-Hill Interamericana, Mxico, 1997.

3.-Beutler E, Lichtman MA, Coller BS, Kipps T. Hematology. 5th International


Edition. United Stated of America, 1995.

4.-Ruiz Argelles GJ. Fundamentos de hematologa; 2. Edicin, Editorial Mdica


Panamericana, Mxico, 1998.

80.- Paciente obesa, con fractura de cadera que ser intervenida quirrgicamente para
reemplazo total de cadera se considera:

a)
b)
c)
d)

Paciente de riesgo bajo para trombosis venosa profunda.


Paciente de riesgo alto para trombosis venosa profunda.
Paciente con riesgo de trombosis arterial.
Paciente con riesgo muy bajo de TEP.

La fisiopatologa de la TVP se resume mediante la trada de Virchow: Estasis sanguneo,


dao endotelial e hipercoagulabilidad. Estas 3 circunstancias aisladamente o en asociacin,
intervienen en el desarrollo de un trombo. Los factores de riesgo enumerados a
continuacin aumentan la probabilidad de desarrollar trombosis mediante uno o ms de los
mecanismos de la trada (Kahn S, 1998):

1. Ciruga mayor, especialmente la ciruga ortopdica, pero tambin los pacientes


sometidos a ciruga abdominal, neurociruga.
2. Neoplasias malignas, el riesgo aumenta en los pacientes que reciben quimioterapia
activa.

3.
4.
5.
6.

Infarto Agudo de Miocardio.


Sndrome Nefrtico.
Ictus isqumico, preferentemente en el miembro hemipljico.
Inmovilizacin prolongada. A mayor tiempo de inmovilizacin mayor riesgo, aunque
se ha documentado una incidencia de TVP del 13% en pacientes encamados durante
8 das.
7. TVP o Embolismos Pulmonares (EP) previos. El riesgo se debe probablemente a la
persistencia de obstruccin al flujo y/o dao en las vlvulas venosas tras la TVP
anterior.
8. Embarazo y postparto.
9. Anticoncepcin oral y Terapia Hormonal Sustitutiva (THS).
10. Alteraciones congnitas o adquiridas que produzcan hipercoagulabilidad:
o Deficiencia congnita de Protena C, Protena S y Antitrombina III.
o Resistencia a la Protena C activada: Esta alteracin se encuentra en el 5%
de la poblacin general y en el 20-40% de los enfermos con TVP.
o Hiperhomocisteinemia
o Disfibrinogenemia
o Presencia de Anticuerpos Antifosfolpido

Estos trastornos se asocian a TVP recurrentes, o en localizaciones atpicas o de aparicin a


edades tempranas (antes de los 45 aos).

Bibliografa:
1.

Ebell MH. Evaluation of the patient with suspected deep vein thrombosis. J Fam
Pract. 2001 Feb;50(2):167-71 [PubMed] [Texto completo]

2. Gabriel Botella F, Labis Gmez M, Bras Aznar JV. Trombosis venosa profunda:
presente y futuro. Med Clin 2000; 114: 584-596. [PubMed]
3. Gorman WP, Davis KR, Donnelly R. ABC of arterial and venous disease. Swollen limb1: General assessment and deep vein thrombosis. BMJ 2000; 320: 1453-1456.
[PubMed] [Texto completo]
4. Green L, Fay W, Harrison V, Kleaveland M, Wahl R, Wakefield T, Weg J, Williams
D. Venous thromboembolism (VTE) [Internet]. Ann Arbor (MI): University of
Michigan Health System 2004
[acceso 18/3/2007]
Disponible en:
http://cme.med.umich.edu/iCME/vte04/ [NGC]

Potrebbero piacerti anche